Sunteți pe pagina 1din 823

OITE 2006

Kimberlly Chhor

1. Which of the following arteries provides the single dominant proximal blood supply to the medial head of the gastrocnemius muscle? 12345Anterior tibial Posterior tibial Peroneal Genicular Sural

This is relevant to the anatomy of myocutaneous gastrocnemius flaps in upper tibial defect reconstructions. Each head of the gastrocnemius is supplied by the sural artery, a branch of the popliteal artery. The sural artery passes directly into the proximal portion of both heads where it arborizes and then travels longitudinally along the length of the muscle. The anterior tibial artery passes from behind above the interosseous membrane into the anterior compartment to supply the muscles of the anterior compartment The posterior tibial artery, from the popliteal artery, enters the posterior compartment supplying the muscles of the posterior comparment and giving off its largest branch, the peroneal artery. In reference to flaps, it supplies the distal portion of the soleus muscle which can be used defects in the mid-tibia. Proximally, the soleus receives its blood supply from the peroneal artery. The genicular arteries, branches of the popliteal artery, include the superior and inferior medial geniculate and the superior and inferior lateral geniculate which supply the structures around the knee and patella, and middle geniculate artery which supply the cruciates and the synovium. The supreme genicular artery, a branch of the femoral artery, supplies the vastus medialis before anastomosing with the other genicular arteries.

2. Large femoral heads, compared to small femoral heads, produce what effect in total hip arthroplasty? 1- Smaller wear surface 2- Decreased polyethylene modulus 3- Increased contact stress 4- Increased range of motion to impingement 5- Increased joint constraint

Preferred Response: 4
Increased range of motion to impingement

RECOMMENDED READING: Amstutz HC, Le Duff MJ, Beaule PE: Prevention and treatment of dislocation after total hip replacement using large diameter balls. Clin Orthop 2004;429:108-116.

Increasing the diameter of the head has several advantages: (1) ROM before neck-shell impingement increases as the ratio of head diameter to neck diameter increases; (2) because the depth of a hemispherical acetabular liner is equal to the radius of the femoral head, the latter must be displaced by a distance equal to its radius before the hip can dislocate, should there be any component or bone to pelvis impingement (for example, a 28mm head has only 14 mm to travel before dislocating, compared with 22 mm for a 44-mm head); (3) the larger head in combination with a thin-walled socket enhances capsular stability; and (4) should dislocation occur, revision may not be required as with a constrained socket. With regards to wear surface, increased volumetric wear of the PE liner is a concern when using a large diameter head. Although PE wear will increase with increased head size even with the newer cross-linked PE, the benefit of increased stability from a big femoral head may outweigh the risk of increased wear and osteolysis. However, the larger head sizes with a metal-on-metal bearing combination produce less wear than smaller head sizes and, depending on the patients size, even larger head sizes, up to 56 mm, are possible.

3.What is the occupational risk of HIV infection following percutaneous exposure (ie, needle stick) from infected blood?
12345Less than 1% 5% 10% 25% Greater than 30%

PREFERRED RESPONSE: 1
Less than 1%

RECOMMENDED READINGS: American Academy of Orthopaedic Surgeons Advisory Statement on Preventing the Transmission of Bloodborne Pathogens. http://aaos.org/wordhtml/papers/advistmt/1018.htm Cardo DM, Culver DH, Ciesielski CA, et al: A case-control study of HIV seroconversion in health care workers after percutaneous exposure. Centers for Disease Control and Prevention Needlestick Surveillance Group. N Engl J Med 1997;337:1485-1490.

The average risk of transmission of human immunodeficiency virus (HIV) to a health care worker after percutaneous exposure to HIV-infected blood has been estimated as 0.3 percent. Increased risk was associated with three factors that were probably indirect measures of the quantity of blood transferred in the exposure: deep injury, injury with a device that was visibly contaminated with the source patient's blood, and a procedure that involved a needle placed in the source patient's vein or artery, which means that the needle probably contained undiluted blood. The risk of HIV transmission was also increased if a health care worker was exposed to blood from a source patient in the terminal stage of AIDS. This association is probably due to the higher titer of HIV in the blood of patients late in the course of AIDS, but it could possibly be due to other factors, such as syncytiuminducing HIV strains in these patients.

4. A 14-year-old boy injured his right shoulder while playing football yesterday. His mother reports that he had trouble breathing overnight. A radiograph and a CT scan are shown in Figures 1a and 1b. Treatment should consist of 1sling immobilization for 3 weeks, followed by early range of motion. 2- closed reduction under conscious sedation in the emergency department, followed by sling immobilization. 3- closed reduction and percutaneous pinning with Kirschner wires in the operating room. 4- open reduction with thoracic surgery back-up and screw stabilization. 5- open reduction with thoracic surgery back-up and soft-tissue reconstruction.

Image 1a 1b

PREFERRED RESPONSE: 5
Open reduction with thoracic surgery back-up and soft-tissue reconstruction

RECOMMENDED READINGS: Waters PM, Bae DS, Kadiyala RK: Shortterm outcomes after surgical treatment of traumatic posterior sternoclavicular fracture-dislocations in children and adolescents. J Pediatr Orthop 2003;23:464-469. Wirth MA, Rockwood CA: Acute and chronic traumatic injuries of the sternoclavicular joint. J Am Acad Orthop Surg 1996;4:268-278.

The medial clavicular epiphysis does not ossify until the 18th to 20th year and it fuses to the with the shaft of the clavicle in the 23rd to 25th year. Many sternoclavicular dislocations are injuries of the medial physis. As with posterior sternoclavicular joint dislocations, prompt closed or open reduction, as posterior displacement of the medial clavicle has been associated with numerous complications, including respiratory distress, venous congestion or arterial insufficiency, brachial plexus compression, and myocardial conduction abnormalities. They recommend operative fixation for patients under 25 years of age with symptoms of the posteriorly displaced clavicle, i.e. trouble breathing overnight. A myriad of procedures have been recommended for repair or reconstruction of the sternoclavicular joint. On the basis of the authors' experience and review of the literature, they advocate surgical resection of the medial clavicle, with maintenance, repair, or reconstruction of the costoclavicular ligaments, when surgery is indicated. Metallicpin fixation of the joint should be avoided, as Steinmann pins, Kirschner wires, threaded pins with bent ends, and Hagie pins have all been reported to migrate and cause serious complications, including death.

5. A right-hand dominant 30-year-old softball player reports clicking and intermittent pain at his right sternoclavicular joint. Examination reveals a moderate prominence, mild pain with palpation, and no local erythema or warmth. The left side is normal. General examination is remarkable for bilateral knee recurvatum and an absence of severe acne. Radiographs are unremarkable. What is the most likely diagnosis? 1- Septic arthritis 2- Atraumatic subluxation 3- Sternoclavicular hyperostosis 4- Friedreichs disease 5- Condensing osteitis

PREFERRED RESPONSE: 2
Atraumatic subluxation

RECOMMENDED READINGS: Higginbotham TO, Kuhn JE: Atraumatic disorders of the sternoclavicular joint. J Am Acad Orthop Surg 2005;13:138-145. Rockwood CA Jr, Odor JM: Spontaneous atraumatic anterior subluxation of the sternoclavicular joint. J Bone Joint Surg Am 1989;71:1280-1288.

Spontaneous atraumatic anterior subluxation of the sternoclavicular joint may occur during overhead elevation of the arm. Affected patients are generally in their teens or twenties, and many demonstrate signs of generalized ligamentous laxity on physical examination. Patients report a sudden subluxation of the medial end of the clavicle, and many remember feeling an associated pop. The majority of cases are not painful, and the subluxation usually reduces with lowering the arm. In a review of 37 patients with spontaneous anterior subluxation of the sternoclavicular joint, subluxations were reproducible and painless in 29 patients. Eighty percent of the patients demonstrated evidence of generalized ligamentous laxity. Isolated septic arthritis of the sternoclavicular joint is uncommon and frequently is associated with an underlying disease or other risk factors. Conditions known to be associated with infectious arthritis are RA, sepsis, infected subclavian central lines, alcoholism, human immunodeficiency virus (HIV) infection, immunocompromised status, renal dialysis, and intravenous drug abuse. Pain, swelling, and tenderness over the sternoclavicular joint, in association with fever, chills, or night sweats, are usual. Sternocostoclavicular hyperostosis, also known as intersternocostoclavicular ossification or

pustulotic arthro-osteitis, is a rare disorder that presents with soft-tissue ossification and hyperostosis between the clavicles. The anterior portions of the upper ribs and sternum, the distal femur and tibia, and the vertebral bodies also may be involved. The etiology of this disorder is unknown, but it is often encountered in association with palmoplantar pustolosis and severe acne. Radiographs demonstrate hyperostosis of the sternum, clavicles, and upper ribs as well as ossification of the costoclavicular, costosternal, and intercostal ligaments.

Aseptic osteonecrosis of the medial clavicle, also called Friedrichs disease, is a rare condition characterized by discomfort, swelling, and crepitus of the sternoclavicular joint in the absence of trauma, infection, or other symptoms. Patients may report loss of ipsilateral shoulder motion. Plain radiographs demonstrate irregularity or curved deformation of the medial end of the clavicle.
Condensing osteitis is a rare condition characterized by sclerosis and enlargement of the medial end of the clavicle with preservation of the sternoclavicular joint. Although its etiology is unknown, chronic mechanical stress at the sternoclavicular joint may play a role. Patients with this disorder are usually women in their late childbearing years. Involvement typically is unilateral, and pain and swelling over the affected area, exacerbated by shoulder abduction, is a typical presentation. Radiographs demonstrate sclerosis and enlargement of the medial clavicle; isolated increased uptake in this region is found on bone scan. MRI and CT scans will demonstrate obliteration of the marrow space.

6. Arch height is maintained during the stance phase of gait primarily by 1- Achilles tendon contraction. 2- posterior tibial tendon contraction. 3- bony and ligamentous structures. 4- unlocking of the transverse tarsal joints. 5- balanced contraction of the peroneus longus and anterior tibialis.

PREFERRED RESPONSE: 3
Bony and ligamentous structures

RECOMMENDED READINGS: Erdemir A, Hamel AJ, Fauth AR, et al: Dynamic loading of the plantar aponeurosis in walking. J Bone Joint Surg Am 2004;86:546-552. Nyska M, Parks BG, Chu IT, et al: The contribution of the medial calcaneal osteotomy to the correction of flatfoot deformities. Foot Ankle Int 2001;22:278-282. Deland JT, de Asla RJ, Sung IH, et al: Posterior tibial tendon insufficiency: Which ligaments are involved? Foot Ankle Int 2005;26:427-435.

Although posterior tibial tendon dysfunction is widely accepted as a significant contributor to flatfoot deformity, the pathology and deformity involve more than the posterior tibial tendon.8Basmajian and Stecko,in an EMG study of standing patients, noted that no deformity occurred in the arch when the posterior tibial tendon was inactive. The superomedial component of the spring ligament ranges from appearing completely disrupted to normal. Most of the other ligaments that support the arch include the plantar fascia, the long and short plantar ligaments, the talocalcaneal interosseous ligament, and the plantar ligaments of the naviculo-cuneiform and tarsometatarsal joints. Additionally the plantar aponeurosis is known to be a major contributor to arch support by transmitting large forces between the hindfoot and forefoot during the stance phase of gait. The plantar aponeurosis originates from the medial tubercle of the calcaneus and inserts into the phalanges through a complex network of fibrous tissue. Hicks described the function of the plantar aponeurosis as being analogous to a windlass mechanism, in which the arch of the foot elevates by winding of the plantar aponeurosis around the heads of the metatarsals during toe extension.

Bryan Ding Questions 7-12

Question 7
7. A 12-year-old patient has an epiphyseal lesion. What is the most likely diagnosis? 1- Giant cell tumor 2- Aneurysmal bone cyst 3- Eosinophilic granuloma 4- Dysplasia epiphysealis hemimelica 5- Chondroblastoma

Question 7
7. A 12-year-old patient has an epiphyseal lesion. What is the most likely diagnosis? 1- Giant cell tumor 2- Aneurysmal bone cyst 3- Eosinophilic granuloma 4- Dysplasia epiphysealis hemimelica 5- Chondroblastoma

Q7
PREFERRED RESPONSE: 5 Chondroblastomas are a cartilaginous lesion associated with giant cells. This tumor usually occurs in the epiphysis of long bones, especially the proximal humerus, distal femur, and proximal tibia. Between 50-75% of chondroblastomas occur in the second decade of life. Because chondroblastoma occurs in the epiphysis and this site is intra-articular, patients frquently present with a painful joint and synovitis. Multinucleated giant cells are a common feature. A closely related tumor is clear cell chondrosarcoma, which is radiologically similar including calcification, but these occur in adults with closed physes and can be histologically distinguished. Chondroblastomas occaisionally locally recur after curettage and have been known to undergo benign pulmonary metastasization. GCTs are found slightly more in females, occurs most frequently about the knee, distal radius and proximal humerus in the juxtarticular location in epiphysis and metaphysic; usually 3rd to 5th decade of life. ABCs are found slightly more in females, during the first 3 decades of life, can occur in any bone, most commonly in femur, tibia, pelvis or vertebral column; pain is the most frequent complaint, and the radiologic hallmark is ballooned out appearance and eccentricity. EGs favor flat marrow-bearing bones, can present at almost any age, but usually in the first 2 decades of life, usually dxd as inflammatory lesions initially, frequently involveing the diaphysis of the bone with periosteal reaction. DEH is a rare disorder usually in males (3:1) children and young adolescents, commonly seen in the knee, talus, and tarsal navicular and first cuneiform joints (usually medial side of the epiphysis). RECOMMENDED READING: Simon MA, Springfield DS (eds): Surgery for Bone and Soft-tissue Tumors. Philadelphia, PA, Lippincott-Raven, 1998, pp 190-191.

Question 8
8. Two years following open reduction and internal fixation of a pilon fracture, which of the following parameters correlates most closely with a poor clinical outcome and inability to return to work? 1- Metaphyseal comminution 2- Quality of reduction 3- Time to surgery 4- Lower level of education 5- Open fracture

Question 8
8. Two years following open reduction and internal fixation of a pilon fracture, which of the following parameters correlates most closely with a poor clinical outcome and inability to return to work? 1- Metaphyseal comminution 2- Quality of reduction 3- Time to surgery 4- Lower level of education 5- Open fracture

PREFERRED RESPONSE: 4 The Williams et al study surveyed 29 pts with 32 pilons for severity of injury and accuracy of articular reduction. They measured their outcomes using: a radiographic arthrosis score, a subjective ankle score, SF-36, and the patients ability to return to work. While radiographic arthrosis was predicted best by severity of injury and quality of reduction, they had no significant relationship to subjective score, SF-36, or return to work, which were all influenced by patient-specific socioeconomic factors, including level of education. Similarly the Pollack study showed worse outcomes with 2 or more comordibities, being married, having an annual personal income <$25,000, lack of a high school diploma, and having been treated in an exfix. So in the end, if you were guessing like I wasyou could probably just blame it on the patient. Again. (According to Fulkerson, stupidity should hurtmaybe it really does.) RECOMMENDED READINGS: Williams TM, Nepola JV, DeCoster TA, et al: Factors affecting outcome in tibial plafond fractures. Clin Orthop 2004;423:93-98. Pollak AN, McCarthy ML, Bess RS, et al: Outcomes after treatment of highenergy tibial plafond fractures. J Bone Joint Surg Am 2003;85:1893-1900.

Question 9
9. Based on the findings shown in the sagittal T1-weighted MRI scan seen in Figure 2, what is the most likely diagnosis? 1- Ganglion cyst 2- Posterior cruciate ligament tear 3- Medial meniscus tear 4- Osteochondral defect 5- Normal findings

Question 9
9.Based on the findings shown in the sagittal T1-weighted MRI scan seen in Figure 2, what is the most likely diagnosis? 1- Ganglion cyst 2- Posterior cruciate ligament tear 3- Medial meniscus tear 4- Osteochondral defect 5- Normal findings

Q9
PREFERRED RESPONSE: 3 The diagnosis is a medial meniscus tear. Things to look for according to Dorsay et al to diagnose a bucket-handle meniscal tear: absent bow tie sign (sens 88.4%), presence of at least one of the displaced fragment signs (sens 90.7%), both of the aforementioned signs (spec 85.7%), double PCL sign (spec 100%), anterior flipped meniscus sign (spec 89.7%). And if the words above didnt paint a clear enough picture in your mind, the other cited 239 pages of textbook below will hopefully have enough pictures to better describe what all those things look like. RECOMMENDED READINGS: Dorsay TA, Helms CA: Buckethandle meniscal tears of the knee: Sensitivity and specificity of MRI signs. Skeletal Radiol 2003;32:266-272. Stoller DW, Cannon WD, Anderson LJ (eds): The Knee: Magnetic Resonance Imaging in Orthopaedics and Sports Medicine, ed 2. Philadelphia, PA, JB Lippincott, 1997, pp 203-442.

Question 10
10. A 14-year-old boy reports the insidious onset and gradual worsening of activity-related local low back pain over the last 6 months. Radiographs are unremarkable, but a bone scan shows increased uptake at L5-S1. A representative cut from the CT scan at L5-S1 is shown in Figure 3. Management should consist of 1- vigorous physical therapy. 2- excision of the lesions. 3- bracing with a molded lumbosacral orthosis. 4- posterolateral L5-S1 arthrodesis. 5- an epidural steroid injection.

Question 10
10. A 14-year-old boy reports the insidious onset and gradual worsening of activity-related local low back pain over the last 6 months. Radiographs are unremarkable, but a bone scan shows increased uptake at L5-S1. A representative cut from the CT scan at L5-S1 is shown in Figure 3. Management should consist of 1- vigorous physical therapy. 2- excision of the lesions. 3- bracing with a molded lumbosacral orthosis. 4- posterolateral L5-S1 arthrodesis. 5- an epidural steroid injection.

Q10
PREFERRED RESPONSE: 3 The diagnosis is spondylolysis, which in children is almost always seen between L5/S1. Tx for an acute spondylolysis would be a thermoplastic lumbosacral orthosis worn full-time for 6 months, to allow the pars defect to heal. If the pars defect is chronic, the brace may be worn until resolution of sx. Exercises are started when the patient is pain-free (e.g. hamstring stretches, pelvic tilts, abdominal strengthening), and athletic activity can be done with or without a low-profile antilordotic brace. Patients usually present with pain localized to the low back aggravated by extension activities and relieved by rest. Radiation of pain is uncommon, and neurologic deficit is rare. The problem is a defect of one or both of the pars interarticularis, generally as a result of repetitive stress to the area, like activities which involve hyperextension of the lumbar spine. The 2 most common types in children are isthmic (more common, fatigue fx of the pars) and dysplastic (congenital dysplasia of the facets, more prone to neuro sx and deformity during growth). As discussed above, PT and activities are resumed after pain sx are resolved. Surgical options are pursued with failure of conservative (in this case non-op) mgmt with a desire to not stop/modify the offending activity. Steroids would likely be more indicated for a dx that caused neuro deficits. RECOMMENDED READINGS: Vaccaro AR (ed): Orthopaedic Knowledge Update 8. Rosemont, IL, American Academy of Orthopaedic Surgeons, 2005, pp 775-787. Hammerberg KW: Spondylolysis and spondylolisthesis, in DeWald RL (ed): Spinal Deformities: The Comprehensive Text. New York, NY, Thieme, 2003, pp 787-800.

Question 11
11. The lateral pillar classification system of Legg-Calve-Perthes disease is determined from an AP radiograph of the pelvis made in what stage of the disease? 1- Increased density 2- Fragmentation 3- Reossification 4- Healed 5- Remodeled

Question 11
11. The lateral pillar classification system of Legg-Calve-Perthes disease is determined from an AP radiograph of the pelvis made in what stage of the disease? 1- Increased density 2- Fragmentation 3- Reossification 4- Healed 5- Remodeled

Q11
PREFERRED RESPONSE: 2 No real discussion here. Quote JBJS 86:10, page 2105 (Oct 2004), The lateral pillar classification was determined from anteroposterior radiographs of the pelvis made in the early fragmentation stage of the disease. If all else fails, just pick B. RECOMMENDED READINGS: Skaggs DL, Tolo VT: Legg-Calve-Perthes Disease. J Am Acad Orthop Surg 1996;4:9-16. Herring JA, Kim HT, Browne R: Legg-Calve-Perthes Disease: Part I. Classification of radiographs with use of the modified lateral pillar and Stulberg classifications. J Bone Joint Surg Am 2004;86:2103-2120

Question 12
12. A 38-year-old man underwent a below-knee amputation 18 months ago and reports that during the heel strike to foot flat phase of gait the knee remains extended and the front of the foot does not strike the ground until late into midstance. What is the most likely cause? 12345Foot is too anterior SACH heel is too rigid Heel on shoe is too high Excessive socket flexion (foot is too dorsiflexed) Excessive use of knee flexors (abnormal gait pattern)

Question 12
12. A 38-year-old man underwent a below-knee amputation 18 months ago and reports that during the heel strike to foot flat phase of gait the knee remains extended and the front of the foot does not strike the ground until late into midstance. What is the most likely cause? 12345Foot is too anterior SACH heel is too rigid Heel on shoe is too high Excessive socket flexion (foot is too dorsiflexed) Excessive use of knee flexors (abnormal gait pattern)

Common problems with prosthetics in BKAs causing too much knee hyperextension at heel off: foot too anterior or plantar flexed (correct answer here), too soft heel cushion (knee flexes when the heel is fully compressed) [not answer 2], insufficient flexion of the socket or posterior displacement of the socket [not answer 4], too long a keel of a SACH (Solid Ankle Cushioned Heelthe standard for the past few decades) foot. Heel too high on shoe would give you a limb length discrepancy forcing your knee into flexion [not answer 3]. And how can using too much knee flexor give you too much knee extension unless you did some undescribed muscle transfer during your BKA [not answer 5]? RECOMMENDED READINGS: Bowker JH, Michael JW (eds): Atlas of Limb Prosthetics: Surgical, Prosthetic, and Rehabilitation Principles, ed 2. Rosemont, IL, American Academy of Orthopaedic Surgeons, 2001, pp 453-478. Blumentritt S, Schmalz T, Jarasch R, et al: Effects of sagittal plane prosthetic alignment on standing trans-tibial amputee knee loads. Prosthet Orthot Int 1999;23:231-238.

Q12 PREFERRED RESPONSE: 1

David Epstein

13. During placement of an iliosacral screw into the body of S1, what pelvic radiographic view should be obtained to help avoid entering the S1 neuroforamen? 1- AP 2- Lateral 3- Inlet 4- Outlet 5- Obturator oblique

PREFERRED RESPONSE: 4 RECOMMENDED READINGS: Routt ML Jr, Nork SE, Mills WJ: Percutaneous fixation of pelvic ring disruptions. Clin Orthop 2000;375:15-29. Routt ML Jr, Simonian PT, Mills WJ: Iliosacral screw fixation: Early complications of the percutaneous technique. J Orthop Trauma 1997;11:584-589.
In his prospective study evaluating 244 percutaneus SI screws Routt used high quality triplanar (Inlet, Outlet, Lateral) flouroscopic imaging with no cases of S1 nerve root postoperative peripheral exam change with 1 year follow up. Five screw insertions were misplaced because of a poor understanding of upper sacral anatomy, one of which led to a transient L5 nerve root neuropraxia. Therefore, he recommends using lateral imaging and CT to vizualize the degree of sacral alar slope and upper sacral morphology. He states that simple orthogonal imaging is insufficient to guide safe SI screw insertions. Although obtaining a lateral image is the take home point of the reference, the answer to the question above is the Outlet view. To avoid entering the S1 neuroforamen, the outlet view should be used. It is obtained by placing the pt supine on the OR table and positioning the beam 35 degrees cephalad so that the xray beam is perpendicular to the plane of the sacrum. This view is then used to guide the pin cephalad to the S1 nerve root foramen. Iliosacral screws can be inserted using flouroscopic guidance with the pt in the supine or prone position. The screw starting point and direction, and the upper sacral morphologic characteristics are important components of inserting the screw safely. The SI screws should be inserted perpendicular to the sacroiliac articulation while the chondral surfaces are avoided.

14. A 25-year-old woman sustains an injury to the tip of her ring finger in a meat slicing machine. Examination reveals volar tissue loss only, with the insertion site of the flexor digitorum profundus tendon exposed. Coverage of the wound is best achieved with what type of flap? 1- V-Y advancement 2- Thenar 3- Neurovascular island 4- Cross-finger 5- Axial flag

PREFERRED RESPONSE: 4 RECOMMENDED READINGS: Trumble TE (ed): Hand Surgery Update 3: Hand, Elbow & Shoulder. Rosemont, IL, American Society for Surgery of the Hand, 2003, pp 479-492. Kappel DA, Burech JG: The cross-finger flap: An established reconstructive procedure. Hand Clin 1985;1:677-683. The above question describes a ring finger injury with volar tissue loss and FDP tendon exposure. The VY flap cannot be used for wounds with primarily volar tissue loss and is reserved for transverse tip injuries or dorsal>volar injuries. A Thenar flap can be used for volar tip injuries but is primarily used for volar tissue loss of the index and middle finger. Neurovascular island flaps are used for more extensive defects and are particularly useful for restoring sensate tissue to the opposing surface of the thumb or index finger. The Axial Flag flap is useful for proximal defects on the dorsum of digits that can not be covered with the above flaps as well as proximal palmar digital defects. The Cross Finger flap is a heterodigital random interpolation flap that is taken from the dorsum of an adjacent finger and applied to the defect. It typically remains attached to the defect for approximately 2 weeks before the flap base is divided. The flap donor site is skin grafted at the time of the first procedure. It is used for wounds with primarily volar tissue loss and would therefore be the best answer from the above choices. The Kappel article confirms that the cross finger flap is reliable and flexible in its application and that patients usually report their flaps to be functional, durable, and free of pain. Cold intolerance, as with other methods of reconstruction, remains a problem. Sensibility in the flaps proved to be functional in the majority of patients. The preservation of length and range of motion is reflected in the fact that disability settlement was a rare occurrence.

15. A 60-year-old woman reports pain and limited shoulder motion after falling 1 year ago. A CT scan is shown in Figure 4. Management should consist of 1- closed reduction under anesthesia, followed by immobilization in an abduction orthosis. 2- open reduction, followed by immobilization in an abduction orthosis. 3- proximal humeral arthroplasty. 4- transfer of the lesser tuberosity into the humeral head defect. 5- humeral rotational osteotomy.

PREFERRED RESPONSE: 3 RECOMMENDED READINGS: Hawkins RJ, Neer CS II, Pianta RM, et al: Locked posterior dislocation of the shoulder. J Bone Joint Surg Am 1987;69:9-18. Neer CS, Rockwood CA Jr: Fractures and dislocations of the shoulder, in Rockwood CA Jr, Green DP (eds): Fractures in Adults, ed 2. Philadelphia, PA, JB Lippincott, 1984, pp 675-985. The case and figure depict an example of a locked posterior dislocation of the shoulder. Hawkins and Neer state that the majority had been missed by the initial treating physician with only 25 of 41 being diagnosed within 6 months with the average interval of diagnosis being one year. The most common mechanism of injury in these patients was a MVA, followed by seizure, electroshock treatment and alcohol related injury. The most common misdiagnosis was frozen shoulder. Once the diagnosis is made, different treatment options exist. If the patient is inactive and a poor risk for surgery, no treatment should be administered. If the dislocation is less than six weeks old and the defect involves less than 20 percent of the articular surface, closed reduction should be attempted. If it is successful, immobilization is necessary for six weeks in abduction orthosis. If closed reduction is unsuccessful, an open reduction with transfer of the lesser tuberosity should be performed. If the dislocation is 6 weeks to 6 months old and the defect involves 20 to 45 percent of the articular surface, transfer of the lesser tuberosity should be done followed by immobilization in abduction orthosis. If the glenoid is normal and the dislocation is more than 6 months old or the defect involves more than 45 percent of the articular surface, a hemiarthroplasty should be done. If the glenoid has been destroyed and if the dislocation is more than 6 months old or the defect involves more than 45 percent of articular surface than a total shoulder replacement should be done. The figure shows a normal appearing glenoid with > 45 percent impression defect of the articular surface of the humeral head therefore 3 is the correct answer.

16. A 22-year-old woman has wrist pain after falling on her outstretched hand 2 weeks ago. Examination reveals tenderness in the anatomic snuffbox. A radiograph is shown in Figure 5. Treatment should consist of
123open reduction and internal fixation using the Russe bone graft technique via a volar approach. open reduction and internal fixation with a compression screw via a volar approach. open reduction and internal fixation with vascularized bone graft via a volar approach. open reduction and internal fixation with vascularized bone graft via a dorsal approach. a cast brace for 6 weeks.

45-

PREFERRED RESPONSE: 2 RECOMMENDED READINGS: Trumble TE (ed): Hand Surgery Update 3: Hand, Elbow, & Shoulder. Rosemont, IL, American Society for Surgery of the Hand, 2003, pp 161-173. Trumble TE, Gilbert M, Murray LW, et al: Displaced scaphoid fractures treated with open reduction and internal fixation with a cannulated screw. J Bone Joint Surg Am 2000;82:633-641.
The above case and figure reveal a 2 week old displaced scaphoid fracture through the waist. Displaced (>1mm) scaphoid fractures require internal fixaiton. Volar fixation is prefferred for waist and distal third fractures, while the dorsal approach is used for proximal pole fractures. Bone graft is not neccesary in this case and should be used liberally for comminuted fractures. In a retrospective review of 35 acute displaced scaphoid fractures treated with internal fixation with a cannulated screw, Trumble et al found that the rate of union was 100 percent regardless of the type of cannulated screw used. A volar approach was used in all patients. The cannulated screw fixation maintained the corrected fracture alignment and promoted healing and return of function which proved cannulated screws to be a safe and effective method of treatment. Of note, the time to union increased with increasing age of the patient, increasing initial displacement of the fracture and the time to union was longer for the patients who smoked.

17. A 14-year-old girl with a 58-degree thoracolumbar idiopathic curve underwent surgery using an anterior single-rod fusion technique. The structural mesh cages, shown in Figure 6, have been associated with 1- more frequent revision surgery. 2- improved maintenance of lumbar lordosis. 3- less coronal curve correction. 4- higher rates of instrumentation failure. 5- higher rates of pseudarthrosis.

PREFERRED RESPONSE: 2 RECOMMENDED READINGS: Sweet FA, Lenke LG, Bridwell KH, et al: Maintaining lumbar lordosis with anterior single solid-rod instrumentation in thoracolumbar and lumbar adolescent idiopathic scoliosis. Spine 1999;24:1655-1662. Ouellet JA, Johnston CE II: Effect of grafting technique on the maintenance of coronal and sagittal correction in anterior treatment of scoliosis. Spine 2002;27:2129-2135. Maintaining instrumented segmental lumbar lordosis after anterior fusion and instrumentation for thoracolumbar and lumbar curves has been difficult. Sweet et al performed a prospective radiographic evaluation of 20 consecutive patients with primary lumbar or thoracolumbar adolescent idiopathic scoliosis who were treated with anterior convex compressive single solid-rod spinal instrumentation and structural titanium mesh (Harms) cages packed with morselized rib autograft with the goal of evaluating whether or not lumbar lordosis would be preserved. The results showed coronal plane correction with preservation of thoracolumbar and lumbar lordosis at 2 years with no instrumentation failures, pseudarthroses, or reoperations. This is in contrast to prior techniques of correction which involved single-rod anterior instrumentation with rib strut or morselized interbody grafting which showed poor results in terms of maintenance of lordosis.

18. A 72-year-old woman with severe rheumatoid arthritis has had three hip dislocations in 1 week. History reveals that she underwent total hip arthroplasty 15 years ago. A radiograph obtained after attempted closed reduction is shown in Figure 7. Management should now consist of 1- open reduction followed by bracing with a hip orthosis for 6 weeks. 2- revision of the acetabular metal shell and liner. 3- revision of both the femoral and acetabular components. 4- revision to a constrained liner, with metal shell retention. 5- revision to a large diameter femoral head with metal shell retention.

PREFERRED RESPONSE: 2 RECOMMENDED READING: Peters CL, Dunn HK: The cementless acetabular component, in Callaghan JJ, Rosenberg AR, Rubash HE (eds): The Adult Hip. New York, NY, Lippincott-Raven, 1998, pp 9931016. The above figure and case show a well fixed cemented femoral component with apparent vertical migration of the acetabular cup and large osteolytic lesions in the supra-acetabular bone and greater trochanter. The acetabular component appears to be a porous coated anatomic acetabular component (PCA, Howmedica) which is composed of chromium-cobalt alloy with a double layer of sintered chromium-cobalt beads and two peripheral pegs for rotational stability. Although initial results with the PCA component were encouraging, longer term follow-up revealed the prevalence of osteolysis in the periacetabular region between 3%-56%. This osteolysis has, in the vast majority of cases, required revision of the acetabular component. In one cumulative series of PCA acetabular components with greater than 7-year follow up totaling 1200 cups, the aseptic loosening rate and the rate of acetabular osteolysis were both 15%. This patient would clearly benefit from revision of the acetabular shell and metal liner.

Marc Fajardo

19. A 32-year-old man has had a 10-year history of intermittent but recurrent ankle sprains. Examination reveals a varus heel position that corrects with a Coleman block test, and there is no lateral ligament instability with anterior or lateral stress testing. What type of orthotic/prosthetic should be prescribed?
1- UCBL insert 2- Arizona brace 3- Rigid orthotic with a medial arch support and 5 degrees of medial heel posting 4- Semi-rigid orthotic with a recessed first metatarsal, a lateral forefoot wedge, a reduced medial arch, and an elevated heel 5- Carbon fiber insole with a Mortons extension

ANSWER: 4 This patient is manifesting symptoms of a long standing pes cavus deformity of the right foot. Ankle instability is a common feature of pes cavus. This is due to a combination of varus hindfoot alignment and weak dynamic stabilizers (peroneus brevis) that can lead to a functionally unstable ankle. Such instability and severe deformity in the long term can lead to degeneration of the ankle joint. The coleman block test shows whether or not the subtalar joint/hindfoot is mobile. When the patient stands with the lateral forefoot and heel on a block, this allows the 1st metatarsal to drop, and the tripod effect of the plantarflexed 1st ray to be eliminated. A flexible hindfoot will assume normal alignment, while a rigid deformity will remain in varus. This patient displays a flexible hindfoot, and with no lateral ligament instability is indicated for conservative management such as bracing. Now to the answers: 1. a UCBL is indicated for the management of isolated foot deformities that are passively correctable with patients who have mild spasticity (cp). 2. an Arizona brace is used for pes planus (severe pronation) and both ant/post tibialis tendinitis. 3. a Rigid orthotic with a medial arch support and 5 degrees of medial heel posting would exacerbate the symptoms of pes cavus. 5. a carbon insole with mortons extension is used for 1st mtp pathology such as turf toe and hallux rigidus. Answer 4 is the best answer because it corrects the hindfoot alignment and removes the tripod effect.

PREFERRED RESPONSE: 4 RECOMMENDED READINGS: Manoli A II, Graham B: The subtle cavus foot, the underpronator. Foot Ankle Int 2005;26:256-263. Bordelon RL: Orthotics, shoes, and braces. Orthop Clin North Am 1989;20:751-757.

20. Hereditary multiple exostosis is an inherited disorder that frequently is associated with mutations of EXT1 and EXT2 genes. These mutations affect what part of the growth plate most directly? 1- Proliferating chondrocytes 2- Prehypertrophic chondrocytes 3- Hypertrophic chondrocytes 4- Periosteum 5- Synovium

ANSWER: HME is an AD disorder characterized by pathology of the EXT1 and EXT2 genes. Recent evidence suggests that they regulate chondrocyte maturation and differentiation necessary for normal endochondral ossification within the growth plate. This is done by a signaling cascade involving a negative feedback loop of normal prehypertrophic chondrocytes. Patients with HME have multiple cartilage-capped exostoses that are histologically benign lesions. Nervertheless, the correct answer is number 2

PREFERRED RESPONSE: 2 RECOMMENDED READING: Stieber JR, Dormans JP: Manifestations of hereditary multiple exostoses. J Am Acad Orthop Surg 2005;13:110-120.

21. In the child with acute onset diskitis, what is the earliest radiographic finding? 1- Narrowing of the intervertebral disk space 2- Scalloping of the superior vertebral body 3- Scalloping of the inferior vertebral body 4- Vertebra magna 5- Loss of normal lumbar lordosis

ANSWER: The normal progression of radiological findings in acute diskitis is as follows: initial radiographs will not demonstrate osseous changes, but a loss of normal lordosis may be seen. When symptoms have been present for 1 week or more, plain radiographs may also reveal isolated intervertebral disk space narrowing. By 3-4 weeks xrays may show saw-tooth erosion of adjacent vertebral end plates. Scalloping of the superior and inverior vertebral body may be seen with long-standing infections. And finally, vertebra magna with resultant canal narrowing, permanent loss of disk height, or block vertebra may be noted in patients with resolved infections. Answer 5 is the right answer

PREFERRED RESPONSE: 5 RECOMMENDED READINGS: Early SD, Kay RM, Tolo VT: Childhood diskitis. J Am Acad Orthop Surg 2003;11:413-420. Szalay EA, Green NE, Heller RM, et al: Magnetic resonance imaging in the diagnosis of childhood discitis. J Pediatr Orthop 1987;7:164-167.

22. Figures 8a and 8b show the injury a 17-year-old boy sustained in a recreational vehicle accident 3 weeks ago. Examination reveals mild asymmetry and regional discomfort referable to the right sternoclavicular joint. The remainder of the physical examination findings are unremarkable. Management should consist of 1- closed reduction. 2- open reduction and pin fixation. 3- sternoclavicular ligamentous reconstruction. 4- nonsurgical treatment with resumption of unrestricted activity in 3 months. 5- nonsurgical treatment with cessation of contact sports indefinitely.

ANSWER: This is a 17 y/o, male 3 weeks out from a possible posterior dislocation of the right sternoclavicular joint, however remember the clavicle is the first long bone to ossify and the medial end of the clavicle the last to ossify. The medial clavicular epiphysis does not ossify until the 18th to 20th year of age, and it fuses with the shaft of the clavicle around the 23rd to 25th year of age. So this injury is probably a medial physis injury of the clavicle. Since the patient is 3 weeks out from injury and displays no other symptoms (such as complications due to pressure of the hilar structures) conservative management is indicated. The majority of these injuries will heal with time and the remodeling process will eliminate most of the bone deformity or displacement. Answer 4 is the correct answer..
PREFERRED RESPONSE: 4 RECOMMENDED READINGS: Iannotti JP, Williams GR: Sternoclavicular joint, in Ianotti JP, Williams GR (eds): Disorders of the Shoulder: Diagnosis and Management, ed 1. Philadelphia, PA, Lippincott Williams & Wilkins, 1999, pp 765-813. Wirth MA, Rockwood CA Jr: Disorders of the sternoclavicular joint: Pathophysiology, diagnosis, and management, in Rockwood CA Jr, Matsen FA II, Wirth M (eds): The Shoulder, ed 3. Philadelphia, PA, WB Saunders, 1999, pp 765-813.

23. An 80-year-old woman reports the sudden development of pain in the left distal thigh. She denies any history of trauma. Figures 9a through 9d show radiographs, a bone scan, and a biopsy specimen. What is the most likely diagnosis? 1- Enchondroma 2- Osteosarcoma 3- Dedifferentiated chondrosarcoma 4- Metastatic breast carcinoma 5- Pagets disease

ANSWER: Characteristics of the radiographs display a permiative lesion over the diaphysis of the distal tibia. There is a cortical reaction with an unmineralized soft tissue mass extension. There are likewise calcifications within the bone matrix. Thus, there is a biphasic quality to the radiographs where there are areas of endosteal scalloping and coritcal thickening that are contrasted with areas of cortical destruction and soft tissue. The histology slides also display cartilage and an abrupt interface to a fibrosarcoma, there is no merging of the tumor types. These characteristics are common in dedifferentiated chondrosarcoma, answer 3. Dedifferentiated chondrosarcoma is the most malignant form of chondrosarcoma. This tumor is a mix of low grade chondrosarcoma and high grade spindle cell sarcoma where the spindle cells are no longer identifiable as having a cartilage origin. Treatment is wide surgical excision however dedifferentiated chondrosaroma has a local recurrence in up to 50% of cases. It has a poor prognonsis, patients rarely survive more than 2 years.

PREFERRED RESPONSE: 3 RECOMMENDED READINGS: Lee FY, Mankin HJ, Fondren G, et al: Chondrosarcoma of bone: An assessment of outcome. J Bone Joint Surg Am 1999;81:326-338. Mitchell AD, Ayoub K, Mangham DC, et al: Experience in the treatment of dedifferentiated chondrosarcoma. J Bone Joint Surg Br 2000;82:55-61.

24. A 10-year-old girl has abdominal pain following a motor vehicle accident where she was restrained in the back seat by a lap belt. Bruising is noted across the lower abdomen. Radiographs reveal a flexion-distraction injury of T12-L1. What is the most likely associated nonskeletal injury? 1- Iliac vein tear 2- Uterine rupture 3- Colonic perforation 4- Cardiac contusion 5- Aortic dissection

ANSWER: School-aged children (4-10) are a special group with respect to occupant restraint systems in motor vehicles. Compared with infants, these children have a lower center of gravity and their body habitus differs from that of adults in 2 ways: the intra-abdominal organs are less protected by the boney thorax and pelvis, and the iliac crests are not adequately developed to serve as anchor points for the belt, and allowing the belt to ride up over the abdomen. Thus, a unique association of injuries have emerged with the use of lap belts. These include intestinal injuries, mesenteric disruption, and lumbar spine dislocation and fractures (so called seat-belt syndrome). The entire class of seat-belt related flexion distraction injuries to the spine are referred to as Chance fractures. These fractures are most likely caused by hyperflexion around the lap belt, with the belt acting as a fulcrum, subjecting the vertebra to tension and distraction. This mechanism of hyperflexion and distraction would explain not only chance fractures but also the association of intra-abdominal injuries, as viscera are crushed between the lap belt and the spine. The literature has shown a relative risk of 50% between chance fractures and bowel injury. In the question above, the patient exhibits ecchymosis over her lower abdomen which most likely suggests the pattern of the lap belt. Xrays also reveal a Chance fracture along T12 L1. Therefore answer 3 is the best answer for reasons as stated above.

PREFERRED RESPONSE: 3 RECOMMENDED READINGS: Reid AB, Letts RM, Black GB: Pediatric chance fractures: Association with intra-abdominal injuries and seatbelt use. J Trauma 1990;30:384-391. Holland AJ, Cass DT, Glasson MJ, et al: Small bowel injuries in children. J Pediatr Child Health 2000;36:265-269.

Kirill Illalov

12345-

Women older than age 35 Women older than age 65 Women with an income above $15,000 Women with high self-esteem Pregnant women

PREFERRED RESPONSE: 5
Explanation: Certain groups stand out at being a high risk: (1) Women aged 19-29 years old in families with annual incomes below $10,000; (2) Pregnant women; (3) Inner-city AfricanAmerican women

RECOMMENDED READING: Zillmer DA: Domestic violence: The role of the orthopaedic surgeon in identification and treatment. J Am Acad Orthop Surg 2000;8:91-96.

26. An 8-year-old girl sustained a Salter-Harris type II fracture of the right distal femoral epiphysis 18 months ago that was treated with closed reduction and percutaneous screw fixation through the metaphyseal fragment. Over the past year, her mother has noted a progressive knock-knee deformity of the right leg. Following removal of the implant, radiographs and a T1-weighted MRI scan are shown in Figures 10a through 10d. Treatment should now consist of 1- distal femoral osteotomy. 2- distal femoral osteotomy and epiphyseodesis. 3- contralateral distal femoral epiphyseodesis. 4- resection of the physeal bridge and interposition of fat. 5- observation.

PREFERRED RESPONSE: 4
Explanation: Patient developed a physeal bony bridge. According to the above article, growth arrest secondary to physeal bridge formation is an uncommon, but well-recognized complication of physeal fractures and other injuries. Regardless of the underlying physiology, physeal bridges may cause angular and/or longitudinal growth disturbances, with progression dependent on the remaining physeal growth potential. Physeal bridge resection and insertion of interposition material releases the tethering effect of the bridge. Physeal bridge resection has become an accepted treatment option for patients with existing or developing deformity and for those with at least 2 years or 2 cm of growth remaining.

RECOMMENDED READINGS: Khoshhal KI, Kiefer GN: Physeal bridge resection. J Am Acad Orthop Surg 2005;13:47-58. Sponseller PD, Stanitski CL: Fractures and dislocations about the knee, in Beaty JH, Kasser JR (eds): Rockwood and Wilkins Fractures in Children, ed 5. Philadelphia, PA, Lippincott Williams & Wilkins, 2001, pp 982-1076.

12345-

Atrophic Oligotrophic Hypertrophic Infected Synovial pseudarthrosis

PREFERRED RESPONSE: 3
Explanation: Atrophic nonunion show little callus formation around a fibrous tissue-filled gap. These commonly occur after open fractures or previous operative procedures where the soft tissue envelope has been separated from the bone, removed, or damaged. If alignement is good and there is no gap, then stabilization under compression, with shingling and bone grafting, will rekindle the fracture healing process. If there is malalignement or a gap, resection of the scar tissue is essential, and in addition to shingling and stabilization, bone grafts may be necessary. Oligotrophic nonunions possess an adequate blood supply, but display little or no callus formation. The cause is typically an inadequate reduction that results in little or no contact at the bony surfaces. Treatment methods that result in union of oligotrophic nonunions include reduction of the bony fragments to improve bone contact, bone grafting to promote bridging of the ununited bony gaps, or a . combination of reduction of the bony fragments and bone grafting.
.

PREFERRED RESPONSE: 3
Hypertrophic nonunions show decreased vascularity and callus formation, with a horse-shoe or elephant-foot configuration on the radiographs. Hypertrophic elephant foot callus is a sign of motion at a vascular nonunion where the interposed tissue is fibrocartilage. Unless there is significant malalignment, there is no need for resection of the nonunion tissue or the ends, and a bone graft is usually unnecessary. Union will occur rapidly when the motion is stopped by stable external or internal fixation devices. Decortication of the nonunion cortices at the time of exposure speeds consolidation. Infected nonunion results from the sequestrum formed by cortical bone death, creation of gaps by osteolytic infected granulation tissue, and motion from loosening of implants. It may require more than one procedure to eliminate the infection, i.e. sequestrectomy, debridment, skin and bone grafts, long periods of external fixation, and non-weight bearing immobilization. . Synovial pseudoarthrosis is a nonunion in which a fluid-filled cavity with a synovial. membrane is present at the fracture site. The treatment is by excising the like pseudoarthrosis tissue, opening the medullary canals and surgical stabilization/bone grafting.

PREFERRED RESPONSE: 3

RECOMMENDED READINGS: Rodriguez-Merchan EC, Forriol F: Nonunion: General principles and experimental data. Clin Orthop 2004;419:4-12. Browner BD, Jupiter JB, Trafton PJ, et al (eds): Skeletal Trauma. Philadelphia, PA, WB Saunders, 1992, pp 247-252.

and the incisura fibularis. 2- distance from the incisura fibularis to the lateral tibial border. 3- distance from the medial malleolus to the talus. 4- overlap of the anterior portion of the lateral tibia on the fibula. 5- overlap of the posterior portion of the lateral tibia on the fibula.

PREFERRED RESPONSE: 1
Explanation: The tibiofibular space (or interosseous clear space) is cartilaginous space between the lateral border of posterior tibia (incisura fibularis) and medial border of the fibula, measured 1 cm above the joint line. Normal tibiofibular relationship is consistent with (1) a tibiofibular clear space of less than 6 mm, (2) tibiofibular overlap on the anterior-posterior view of greater than 6 mm or 42% of fibular width; (3) tibiofibular overlap on the mortise view of greater than 1 mm.

RECOMMENDED READINGS: Harper MC, Keller TS: A radiographic evaluation of the tibiofibular syndesmosis. Foot Ankle Int 1989;10:156-160. Ostrum RF, De Meo P. Subramanian R: A critical analysis of the anteriorposterior radiographic anatomy of the ankle syndesmosis. Foot Ankle Int 1995;16:128-131.

sensitivity to touch, and a mottled skin color appearance. Repeat radiographs are normal. What is the most likely diagnosis? 12345Compartment syndrome of the forearm Carpal tunnel syndrome Complex regional pain syndrome, type 1 Allergic reaction to the splint material Rupture of the extensor pollicis longus tendon

PREFERRED RESPONSE: 3
Explanation: Complex regional pain syndrome, formerly known as RSD and causalgia, is a condition with unknown underlying pathophysiology. It is more common in individuals 40 to 60 years of age, especially women. Complex regional pain syndrome, type I, is triggered by tissue injury. It is different from CRPS, type II, in that there is no underlying nerve injury. Both types share the same diagnostic criteria: (1) spontaneous pain is not limited to the territory of a single peripheral nerve, and is disproportionate to the inciting event, (2) there is a history of edema or abnormal sweating in the region of pain since the inciting event, (3) no other conditions can account for the degree of pain and dysfunction.

RECOMMENDED READING: Trumble TE (ed): Hand Surgery Update 3: Hand, Elbow, & Shoulder. Rosemont, IL, American Society for Surgery of the Hand, 2003, pp 335-351.

and a radiograph of the pelvis are shown in Figures 11a and 11b. Management should consist of 1- 3 weeks of traction, followed by closed reduction of the hips, possibly with adductor longus releases. 2- observation of the hips as long as function is not impaired. 3- a trial of abduction splinting with a Pavlik harness. 4- bilateral open hip reduction with femoral shortening and an acetabular redirection osteotomy. 5- bilateral capsular interposition arthroplasty (Colonnas operation).

PREFERRED RESPONSE: 2
Explanation: Arthrogryposis Multiplex Congenita is used to describe congenital nonprogressive limitation of movement including, but not limited to, amyoplasia, myelomeningocele, congenital muscular dystrophy, Larsens syndrome, Mobius syndrome, and skeletal dysplasias. Amyoplasia is the most common form (40%) of AMC and is likely the case here. The others can be excluded based on the normal skeletal enzymes (muscular dystrophies), obvious phenotypic abnormalities (myelomeningocele, skeletal dysplasias), and the absence of abnormal facial features and normal intelligence (Larsens, Mobius). The management of bilateral hip dislocation in this patient population is controversial. Functional ambulation without pain is possible with bilateral dislocations. Yah et al. noted in their 20 year follow up that in their group of 19 patients (with 13 dislocations, 9 subluxations and 16 contractures) that closed treatment always failed in the treatment of dislocated hips. The also noted that the function of the openly reduced hips was comparable with others. Advocates of surgical treatment believe that reduction improves the quality and efficiency of gait. .

PREFERRED RESPONSE: 2

RECOMMENDED READINGS: Yau PW, Chow W, Li YH, et al: Twenty-year follow-up of hip problems in arthrogryposis multiplex congenita. J Pediatr Orthop 2002;22:359-363. Sponseller PD (ed): Orthopaedic Knowledge Update: Pediatrics 2. Rosemont, IL, American Academy of Orthopaedic Surgeons, 2002, pp 277-283. Alman BA, Goldberg MJ: Syndromes of orthopaedic importance, in Morrissy RT, Weinstein SL (eds): Lovell and Winters Pediatric Orthopaedics, ed 5. Philadelphia, PA, Lippincott Williams & Wilkins, 2001, pp 287-338.

Dan Laino

31. A reverse total shoulder arthroplasty is most appropriate in which of the following situations?
1- Young laborer with degenerative arthritis superimposed on a massive, unrepairable rotator cuff tear 2- Elderly individual with painful shoulder motion limited to 140 degrees of forward flexion 3- Elderly individual with a painful, arthritic shoulder and an irreparable rotator cuff tear, with active forward flexion of 30 degrees 4- Sedentary 45-year-old individual with a malunited four-part proximal humerus fracture 5Failed hemiarthroplasty secondary to glenoid wear

PREFERRED RESPONSE: 3
Unconstrained shoulder arthroplasty has been a standard surgical option in the treatment of patients with massive unbalanced rotator cuff tears and arthrosis. Total shoulder replacement, however, has been abandoned because the excessive shearing forces produce what is known as the rocking-horse phenomenon, leading to superior eccentric loading on the glenoid and glenoid loosening. Hemiarthroplasty has consequently become the recommended treatment option for the arthritic shoulder in patients with severe cuff deficiency, but the results in these patients have been less predictable, in terms of both mobility and pain relief . This is why Neer et al proposed the term limited-goal rehabilitation for these hemiarthroplasties: they allow some improvement in shoulder pain and rotation but almost no improve ment in active elevation or abduction in patients with preoperative pseudo-paralysis. Elevation occasionally exceeds the horizontal level but pain relief is often inconsistent, and early good results can deteriorate as a result of glenoid and/or acromial erosion. Grammont developed a reverse prosthesis (Delta III) that has been used in Europe and has recently been FDA approved in the USA. This is a reverse prosthesis in that the glenoid component is hemispheric and the humeral component is cup -shaped. This design is meant to decrease glenohumeral joint forces (and subsequently decrease glenoid component loosening) as well as optimizing the elevating action of the deltoid to compensate for a deficient rotator cuff. In summary, the main biomechanical advantages of the reverse prosthesis according to Grammonts concept are as follows: (1) the large ball offers a greater potential arc of motion and more stability than a s mall ball, (2) the small lateral offset (absence of neck) places the center of rotation directly in contact with the glenoid surface and reduces the torque at the point of fixation of the glenoid component, (3) medializing the center of rotation recruits more of the deltoid fibers for elevation or abduction, and (4) lowering the humerus increases tension on the deltoid. These biomechanical properties lead to better functioning of the deltoid, compensating for the lack of a functional rotator cuff. #1 is wrong because it is important to note that results for this prosthesis have only been reported in older patients, with limited activity levels. One can only assume at present that the higher activity levels of younger patients will lead to higher failure rates (direct quote from J Shoulder Elbow Surgery)

#2, #4 and #5 are wrong as the reverse TSA is indicated for pts with irreparable RTC tears, which these patients dont have

RECOMMENDED READING: Goutallier D, Postel JM, Zilber S, et al: Shoulder surgery: From cuff repair to joint replacement. An update. J Bone Joint Surg Am 2003;70:422-432. De Buttet M, Bouchon Y, Capon D, et al: Abstract: Grammont shoulder arthroplasty for osteoarthritis with massive rotator cuff tears: Report of 71 cases. J Shoulder Elbow Surg 1997;6:197.

32. During normal knee flexion from 0 degrees to 100 degrees, which of the following kinematic motions occurs? 1- Both the lateral and medial femoral condyles move posteriorly an equal distance.
2- The lateral condyle moves posteriorly more than the medial condyle. 3- The medial condyle moves posteriorly more than the lateral condyle. 45The lateral condyle moves anteriorly. The medial condyle moves anteriorly.

Preferred Response: 2
This study from a biomechanical journal looked at deep knee bending under live fluoroscopy in 10 normal knees and 5 ACL deficient knees. While performing a deep knee bend, all 10 normal knees experienced posterior femoral translation of their lateral condyle and minimal change in the position of the medial condyle. The average amount of posterior femoral translation of the lateral condyle from 0 to 120 flexion was 21.07mm (standard deviation 9.30), whereas the average medial condyle translation was 1.94mm (standard deviation 1.86), in the posterior direction. All 10 knees experienced a normal axial rotation pattern during a deep knee bend (tibia internally rotating with increased knee flexion, i.e., screw home mechanism), because the posterior translation of femorotibial contact laterally was greater than that observed medially. The average amount of axial rotation for the 10 subjects from 0 to 120 flexion was 23.67_ (standard deviation, 9.56) in the normal direction. While performing a deep knee bend, all five subjects with an ACLD knee experienced posterior femoral translation of their lateral condyle with increased translation of the medial condyle when compared to normal knees. The average amount of posterior femoral translation of the lateral condyle from 0 to 120 of knee flexion was 17.00mm (standard deviation, 7.31), while the medial condyle translation was 4.65mm (standard deviation, 3.99), in the posterior direction. All subjects experienced an axial rotation pattern similar to that of the normal knee subjects only during the first 30. After 30 the axial rotation pattern of the ACLD knees exhibited different behavior than normal knees (changing slope from positive to negative in Fig. 13). The average amount of axial rotation for the five subjects from 0 to 120 of flexion was 9.87_ (standard deviation=3.23).

The conclusion of this article is that the abnormal knee kinematics in ACL deficient knees may be detrimental

#1 is wrong b/c the condyles both move posteriorly, but not an equal distance #3 is wrong b/c the lateral condyle moves posteriorly more than the medial condyle #4 and #5 are wrong b/c neither condyle moves anteriorly

RECOMMENDED READING: Dennis DA, Mahfouz MR, Komistek RD, et al: In vivo determination of normal and anterior cruciate ligament-deficient knee kinematics. J Biomech 2005;38:241-253.

33. A greater trochanteric entry site for femoral intramedullary nailing, compared to a piriformis fossa starting point, is associated with

12-

more disruption of the gluteus medius tendon.


more disruption of the short external rotator tendons.

3- more disruption of the medial femoral circumflex artery. 4greater hoop stresses with an anterior starting point.

5- greater colinearity with a long axis of the femoral shaft.

PREFERRED RESPONSE: 1
This question is based mostly on a cadaveric study by Dora et al in which cadaver femurs were nailed and the soft tissues were then dissected afterwards to assess the soft tissue damage. Moein et al (Injury 2005) did a similar cadaveric study. According to these studies: #1 is correct as there is higher rate of injury to the gluteus medius tendon with a greater troch starting point, but also of note the gluteus medius muscle was damaged in all cases of a piriformis starting point. (from Moein et al, Injury 2005) #2 is incorrect as Dora found a greater incidence of short external rotator damage with a piriformis starting point. #3 is also incorrect as Dora found a greater incidence of damage to the MFCA with a piriformis starting point

#4 is incorrectthe Johnson article comments that an anterior starting point does indeed create higher hoop stress at the fracture site, but this article was in regard to the piriformis starting point.
#5 is incorrect because there is greater colinearity with the long axis of the femoral shaft with a piriformis starting point. RECOMMENDED READINGS: Dora C, Leunig M, Beck M, et al: Entry point soft tissue damage in antegrade femoral nailing: A cadaver study. J Orthop Trauma 2001;15:488-493. Johnson KD, Tencer AF, Sherman MC: Biomechanical factors affecting fracture stability and femoral bursting in closed intramedullary nailing of femoral shaft fractures, with illustrative case presentations. J Orthop Trauma 1987;1:1-11. Winquist RA, Hansen ST Jr, Clawson DK: Closed intramedullary nailing of femoral fractures: A report of five hundred and twenty cases. J Bone Joint Surg Am 1984;66:529-539.

34. What region of the growth plate seen in Figure 12 is commonly damaged in a Salter-Harris type I physeal injury?

12345-

A B C D E

PREFERRED RESPONSE: 4 The physis consists of the reserve zone, proliferative zone, and hypertrophic zone. Salter Harris I injuries occur through the hypertrophic zone, which corresponds to D on the image. Looking at the diagrams from the reference, it appears that on the image A=the bony epiphysis, B=reserve zone, C=proliferative zone, D=hypertrophic zone, E=metaphysis

RECOMMENDED READING: Buckwalter JA, Einhorn TA, Simon SR (eds): Orthopaedic Basic Science: Biology and Biomechanics of the Musculoskeletal System, ed 2. Rosemont, IL, American Academy of Orthopaedic Surgeons, 2000, pp 77-109.

35. 7-year-old girl has late-onset idiopathic scoliosis. The natural history of this condition, compared with unaffected control subjects, is associated with which of the following?

12-

Higher mortality rates Increased shortness of breath

345-

Increased chronic and acute back pain


Increased disability More frequent episodes of depression

PREFERRED RESPONSE: 3
This question is based on a study by Weinstein et al. that is based on a prospective natural history study with 50 year follow up of late onset idiopathic scoli patients at the Universityof Iowa. #1 is incorrect because there was no significant difference in mortality rates between the study population and that expected in the general population #2 is incorrect b/c the study population did not complain of shortness of breath significantly more than the control population. They did find that a Cobb angle greater than 50 degrees at skeletal maturity as well as large thoracic curves >80 degrees were associated with a higher rate of subjective SOB. They comment that only patients with thoracic curves > 100 degrees are at risk from death from cor pulmonale. #3 is correct b/c the scoliosis patients did have a higher rate of acute and chronic back pain compared with the control group, however this does not appear to cause excessive disability. #4 is incorrect b/c there is no significant difference between the scoli patients and the control group regarding disability at work or capacity to perform ADLs. #5 is incorrect b/c they found no significant difference in depression indices between scoli patients and control groups.

Here is the study conclusion: By closely studying this group of patients for more than 50 years, we have learned that patients with untreated LIS can function well as young adults, become employed, get married, have children, and grow to become active older adults. Unfortunately, patients with untreated LIS can develop significant deformity, and the cosmetic aspect of this condition cannot be disregarded. The physical outcomes demonstrated in this cohort born many decades ago can be used to predict the likely experience of a similar set of untreated patients born later in the century, although what is less sure is if a contemporary cohort (and their peers) would be as accepting of deformity as these patients have been.

It is essential that community physicians and the public recognize the difference in clinical course between early-onset scoliosis and LIS, and recognize that the latter is likely to cause little physical impairment other than back pain and cosmetic concerns. Curves less than 30 at skeletal maturity rarely get worse. However, back pain may arise in any patient regardless of curve size or location. School screening programs, if conducted at all, should aim for low false-positive rates, because the high error rates have led to undue concern for adolescents and their parents, as well as contributing to expensive, unnecessary radiographs and specialty consultations. Community physicians should be concerned with noticeable body asymmetries as noted on the forward bend test in the thoracic, thoracolumbar, and lumbar area, shoulder height discrepancies, and trunk shift, especially in skeletally immature patients. The recommendation of bracing and surgery must be made on an individual basis with the patient and family well-informed of the natural history of the disease

RECOMMENDED READINGS: Weinstein SL, Dolan LA, Spratt KF, et al: Health and function of patients with untreated idiopathic scoliosis: A 50-year natural history study. JAMA 2003;289:559-567. Weinstein SL, Zavala DC, Ponseti IV: Idiopathic scoliosis: Long-term follow-up and prognosis in untreated patients. J Bone Joint Surg Am 1981;63:702-712.

36. Figure 13 shows the clinical photograph of a patient who underwent a below-knee amputation. Why are the dog ears at the edge of the incision typically present?

123-

Removal risks the posterior flap blood supply. They improve the initial cosmetics of the limb. They allow for early weight bearing.

45-

They are necessary for prosthetic fitting.


The surgeon has over-rotated the posterior flap.

PREFERRED RESPONSE: 1
The references are not really that helpful for this question, and I had to go to a vascular surgery textbook for this one. There are several different techniques for performing a BKA, but the posterior flap technique is a good choice for patients with vascular disease because the posterior tissue of the leg is usually better perfused than the anterior tissue. With this technique, dog ears commonly result. These should not be routinely trimmed because this endangers the blood supply to the posterior flap and surrounding skin (#1, correct answer). Small dog ears typically remodel. Dog ears do not improve the cosmetics of the limb (#2) and do not affect weight bearing (#3). They are not necessary for prosthetic fitting (#4). Small dog ears are common with posterior flaps and are not the result of over-rotation of the flap (#5).
RECOMMENDED READINGS: Falstie-Jensen N, Christensen KS, BrochnerMortensen J: Long posterior flap versus equal sagittal flaps in below-knee amputation for ischaemia. J Bone Joint Surg Br 1989;71:102-104.
Manoli A II: A long posterior flap in below-knee amputations for peripheral vascular disease: Rationale and technique. Foot Ankle Int 1998;19:110-112.

Meredith Lazar

37. A 46-year-old woman has had plantar heel pain for the past 5 months. The pain is most severe when she arises out of bed in the morning and when she stands after being seated for a period of time. Initial treatment should consist of
1. 2. 3. 4. 5. surgical lengthening of the Achilles tendon. surgical release of the plantar fascia. a custom orthosis. a stretching program and a cushioned insert. a corticosteroid injection.

PREFERRED RESPONSE: 4
This question asks you to diagnos and then recall first line treatment for proximal plantar fascitis (heel pain syndrome). Plantar fasciitis is a repetitive overload injury causing microtrauma to the attachment of the plantar fascia at the inferior aspect of the calcaneus. It causes traction periostitis and microtears resulting in pain and inflammation. Harty et als study provided patients w/ locked Bledsoe braces at various degrees of knee flexion to demonstrate a progressive increase in forefoot loading. They were then able to implicate that an increase in hamstring tightness may induce prolonged forefoot loading and through the windlass mechanism be a factor that increases repetitive injury to the plantar fascia. The symptoms of plantar fascitis are: early morning pain, inability to bear weight, mild erythema, and maximal point tenderness over the medial calcaneal tuberosity at the insertion of the plantar fascia. Most patients (approx 85%) find relief with nonoperative treatment which may include shoe modification, prefabricated inserts, custom orthoses, stretching exercises, physical therapy, nonsteroidal antiinflammatory agents, cortisone injections, night splints, and casting. The Pfeffer article is a prospective randomized study comparing treatment results w/ differing shoe inserts at 8 weeks. The study has all groups doing appropriate stretching exercises and compares 3 types of ready-made cushions, 1 custom orthotic, and no orthotic (control is stretching alone). They all were effective treatment modalities, however the custom orthotic was worse for pts that stood > 8 hrs/day. This trial proved that the use of relatively inexpensive prefabricated inserts, along with Achilles tendon and plantar fascia stretching (CORRECT ANSWER 4) is more effective than a custom polypropylene orthosis (WRONG ANSWER 3) for the initial treatment of proximal plantar fascitis. A steroid injection may be helpful only if an area of focal tenderness is present (WRONG ANSWER 5). Surgical intervention is indicated after several months of failed non-operative treatment. The procedure of choice would be plantar fascia release and heel spur removal (Du Vries procedure) (WRONG ANSWER 2). Achilles tendon lengthening is done for cavus deformity and not heel pain (WRONG ANSWER 1).

RECOMMENDED READINGS:
Pfeffer G, Bacchetti P, Deland J, et al: Comparison of custom and prefabricated orthoses in the initial treatment of proximal plantar fasciitis. Foot Ankle Int 1999;20:214-221. Richardson EG: Heel pain, in Coughlin MJ, Mann RA (eds): Surgery of the Foot and Ankle, ed 7. St Louis, MO, Mosby, 1999, pp 862Harty J, Soffe K, OToole G, et al: The role of hamstring tightness in plantar fasciitis. Foot Ankle Int 2005;26:1089-1092.

38. Avulsion fractures of the anterior inferior iliac spine in adolescent athletes occur from forceful contraction of which of the following muscles?
1. 2. 3. 4. 5. Sartorius Tensor fascia lata Iliopsoas Rectus femoris Internal and external obliques

PREFERRED RESPONSE: 4
According to OKU, an avulsion fracture is an injury through an apophyseal cartilage plate caused by a large distraction force. This is a Salter Harris I fracture and can be through AIIS, ASIS, ischial tuberosity, lesser trochanter, or iliac crest. They occur almost exclusively in the athletic adolescent between 11-17 yrs old. Most common type of pediatric pelvic fractures and occur during running or sprinting. Diagnosis of AIIS avulsion often requires oblique xray, or may use CT or MRI. AIIS fractures are caused by avulsion of the straight head of the rectus femoris (CORRECT ANSWER 4). Treatment is conservative w/ protected weight bearing and rest. ASIS fractures are caused by sartorius and tensor fascia lata (WRONG ANSWERS 1 and 2) and the activity (like swinging baseball) is when hip is extended and knee is flexed. Fractures of ischial tuberosity are most common overall and from semitendinosus, semimembrinosus and adductors (hurdlers are at risk). Iliac crest fracture result in tear of internal and external obliques (WRONG ANSWER 5) and lesser tuberosity fractures are iliopsoas (WRONG ANSWER 3).

RECOMMENDED READINGS: Garrick JG (ed): Orthopaedic Knowledge Update: Sports Medicine 3. Rosemont, IL, American Academy of Orthopaedic Surgeons, 2004, pp 139-153. DeLee JC, Drez D Jr, Miller MD: Hip and pelvic injuries in the young athlete, in DeLee JC, Drez D Jr, Miller MD (eds): Orthopaedic Sports Medicine, ed 2. Philadelphia, PA, WB Saunders, 2003, pp 1463-1479.

39. In patients with ipsilateral femoral neck and shaft fractures, what percent of femoral neck fractures are significantly displaced?
1. 2. 3. 4. 5. 5% 20% 40% 70% 90%

PREFERRED RESPONSE: 3
Bennet et al state that there are several unique features of ipsilateral femoral neck and shaft fractures which are thought to be due to a mechanism of injury in which the femur is longitudinally loaded at the flexed knee while positioned in neutral adduction-abduction. The resultant femoral neck fractures are usually vertical with minimal displacement. The shaft fracture is most often in the middle one third. Examination of the femoral neck fractures in their series showed most to be low-energy injuries with minimal to no displacement. Only 15 of 37 (40%) (CORRECT ANSWER 3) were displaced enough to require an attempt at reduction. In contrast, the associated shaft fractures were often characteristic high-energy fractures. Of note in the article is that 39% had ipsilateral knee injury patella fractures were common. There was an 8% incidence of non-union of the neck fxs. Total incidence of AVN in this series was is 4%. Also note the incidence of delayed diagnosis of ipsilateral hip and femur fractures is around 20%. The article concludes with the prognosis with regard to nonunion and AVN for femoral neck fractures in young adults is better when the ipsilatera1 femoral shaft is also fractured. There is no evidence that antegrade rodding adjacent to an unstabilized femoral neck fracture alters the prognosis. Wolinsky et al report overall incidence for ipsilateral femoral neck and shaft fractures to range from 2.5-6% of femoral shaft fractures. They also mention that the delayed diagnosis rate is 20-30% depending on the study. According to this study, the femoral neck fracture line is almost vertical and nondisplaced or minimally displaced in 26-59% of cases. (Hence, from that one can also assume approx 40% are significantly displaced).

RECOMMENDED READINGS:

Bennett FS, Zinar DM, Kilgus DJ: Ipsilateral hip and femoral shaft fractures. Clin Orthop 1993;296:168-177. Wolinsky PR, Johnson KD: Ipsilateral femoral neck and shaft fractures. Clin Orthop 1995;318:81-90.

40. Which of the following is considered a critical step in the pathologic process of metastasis for a malignant cell?
1. 2. 3. 4. 5. Loss of cell motility Intravasation Loss of extravasation Prevention of angiogenesis Increased apoptosis

PREFERRED RESPONSE: 2

From Buckwalters book, Metastasis is an extremely complex process, requiring a series of coordinated cellular events and unique behaviors. The steps in the process are: cell motility (WRONG ANSWER 1), invasion of normal tissue matrices, transgression of endothelial basement membranes or intravasation (CORRECT ANSWER 2), attachment to endothelium at remote site, transgression of endothelial basement membrane in organ of implantation or extravasation (WRONG ANSWER 3), invasion of local host, colony establishment and proliferation in new environment, induction of local angiogenesis to support tumor growth (WRONG ANSWER 4), and possible repetition of the cycle of metastasis from the new site. The bone destruction secondary to metastases is actually caused by the activation of osteoclasts, not direct destruction of bone by tumor cells. That process is via neovascularization and blood vessel invasion, embolism of multi-cell aggregates that then arrest in bone compartments which then activate osteoclasts. Mundy and Yoneda described the cellular events in the metastatic process: attachment of tumor cells to the basement membrane, production by the tumor cells of proteolytic enzymes that can disrupt the basement membrane, migration of the tumor cells through the basement membranes under the specific control of chemotactic factors, and capability of the tumor cells to stimulate the activity of the osteoclasts that are responsible for degrading bone and causing osteolytic lesions. Apoptosis is programmed cell death and not directly related to the process of metastasis, but really is related to the process of malignancy (WRONG ANSWER 5).
RECOMMENDED READINGS: Buckwalter JA, Einhorn TA, Simon SR (eds): Orthopaedic Basic Science: Biology and Biomechanics of the Musculoskeletal System, ed 2. Rosemont, IL, American Academy of Orthopaedic Surgeons, 2000, pp 19-76. Menendez LR (ed): Orthopaedic Knowledge Update: Musculoskeletal Tumors. Rosemont, IL, American Academy of Orthopaedic Surgeons, 2002, pp 305-312.

41. When comparing antegrade nailing of femoral shaft fractures treated with a fracture table versus manual traction, the manual traction group had significantly less
1. 2. 3. 4. 5. fluoroscopy time. shortening of the femur. assistants per case. exchange nailings. internal rotational deformities.

PREFERRED RESPONSE: 5
This question is directly from the JBJS article by Stephen et al. The study randomized 87 femur fractures into manual traction (position supine w/ bump to elevate operative side 30 degrees) and fracture table cases of IM Nailing of femur shaft fractures. Their goal was to compare quality of reduction, operative time, complications, and functional status of the patients. They used CT scan to check for quality of reduction and rotation post-op. The results of the study were: no significant differences between the groups with respect to age, gender, GCS, ISS, side or mechanism of injury, fracture type, or time from injury to treatment. Internal malrotation was significantly more common when the fracture table had been used (internally rotated by >10) as compared to the group treated with manual traction (p = 0.007) (CORRECT ANSWER 5). Total operative time, from the beginning of the patient positioning to the completion of the skin closure, was decreased in the manual traction group. There was no significant difference between the two treatment groups with regard to the number of assistants per case (WRONG ANSWER 3), fluoroscopy time (WRONG ANSWER 1), and other complications including femoral shortening or lengthening (WRONG ANSWERS 2 and 4) or functional status of the patient at one year. The internal malrotation seen after use of the fracture table may have resulted from the practice of internally rotating the limb to facilitate access to the starting point and to improve visualization of the hip in the lateral plane. Furthermore, once the limb is positioned on the fracture table and the procedure is started, it can be difficult to assess rotation clinically because the limb cannot be moved readily. A theoretical advantage of using the fracture table is that it can be done without assistants but in this study the average number and experience of scrubbed assistants were the same (WRONG ANSWER 3). Reported disadvantages to using fracture tables include: pudendal nerve palsies, well-leg compartment syndromes, and skin sloughs of the perineum. The Wolinsky article was an earlier study on the same subject and their goal was to compare and contrast operative times in fracture table vs. manual traction for femoral nailings. Their study was consistent w/ the JBJS article stating that manual traction is significantly faster. This study doesnt comment on the other variables asked i n this question.

RECOMMENDED READINGS: Stephen DJ, Kreder HJ, Schemitsch EH, et al: Femoral intramedullary nailing: Comparison of fracture-table and manual traction. J Bone Joint Surg Am 2002;84:1514-1521. Wolinsky PR, McCarty EC, Shyr Y, et al: Length of operative procedures: Reamed femoral intramedullary nailing performed with and without a fracture table. J Orthop Trauma 1998;12:485-495.

42. A 13-year-old patient with sickle cell anemia has progressive forearm pain and diffuse fusiform swelling of the forearm. Radiographs show periostitis along the distal metaphysis of the radius. To help differentiate between osteomyelitis and bone infarction, which of the following studies should be ordered? 1. Sequential radiographs 2. Sequential bone marrow scans and a technetium Tc 99m bone scan 3. Single photon emission computed tomography bone scan 4. Gallium scan and CT 5. Positron emission tomography scan

PREFERRED RESPONSE: 2 The differentiation of bone infarction from acute osteomyelitis in patients with sickle-cell disease is
challenging because the clinical presentations of the two diseasess are similar. In both conditions, a child may have a painful, swollen, and tender limb that has a limited range of motion. Fevers are common in both conditions, and interpretation of the erythrocyte sedimentation rate is difficult. Xrays are of little value in differentiating bone infarction from acute osteomyelitis at the initial presentation, as they may appear normal or show periosteal new bone along the diaphysis in either condition. (WRONG ANSWER 1). Blood cultures are only + in 50% of cases of osteomyelitis. Bone infarction leads to decreased uptake on bone marrow scan and acute osteomyelitis leads to normal uptake. Both conditions lead to variable uptake on bone scan (WRONG ANSWER 3). Bone infarction is much more common and the treatment is conservative and w/o long term antibiotics. Therefore appropriate identification of osteomyelitis is important. The Skaggs article sought to see if these two modalities together may help diagnose osteomyelitis as both alone have proven inadequate. Their results show, bone infarction during a vasoocclusive crisis led to reduced activity of the radionuclide on bone-marrow scans and corresponding abnormal activity on bone scans. In contrast, acute osteomyelitis resulted in normal activity on bone-marrow scans and abnormal activity on bone scans. These findings indicate that the combination of sequential bone-marrow and bone scans within a twenty-four-hour period is a useful tool for early differentiation between osteomyelitis and bone infarction (CORRECT ANSWER 2). Bone-marrow scanning targets the reticuloendothelial system of bone marrow and resident white blood cells in the marrow. In contrast, bone scanning reflects reparative osteoblastic response. Some studies have reported moderate success to differentiate the two w/ ultrasound. More evidence is needed to determine if MRI will be successful to differentiate the two processes and nuclear medicine studies have proven to be more cost-effective than magnetic resonance imaging, especially in children with multiple sites of abnormality. Furthermore, radionuclide imaging permits an assessment of the entire appendicular and axial skeleton, allowing for identification of multiple sites of involvement. Finally, radionuclide imaging may be more accurate than magnetic resonance imaging in children because of the high signal intensity of hematopoietic bone, in contrast with the increased fat content of adult bone marrow. The Chambers study reports limited experience w/ bone scan and says that gallium scans arent used for this purpose (WRONG ANSWER 4). None of the recommended readings said anything about using PET scans to diagnose these diseases (WRONG ANSWER 5).

RECOMMENDED READINGS:
Skaggs DL, Kim SK, Greene NW, et al: Differentiation between bone infarction and acute osteomyelitis in children with sickle-cell disease with use of sequential radionuclide bone-marrow and bone scans. J Bone Joint Surg Am 2001;83:1810-1813. Sponseller PD (ed): Orthopaedic Knowledge Update: Pediatrics 2. Rosemont, IL, American Academy of Orthopaedic Surgeons, 2002, pp 27-41. Chambers JB, Forsythe DA, Bertrand SL, et al: Retrospective review of osteoarticular infections in a pediatric sickle cell age group. J Pediatr Orthop 2000;20:682-685.

Will Min

43. A 54-year-old man has had pain radiating from his buttock to his foot after lifting a heavy box 2 weeks ago. Examination reveals decreased sensation within the first web space. MRI scans are shown in Figures 14a and 14b. Initial management should consist of 1- an epidural steroid injection. 2- IV steroids for 24 hours. 3- a short course of oral steroids. 4- surgical diskectomy. 5- physical therapy and anti-inflammatory drugs.

PREFERRED RESPONSE: 5
EXPLANATION: The presentation and images are consistent with a lumbar disc herniation affecting the L5 distribution. In the treatment of lumbar disc hernations, the natural history of this disorder is variable and incompletely understood. However, Saal, et al. (Spine 1989) found that nonoperative treatement, consisting of an aggressive rehabilitation program, resulted in good or excellent outcomes in 90% of patients. Their follow-up study (referenced below) was a follow-up of MRI findings, which documented no increased risk for perineural fibrosis or adhesions with nonsurgical management. Webers prospective randomized study found that the operative patients had statistically better results compared with nonoperative patients at 1- and 4- year follow-up. However, statistical differences were not found at 10year follow-up (even though surgical patients still tended to do better than the nonsurgical patients). While Webers study reports slightly more modest results as compared to Saals studies when considering non-operative treatment, both do suggest that an initial course of non-operative management in the early onset of lumbar disc herniations with physical therapy and medication may yield satisfactory outcomes.

RECOMMENDED READINGS: Weber H: Lumbar disc herniation: A controlled, prospective study with ten years of observation. Spine 1983;8:131-140. Saal JA, Saal JS, Herzog RJ: The natural history of lumbar intervertebral disc extrusions treated nonoperatively. Spine 1990;15:683-686.

44. Which of the following is considered the most common presentation of child abuse? 1- Skin lesions 2- Fractures 3- Developmental delay 4- Abdominal injury 5- Ocular lesions

PREFERRED RESPONSE: 1
EXPLANATION: In their review of 371 children, McMahon et al. (JBJS, 1995) found that soft-tissue injuries were present in 92% of children suspected of being victims of child abuse. Any soft-tissue injury in a child younger than 9 months is suspicious and indicates possible abuse. Soft-tissue injuries of the head and face are much more common in abused children and are rare in the absence of abuse. Abdominal trauma is the second leading cause of death due to child abuse, while head trauma is the most common cause of death due to child abuse. RECOMMENDED READINGS: Akbarnia BA, Campbell RM: The role of the orthopaedic surgeon in child abuse, in Morrissy RT, Weinstein SL (eds): Lovell and Winters Pediatric Orthopaedics, ed 5. Philadelphia, PA, Lippincott Williams & Wilkins, 2001, pp 14231445. Kocher MS, Kasser JR: Orthopaedic aspects of child abuse. J Am Acad Orthop Surg 2000;8:10-20.

45. Following successful replantation of a finger, what is the most common secondary surgery likely to be performed? 1- Capsulotomy 2- Arthrodesis 3- Tenolysis 4- Neurolysis 5- Delayed amputation

PREFERRED RESPONSE: 3
EXPLANATION: Wangs article discusses the incidence and sequence of secondary procedures following digital replantation. In published series addressing this topic, the overall frequency of postreplantation surgery ranged from 2.9-93.2%. Tendon procedures accounted for 47.2% of cases, and comprised the leading type of secondary operations. Tendon procedures were followed by joint procedures (18.9%), skeletal stabilization (12%), skin coverage (11.4%), nerve reconstruction (8.9%), and late amputation (1.6%). The number of secondary procedures per patient averaged from 1-4.5, depending on prioritization of different procedures. The order of restoration procedures should be as follows: supple skin coverage, skeletal stability, and protective and proprioceptive sensation. Joint reconstruction follows these procedures, which in turn is followed by tendon reconstruction. RECOMMENDED READINGS: Waikakul S, Sakkarnkosol S, Vanadurongwan V, et al: Results of 1018 digital replantations in 552 patients. Injury 2000;31:33-40. Wang H: Secondary surgery after digit replantation: Its incidence and sequence. Microsurgery 2002;22:57-61.

46. Stripe wear in ceramic-on-ceramic hip arthroplasty indicates which of the following problems? 1- Damage to the liner at the time of its insertion into the shell 2- Damage to the femoral head because of lift-off separation of the femoral head during gait 3- Wear of the acetabular liner because of third-body wear 4- Wear of the Morse taper portion of the femoral head because of corrosion 5- Backside wear of the acetabular liner

PREFERRED RESPONSE: 2
EXPLANATION: Yamamoto, et al. investigated the articular bearing surfaces retrieved from three patients with ceramic-on-ceramic total hip arthroplasties using highquality alumina. The duration of implantation was 3.5, 3.0, and 2.5 years, respectively. Dislocation of the hip joint after implantation did not occur in any patients. The retrieved prostheses were examined by stereomicroscopy and scanning electron microscopy (SEM). In the main load-bearing area, no wear marks were seen in any sockets. However, in all three cases, stripe scars were observed at the rim of the alumina inlay under stereomicroscopy, despite the lack of indentation in the socket and the stem, which suggests impingement. Such scars were not observed on the load-bearing surface. Under SEM, the scars showed excessive wear, including material excavation, that has not been observed in conventional hip simulator experiments. These findings are similar to those of experiments in which a simulator was programmed to distract the femoral head from the socket. The present results suggest that the femoral head can separate from the socket without neck-socket impingement in vivo. RECOMMENDED READINGS: Yamamoto T, Saito M, Ueno M, et al: Wear analysis of retrieved ceramic-on-ceramic articulations in total hip arthroplasty: Femoral head makes contact with the rim of the socket outside of the bearing surface. J Biomed Mater Res B Appl Biomater 2005;73:301-307. Manaka M, Clarke IC, Yamamoto K, et al: Stripe wear rates in alumina THR-comparison of microseparation simulator study with retrieved implants. J Biomed Mater Res B Appl Biomater 2004;15:149-157.

47. A 75-year-old woman reports an enlarging mass around her right thigh. Figures 15a through 15d show radiographs and biopsy specimens. What is the most likely diagnosis? 1- Metastatic breast carcinoma 2- Multiple myeloma 3- Myositis ossificans 4- Chondrosarcoma 5- Osteosarcoma

PREFERRED RESPONSE: 5
EXPLANATION: The brief clinical history, X-Ray examinations, and histological findings are consistent with the diagnosis of Osteosarcoma. Osteosarcoma is the most common primary malignancy of bone and occurs predominantly in teenagers and young adults. The incidence is 400 cases per year in the US and males are slightly more often affected. Half of the lesions are located in the distal femur or proximal tibia. The proximal humerus, proximal femur and pelvis are the next most common sites. Patients may present with pain and swelling, but usually do not have systemic symptoms and feel well. On physical examination the mass typically is slightly tender, firm, and fixed to the bone. On laboratory studies alkaline phosphatase and lactate dehydrogenase can be elevated. Usually osteosarcoma is discovered on plain radiographs revealing bony destruction, elevation of the periosteum that may appear as the characteristic Codman triangle and in some cases a contiguous soft tissue mass. A CT of the primary lesion to delineate the location and a high resolution CT scan of the chest to rule out pulmonary metastases should be obtained. Skeletal scintigraphy, which typically shows a defined region of increased tracer uptake, is useful to determine the extent of the lesion, to evaluate for metastases to the bone, and for early detection of local recurrence. Histologically, regions of malignant mesenchymal stroma and areas of neoplastic osteoid scattered in & about lobules of low grade mature cartilage is seen. Normally appearing trabeculae are absent, and chondroblastic differentiation may be seen, which by itself might indicate periosteal chondrosarcoma.

RECOMMENDED READINGS: Hillman A, Hoffmann C, Gosheger G, et al: Malignant tumor of the distal part of the femur or the proximal part of the tibia: Endoprosthetic replacement or rotationplasty. Functional outcome and qualityof-life measurements. J Bone Joint Surg Am 1999;81:462-468. Kawai A, Muschler GF, Lane JM, et al: Prosthetic knee replacement after resection of a malignant tumor of the distal part of the femur: Medium to long-term results. J Bone Joint Surg Am 1998;80:636-647. Manoso MW, Healey JH, Bolond PJ, et al: Denovo osteogenic sarcoma in patients older than forty. Clin Orthop 2005;438:128-136.

48. An active 46-year-old woman has had an 18month history of progressive hindfoot pain that has failed to respond to nonsurgical therapy. She has an asymmetrical planovalgus foot, an equinus contracture, and a flexible deformity. In addition to a lengthening of the gastrocsoleus complex, recommended surgery should include 1- in situ triple arthrodesis. 2- isolated flexor digitorum longus (FDL) transfer to the medial cuneiform. 3- lateral column lengthening, medializing calcaneal osteotomy, and FDL transfer to the navicular. 4- lateralizing calcaneal osteotomy, FDL transfer to the navicular, and peroneus longus to peroneus brevis tendon transfer. 5- plantar flexion first tarsometatarsal fusion (Lapidus) and lateralizing calcaneal osteotomy.

PREFERRED RESPONSE: 3
EXPLAINATION: In patients with posterior tibial tendon insufficiency (which is the most common cause of acquired adult flatfoot deformity), the condition is classificed on the basis of clinical and radiographic findings into four stages: stage I no notable clinical deformity, with clinical complaints of pain along the course of the tendon and local inflammatory changes; stage II dynamic deformity of the hindfoot; stage III fixed deformity of hindfoot and fixed forefoot supination deformity (but no obvious evidence of ankle deformity; stage IV ankle involvement secondary to long-standing fixed hindfoot deformities. The patient in this vignette is classified as a stage II. Opinions regarding treatment of stage II conditions has the greatest degree of controversy. The historical foundation for treatment of this stage of posterior tibial tendon insufficiency is provided by Mann, et al., JBJS 1985, who reported on acceptable pain relief following debridement of the posterior tibial tendon and transfer of the FDL to the navicular. However, soft-tissue transfer alone in this case does not correct the underlying deformity, and bone procedures are incorporated to supplement the dynamic transfers to replace the insufficient posterior tibial tendon. Myerson, et al.s article highlighted the early successful results of joint-sparing operations (medializing osteotomy of the calcaneal tuberosity in addition to tendon transfers, medializing calcaneal osteotomy, and lateral column-lengthening osteotomy through the anterior calcaneus, and heel-cord lengthening).

RECOMMENDED READINGS: Beals TC, Pomeroy GC, Manoli A II: Posterior tendon insufficiency: Diagnosis and treatment. J Am Acad Orthop Surg 1999;7:112-118. Mosier-LaClair S, Pomeroy G, Manoli A II: Intermediate follow-up of the double osteotomy and tendon transfer procedure for stage II posterior tibial tendon insufficiency. Foot Ankle Int 2001;19:520-524. Myerson MS, Corrigan J, Thompson F, et al: Tendon transfer combined with calcaneal osteotomy for treatment of posterior tibial tendon insufficiency: A radiological investigation. Foot Ankle Int 1995;16:712-718.

Chris Ropiak

49. Which of the following alphabet soup lesions of the shoulder will more likely require open rather than arthroscopic repair?
1- Anterior labroligamentous periosteal sleeve avulsion (ALPSA) 2- Partial articular supraspinatous tendon avulsion (PASTA) 3- Superior labrum anterior and posterior (SLAP) 4- Humeral avulsion of the glenohumeral ligament (HAGL) 5- Glenolabral articular disruption (GLAD)

PREFERRED RESPONSE: 4
Stein et als article reviews arthroscopic treatment of various causes of anterior shoulder instability. Listed amongst contraindications to arthroscopic repair are patients whose anterior instability is due to glenoid bone loss, attenuated capsulolabral tissue, engaging Hill Sachs lesions, and HAGL lesions. ALPSA, PASTA, SLAP lesions all had comparable success rates when treated with open vs arthroscopic repair. Eliminating choices 1,2,3.
The second reference speaks of 5 cases of glenolabral articular disruptions (GLAD), as being another source of anterior shoulder pain, though is not associated with anterior shoulder instability. At surgery, all 5 of these lesions were all associated with superficial anterior inferior labral tear. The history of all lesions included an adduction force across the chest from an abducted and externally rotated position. Arthroscopic debridement of the labral tear with glenoid articular chondroplasty or abrasion arthroplasty is the treatment of choice, with all symptoms being relieved after 3 months of surgical debridement. Eliminating choice 5.

RECOMMENDED READING: Stein DA, Jazrawi L, Bartolozzi AR,: Arthroscopic stabilization of anterior shoulder instability: A review of the literature. Arthroscopy 2002; 18:912-924. Neviaser TJ: The GLAD lesion: Another cause of anterior shoulder pain. Arthroscopy 1993;9:22-23

50. The magnitude of the difference in the means of the control and experimental groups in a study with respect to the pooled standard deviation defines the
12345study power variance confidence interval effect size false positive

PREFERRED RESPONSE: 4
Definitions:
Study power the probably of rejecting the null hypothesis when it is indeed false. It can be calculated using the probability of a type 1 error (a) and that of a type 2 (b). Study power can be calculated as 1 - b. Variance - in a population of samples, the mean of the squares of the differences between the respective samples and their mean, expressed mathematically as:

where n is the number of samples, x i is the value of sample, i is the mean of the samples, and 2 is the variance. Note: The square root of the variance, , is the standard deviation.

Confidence interval - A statistical range with a specified probability that a given parameter lies within the range. Confidence intervals are usually calculated so that this percentage is 95%. Effect size - is the size of the relationship between two variables and is usually defined as the difference in mean outcomes between the treatment and control group. Effect size is also an indicator of the strength of the difference between two groups. False positive A positive test result in a subject that does not possess the attribute for which the test is being conducted. RECOMMENDED READING: Therefore the correct answer is 4
Buckwalter JA, Einhorn TA, Simon SR (eds): Orthopaedic Basic Science: Biology and Biomechanics of the Musculoskeletal System, ed 2. Rosemont, IL, American Academy of Orthopaedic Surgeons, 2000. pp 1-17

51. To minimize the risk of iatrogenic traction injury to the sciatic nerve during open reduction and internal fixation of the acetabulum through the Kocher-Langenbeck exposure, the leg should be in what position?
12345extended at the hip and extended at the knee extended at the hip and flexed at the knee flexed at the hip and extended at the knee flexed at the hip and flexed at the knee abducted at the hip and extended at the knee

PREFERRED RESPONSE: 2
In the reference paper given the investigators used fresh cadaveric specimens with pressure transducers in the sciatic nerve just distal to the femoral attach of the gluteus maximus. Intraneural pressures were measured with the hip placed in 0, 45, and 90 degrees of flexion while the knee was positioned in 90, 45, and 0 degrees of flexion. They found that tissue fluid pressure within the sciatic nerve exceeded previously defined critical thresholds for alteration of neural microcirculation and function. Although increased intraneural pressures were realized as the hip was positioned in greater flexion and the knee was extended, clinically relevant pressures were realized only when the hip was flexed to 90 degrees and the knee was fully extended. Pressures with the limbs in these positions were fifty-five millimeters of mercury (range 38 to 74 millimeters of mercury). In conclusion, intraneural sciatic nerve pressures are influenced by the position of the ipsilateral hip and knee, with the magnitude of the pressure elevation being related to the excursion of the nerve as the linear distance between the greater sciatic notch and the distal aspect of the leg increases. The position with the least excursion therefore is with the hip extended and the knee flexed. Making choice 2 the correct response. RECOMMENDED READING: Borrello J Jr, Kantor J, Ungacta F, et al: Intraneural sciatic nerve pressures relative to the position of the hip and knee: A human cadaveric study. J Orthop Trauma 2000;14:255-258

52. An active 23-year-old man injured his ring finger playing football 3 months ago. Examination reveals a palmer mass at the base of the ring finger, and he has absent distal interphalangeal joint flexion but full passive motion. Radiographs are shown in Figures 16a and 16b. What treatment option will best restore full finger function?
1- Direct repair of the flexor digitorum profundus tendon 2- Insertion of an active silicone tendon rod implant 3- One-stage flexor tendon grafting 4- Two-stage flexor tendon grafting 5- Excision of the palmar mass and distal interphangeal joint fusion

PREFERRED RESPONSE: 4
The situation in this question involves an FDP tendon rupture that has scarred down, but with good PROM of the DIP. Choice 1 is incorrect because the scarred FDP will not be able to be repaired this far out from the injury due to contraction of the tendon and scarring. Choice 2 is incorrect because I am not sure what is meant by an active silicon tendon rod implant. That sounds like they are talking about stage I of a two staged procedure. Choice 3 is incorrect because one-stage flexor tendon grafting is generally contraindicated in flexor tendon ruptures that are more than 6 weeks old. Typical tendon grafts include palmaris longus, extensor indicis, and toe extensors. Choice 5 is incorrect because the question asks which treatment option will best restore full finger function. This choice has you fusing the DIP joint not trying to restore function. In two-stage flexor tendon reconstruction, stage I involves excising the scarred tendon remains (with care taken to preserve the pulley) and the implantation, typically, of a silicon-dacron tendon spacer implant. During wound healing, a passive exercise program is used to mobilize the finger prior to the second stage of the reconstruction. Typically, stage II is performed after 3 months. By this time, a pseudo-synovial sheath should have formed. During this part of the procedure the implant is replaced with a long tendon graft, with care taken to disturb the newly formed sheath as little as possible. In the paper published by LaSalle and Strickland in 1983, they reported 27% excellent, 28% good, and 30% fair results following two-staged flexor tendon reconstruction for restoration of a digital function following flexor tendon loss in a badly scarred digit. Because I am well versed in orthopedic literature that was published when I was four-years-old, I knew that the best treatment option for the above scenario is a two-stage flexor tendon graft, Choice 4. Greens lists as indications for two -stage flexor tendon reconstruction: severely injured digits, involving crush injuries associated with underlying fracture or overlying skin damage and excessive scarring of the tendon bed. Can I get a, Good Job, Buddy?

RECOMMENDED READING: Amadio PC, Wood MB, Cooney WP III, et al: Staged flexor tendon reconstruction in the fingers and hand. J Hand Surg Am 1988;13:559-562. LaSalle WB, Strickland JW: An elavulation of the two-stage flexor tendon reconstruction technique. J Hand Surg Am 1983;8:263-267. Schneider LH: Flexor tendons: Late reconstruction, in Green DP, Hotchkiss RN, Pederson WC (eds):Greens Operative Hand Surgery. Ed 4 Philadelphia, PA, Churchill Livingstone, 1999, pp 1898-1949.

53. A 30-year-old man elects to undergo a Syme (through the ankle) amputation following a gunshot wound to his foot. Examination reveals that the plantar skin is intact and there is no sign of infection. When performing this procedure, which of the following factors is the most important to ensure a good result?
12345Perform the procedure in two stages. Leave both malleoli intact. Avoid a hypermobile heel pad. Attach the tibialis anterior to the distal tibia. Place a drain through the plantar heel pad.

PREFERRED RESPONSE: 3
A Symes amputation is a disarticulation of the ankle with removal of the talus and calcaneous, while carefully preserving the plantar skin and heel pad to cover the distal tibia. The malleoli should also be removed to help narrow the stump, which helps prosthetic fitting. Choice 1 is incorrect because in OKU: Foot and Ankle 2, it states that in the past it was recommended that this amputation be done in two stages, delaying the removal of the malleoli by about 6 weeks; but a recent prospective randomized study demonstrated similar wound healing results in onestage versus two-stage procedures. Thus, there is no benefit of making this a two surgery amputation. Choice 2 is incorrect because a Symes amputation includes the removal of the malleoli, and keeping the malleoli intact does not ensure a good result. Of note, the Brodsky citation states that the malleoli should not be resected in children unless severe pressure problems arise, and then they should be resected below the physeal line. Choice 4 is incorrect because during the operation the anterior tendons are pulled down with a clamp and divided so that they can retract. Choice 5 is incorrect because placing a drain through the plantar heel pad does by no means help ensure a good result and none of the references addressed such placement of a drain. Choice 3 is the preferred response for the following reason. In the Brodsky citation it lists the most common complication of this surgery is diabetic and dysvascular patients failing to heal. This question does not paint the picture of a vasculopath. The second most common problem with this procedure is a
hypermobile heel pad. Therefore, the correct choice is 3. RECOMMENDED READING: Brodsky JW: Amputations and prostheses of the foot and ankle, in Coughlin MJ, Mann RA (eds): Surgery of the Foot and Ankle, ed 7. St Louis, MO, Mosby, 1999, pp 970-1006. Mizel MS, Miller RA, Scioli MW (eds): Orthopaedic Knowledge Update: Foot and Ankle 2. Rosemont, IL, American Academy of Orthopaedic Surgeons, 1998, pp 305-314.

54. What is the primary risk factor for the development of a diabetic foot ulcer?
1- Peripheral vascular disease 2- Malnutrition related to chronic renal disease 3- Motor neuropathy 4- Decreased oxygen tension of the skin 5- Loss of protective sensation from the peripheral neuropathy

PREFERRED RESPONSE: 5
Although all of the above choices may be contributing to a diabetic foot ulcer, neuropathy, especially sensory neuropathy, is the most preeminent source or initiating even of almost all ulcerations in diabetics. The loss of protective sensation combined with acute trauma, recurrent trauma, repetitive trauma, or even microtrauma leads to most instances of soft tissue breakdown in the diabetic foot straight from the Brodsky citation and echoed by the OKU citation. Choice 5.

RECOMMENDED READING: Koval KJ (ed): Orthopaedic Knowledge Update 7. Rosemont, IL, American Academy of Orthopaedic Surgeons, 2002, pp 565-578. Brodsky JW: Amputations and prostheses of the foot and ankle, in Coughlin MJ, Mann RA (eds): Surgery of the Foot and Ankle, ed 7. St Louis, MO, Mosby, 1999, pp 895969.

Jeff Van Gelderen

55. Which of the following mechanisms of injury is most likely to result in an isolated posterior cruciate ligament injury? 1- External rotation - valgus knee injury 2- Internal rotation - varus knee injury 3- Direct blow to the posterior knee 4- Fall on the knee with the foot plantar flexed 5Fall on the knee with the foot dorsiflexed

PREFERRED RESPONSE: 2
Posterior cruciate ligament (PCL) injuries have a reported incidence of between 3 and 37%, depending on the clinical setting. The most common mechanism of injury in motor vehicle accidents is a dashboard injury or direct force to the proximal anterior tibia. Sports related injuries result from hyperflexion of the knee with the foot typically plantarflexed. The latter mechanism is the most common cause of isolated PCL injuries, while in the trauma population as many as 95% of patients with knee injuries have combined ligamentous damage. Improved knowledge at an anatomical, biomechanical and clinical level has provided the orthopaedist with a more defined treatment algorithm. Isolated, partial PCL injuries (grades I and II) can best be treated nonoperatively while complete injuries (grade III) may require operative treatment based on clinical symptoms. All combined ligamentous injuries usually respond best with surgical management. As we do not see many isolated PCL injuries in clinic, it is insightful to ask the etiology of these injuries. There is considerable variability in the reported incidence of PCL injuries in the literature. This disparity clearly results from differences in the population of individuals examined. The most frequent mechanism of injury in this population was a hyperflexion injury. By contrast, in an emergency room setting where 56.5% of injuries were traumatic-relatedand 32.9% were athletic-related, Although it was difficult to delineate the exact mechanism of injury in all cases, the majority of PCL tears occurred in motor vehicle and motorcycle accidents.

RECOMMENDED READINGS: Janousek AT, Jones DG, Clatworthy M, et al: Posterior cruciate ligament injuries of the knee joint. Sports Med 1999;28:429-441. Schulz MS, Russe K, Weiler A, et al: Epidemiology of posterior cruciate ligament injuries. Arch Orthop Trauma Surg 2003;123:186-191.

Three mechanisms have been proposed for rupture of the PCL (see fig below).

Hyperflexion The most common injury in sport is a hyperflexion injury, in which the patient falls onto a flexed knee. The traumatic forces are directed proximally up the tibia, and the tibia subluxes posteriorly. As the knee flexes, the anterolateral band of the PCL tightens.With sudden hyperflexion, the tension of the PCL increases beyond its elastic limits and plastic deformation or rupture occurs. The PCL impinges between the posterior tibial plateau and roof of the femoral notch, and the PCL is guillotined. There is nearly always an isolated intrasynovial intrasubstance tear and often the posteromedial bundle remains intact. There is rarely greater than grade II laxity. These lesions heal to some extent and with time the laxity may decrease one grade. Pretibial Trauma The most common traumatic mechanism is the dashboard injury. The knee is in a flexed position and a posteriorly directed force is applied to the pretibial area. An intrasubstance tear at the level of the tibial plateau or a tibial avulsion occurs. Significant trauma will result in damage to the meniscofemoral ligaments. If the force is anteromedial and a rotational component is present the posterolateral corner can be torn. Pretibial trauma with the knee near extension initially tightens the posteromedial bundle. Thus, the PCL may sleeve off its origin. These patients often have greater instability than the hyperflexion group. Hyperextension Hyperextension can result in tearing of the PCL and posterior capsule and can progress to dislocation and neurovascular compromise. The PCL injury is frequently a proximal disruption at the femoral attachment.

56. A 14-year-old male athlete sustains a twisting, valgus stress injury to his knee. Radiographs are shown in Figures 17a and 17b. The analogous knee injury in the adult would include 1- anterior and posterior cruciate ligament tears. 2- an anterior cruciate ligament tear and a posterolateral capsule avulsion. 3- a posterior cruciate ligament tear and a deep medial collateral ligament tear. 4- a posterior cruciate ligament tear and a posterolateral capsule avulsion. 5- a posterior cruciate ligament tear and a posteromedial capsule avulsion.

PREFERRED RESPONSE: 2
The point of this question is to contrast an adolescent knee injury with a segond fracture in an adult. The Segond fracture is an avulsion fracture involving the proximal tibia immediately distal to the lateral plateau. It was described by Paul Segond in 1879, in cadaveric experiments, as a cortical avulsion of the tibia at the site of insertion of the middle third of the lateral capsular ligament (LCL) resulting from excessive internal rotation and varus stress. Numerous studies have demonstrated an association of the Segond fracture with tears of the anterior cruciate ligament (75%100% of patients), meniscal tears (66%75% of patients), damage to the structures of the posterolateral corner of the knee, and other avulsion injuries. Thus, the presence of a Segond fracture may indicate substantial meniscoligamentous injury, and anterolateral rotational instability must be considered to be present until proven otherwise. The associated disruption of the anterior cruciate ligament accentuates this instability. However, the clinical diagnosis in the acute stage is difficult due to pain, muscle spasm, hemarthrosis, and edema. Thus, it is important to recognize this seemingly small bone injury, which reflects a major ligamentous disruption, to avoid the potential clinical consequence of late knee instability if the necessary treatment is withheld.
RECOMMENDED READINGS: Price DT, Phillips JH, DeVito DP: Management of fractures, in Morrissy RT, Weinstein SL (eds): Lovell and Winters Pediatric Orthopaedics, ed 5. Philadelphia, PA, Lippincott Williams & Wilkins, 2001, pp 1319-1422. Campos JC, Chung CB, Lektrakul N, et al: Pathogenesis of the Segond fracture: Anatomic and MR imaging evidence of an iliotibial tract or anterior oblique band avulsion. Radiology 2001;219:381-386. Kocher MS, Mandiga R, Klingele K, et al: Anterior cruciate ligament injury versus tibial spine fracture in the skeletally immature knee: A comparison of skeletal maturation and notch width index. J Pediatr Orthop 2004;24:185-188.

This is contrasted to knee injuries in adolescents where both midsubstance anterior cruciate ligament (ACL) injury and tibial spine avulsion fracture occur. The purpose of the study cited in Kochers paper was to determine whether there are differences in skeletal maturation or notch parameters between these two groups. A retrospective case-control study of 25 skeletally immature patients with tibial spine fractures compared with 25 age- and sex-matched skeletally immature patients with midsubstance ACL injuries was performed. Bone age and notch width index were determined from preoperative radiographs. To better understand notch width index, please refer to the following:

FIGURE 1. Notch width index (NWI): the ratio of the width of the intercondylar notch (line AB) to the width of the distal femur (line CD) at the level of the popliteal groove on the notch view radiograph.

There were no significant differences between the tibial spine fracture group and the ACL injury group with respect to skeletal maturation, comparing bone age to chronological age (-0.5 vs. 0.3 years; P = 0.617). The ACL injury group had narrower notch indices than the tibial spine fracture group (0.230 vs. 0.253; P = 0.020). Thus, in a comparison of age- and sex-matched skeletally immature patients, those with midsubstance ACL injuries had a significantly narrower notch index than those with tibial spine fractures. This association may account for some of the variation in injury patterns in the skeletally immature knee.

57. A lower discharge Glasgow Coma Scale score in a patient with a femoral fracture and a head injury has been associated with 1- surgery within 24 hours after injury. 2- hypotension and hypoxia. 3- prolonged length of stay in the ICU. 4- major nonorthopaedic surgeries. 5- the presence of pneumonia.

Preferred Response: 2
Fracture of the femur is a morbid and potentially fatal injury. Management of femur fractures has evolved considerably. Intramedullary nailing is currently the procedure of choice in most trauma centers. Although internal fixation is clearly a major advance in the treatment of femur fractures, the optimal timing of fracture fixation is still debated. Although common, extremity injuries are appropriately given a lower priority than hemodynamic stabilization and treatment of life-threatening injuries to the head, thorax, and abdomen. Early fixation of fractures is defined as occurring within 24 hours of admission. Early internal fixation of long bone fractures is increasingly performed in the majority of patients. Rapid fracture fixation reduces inflammation at the fracture site, reduces pain and subsequent need for narcotic medications, and promotes early mobilization of patients and improved pulmonary mechanics. In patients with isolated femur injuries, expeditious fixation is associated with a decreased incidence of mortality and postoperative complications, especially pulmonary complications such as fat embolization syndrome (FES), pneumonia, pulmonary embolism (PE), and acute respiratory distress syndrome (ARDS).
RECOMMENDED READINGS: Scalea TM, Scott JD, Brumback RJ, et al: Early fracture fixation may be just fine after head injury: No difference in central nervous system outcomes. J Trauma 1999;46:839 -846. Brundage SI, McGhan R, Jurkovich GJ, et al: Timing of femur fracture fixation: Effect on outcome in patients with thoracic and head injuries. J Trauma 2002;52:299-307. Jaicks RR, Cohn SM, Moller BA: Early fracture fixation may be deleterious after head injury. J Trauma 1997;42:1-5.

The outcomes from the Brundage study argue for early fixation, despite any concomitant head or thorax injuries. In this study, Acute respiratory distress syndrome, pneumonia, hospital length of stay, and intensive care unit length of stay were lowest in the group fixed within 24 hours, even in patients with concomitant head or chest trauma. Fixation between 2 and 5 days was associated with a significantly increased incidence of acute respiratory distress syndrome, pneumonia, and fat embolization syndrome in patients with concurrent chest trauma (p < 0.0001). In head-injured patients, discharge Glasgow Coma Scale score was highest in the group fixed within 24 hours. Timing of operative fixation did not affect mortality. The Jaicks article, however, appears to be what they were looking for with this question. In summary, the findings were: early FF led to a significantly greater fluid administration in patients with head injuries, and (2) hypoxemia and hypotension, risk factors for secondary brain injury, appeared to be more common in the early FF group. In the head-injured patient, the avoidance of secondary brain injury is of paramount importance. Hypotension and hypoxemia are the two most significant determinants of secondary brain injury. Operative procedures may increase the risk of producing or prolonging episodes of hypotension or hypoxemia. These issues raise the question as to whether head-injured patients are a subset of multiple trauma patients in which early FF is actually harmful.

58. Which of the following factors is considered the strongest predictor of a fragility fracture for a patient? 1- Low bone mineral density 2- Inadequate dietary intake of calcium and vitamin D 3- Postmenopausal woman 4- Prior fragility fracture 5- A woman on hormone replacement therapy for several years

Preferred Response: 4
Numerous studies have reported increased risks of hip, spine, and other fractures among people who had previous clinically diagnosed fractures, or who have radiographic evidence of vertebral fractures. However, there is some variability in the magnitudes of associations among studies. In the JBMS study, they look at the literature and performed a statistical synthesis of the risk of future fracture, given a history of prior fracture. The strongest associations were observed between prior and subsequent vertebral fractures; women with preexisting vertebral fractures (identified at baseline by vertebral morphometry) had approximately 4 times greater risk of subsequent vertebral fractures than those without prior fractures. This risk increases with the number of prior vertebral fractures. Most studies reported relative risks of approximately 2 for other combinations of prior and future fracture sites (hip, spine, wrist, or any site). The confidence profile method was used to derive a single pooled estimate from the studies that provided sufficient data for other combinations of prior and subsequent fracture sites. Studies of peri- and postmenopausal women with prior fractures had 2.0 (95 % CI = 1.8, 2.1) times the risk of subsequent fracture compared with women without prior fractures. For other studies (including men and women of all ages), the risk was increased by 2.2 (1.9, 2.6) times. Patients with a history of prior fracture, therefore, should receive further evaluation for osteoporosis and fracture risk.
RECOMMENDED READINGS: Klotzbuecher CM, Ross PD, Landsman PB, et al: Patients with prior fractures have an increased risk of future fractures: A summary of the literature and statistical synthesis. J Bone Miner Res 2000;15:721-739. Koval KJ (ed): Orthopaedic Knowledge Update 7. Rosemont, IL, American Academy of Orthopaedic Surgeons, 2002, pp 141-154.

59. A 21-year-old football player reports recurrent anterior dislocation of the glenohumeral joint despite undergoing three surgeries to stabilize the shoulder. Radiographs of the affected shoulder and the normal contralateral shoulder are shown in Figures 18a and 18b. What is the most likely reason for the failure of the surgical procedures? 1- Large, engaging Hill-Sachs lesion 2- Significant defect of the anterior inferior glenoid 3- Unrecognized lateral capsular avulsion 4- Labral nonunion 5- Subscapularis deficiency

Preferred Response: 2

RECOMMENDED READINGS: Burkhart SS, Debeer JF, Tehrany AM, et al: Quantifying glenoid bone loss arthroscopically in shoulder instability. Arthroscopy 2002;18:488-491. Bernageau J, Patte D, Bebeyre J, et al: Value of the glenoid profil in recurrent luxations of the shoulder. Rev Chir Orthop Reparatrice Appar Mot 1976;62:142-147.

The defect alluded to in the question is significant due to the glenoids important role in shoulder stabilization. The glenoid is positioned by scapulothoracic motion to act as a stable platform for the humeral head during active arm abduction. Intuitively, it appears that maintaining the glenoid platform perpendicular to the direction of the net humeral force will optimize osseous contributions to glenohumeral stability as well as the mechanics of concavity compression. The importance of concavity compression and glenoid positioning may be reflected in the clinical experience that many MDI patients respond to a rehabilitative exercise program directed at improving strength and neuromotor coordination of the rotator cuff and scapular musculature. In the Burkhart article, there was an investigation of glenoid structure and its consistent anatomic landmarks as determined by arthroscopic means in live subjects and by direct measurement in fresh-frozen cadaver specimens. There was an arthroscopically evaluated and measured the location of the bare spot of the glenoid in 56 subjects that had no evidence of instability (average age, 40 years). They compared this to the exact location of the glenoid bare spot in 10 cadaver shoulders (average age, 76 years). The bare spot of the glenoid was a consistent reference point from which to determine glenoid bone loss because it was located almost exactly at the center of the circle that was defined by the articular margin of the inferior glenoid below the level of the midglenoid notch. The tightly clustered standard deviations of the bare spot measurements in both the live subjects and the cadaver specimens confirmed its consistent location.

60. When comparing a formal course of physiotherapy to a regimen of home exercises following an uncomplicated fracture of the distal radius, formal therapy offers which of the following results? 1- No significant benefits 2- Less wrist pain at 1 year 3- Better grip strength at 1 year 4- Better patient satisfaction at 1 year 5- Better hand function at 1 year

Preferred Response: 1
Fractures of the distal radius account for approximately 14% of all fractures and are seen most commonly in elderly women. In the area where the JBJS study was performed, around 950 people per year sustain this, and it often results in permanent disability, particularly in the elderly. Although physiotherapy is generally regarded as an important part of rehabilitation after fracture of the distal radius, there are few clinical data to support this view. Patients are either referred routinely to physiotherapy departments, or if they have symptoms which may interfere with recovery. These patients may receive physiotherapy for extended periods of time.

RECOMMENDED READINGS: Trumble TE (ed): Hand Surgery Update 3: Hand, Elbow, & Shoulder. Rosemont, IL, American Society for Surgery of the Hand, 2003, pp 105-121. Wakefield AE, McQueen MM: The role of physiotherapy and clinical predictors of outcome after fracture of the distal radius. J Bone Joint Surg Br 2000;82:972-976

The JBJS article looked at the role of home exercises vs. a dedicated PT regimen. The course of physiotherapy had no proven effect on the rate or quality of recovery after an uncomplicated fracture of the distal radius. This supports earlier studies which showed that home exercises given by a physiotherapist soon after the period of immobilization, are equally effective in restoring function. Many patients are currently being referred unnecessarily for physiotherapy, expending valuable time and effort by both the physiotherapist and patient. In order to identify the group of patients at risk of poor outcome, multivariate analysis showed that a patient over 55 years of age presenting with malunion at six weeks after a displaced fracture, with a history of functional impairment before the injury, is at high risk of poor outcome. A high level of pain, reduced rotation of the forearm and a reluctance to use the wrist and hand for functional activities also carry a poor prognosis. It was impossible formally to evaluate the effect of physiotherapy in these 'at-risk' patients because of the small size of the subgroups; until proven otherwise a course of physiotherapy should still be offered to these patients. In order to make useful comparisons between groups, all the modalities examined were standardized by expressing the degree of dysfunction as a percentage of the uninjured contralateral limb. This minimizes the variation within groups, which may often be considerably affected by a single individual and, in addition, gives a measure of return to normality for any one patient since each patient acts as his own control. ANCOVA was used in the direct comparison of the two groups since this method takes into account the widespread variation in baseline measurements between patients, as well as examining improvement in performance.

Brad Wasserman

61. A 17-year-old gymnast has had worsening midfoot pain and swelling for the past 3 weeks. She denies any trauma but admits to a recent increase in her training regimen. Radiographs are normal. CT and MRI scans are shown in Figures 19a and 19b. Management should now consist of
1 2 3 4 5 open reduction and internal fixation and bone grafting. a short leg non-weight-bearing cast. open reduction and internal fixation. percutaneous screw fixation. observation.

61. A 17-year-old gymnast has had worsening midfoot pain and swelling for the past 3 weeks. She denies any trauma but admits to a recent increase in her training regimen. Radiographs are normal. CT and MRI scans are shown in Figures 19a and 19b. Management should now consist of 1 2 3 4 5 open reduction and internal fixation and bone grafting. a short leg non-weight-bearing cast. open reduction and internal fixation. percutaneous screw fixation. observation.

PREFERRED RESPONSE: 2 RECOMMENDED READINGS: Khan KM, Fuller PJ, Bruckner PD, et al: Outcome of conservative and surgical management of navicular stress fracture in athletes: 86 cases proven with computerized tomography. Am J Sports Med 1992;20:657-666. Lee S, Anderson RB: Stress fractures of the tarsal navicular. Foot Ankle Clin 2004;9:85-104.

Scans show a nondisplaced navicular stress fracture. The history of a recent increase in training is a key part from the history. This fracture can rarely be detected in its early stages with plain xrays, and often results in a delay in diagnosis. It is often diagnosed on CT Scan. The 2nd article is a review article. It emphasized the ill-defined presentation of pt with this injury. Navicular stress fx results from repetitive cyclical loading on the forefoot, which is common in athletes; these injuries are uncommon in the general population. It is most common in explosive pushoff or change of direction sports. Onset of symptoms is slow and insidious. There needs to be a high index of suspicion in patients who present with midfoot or ankle pain, worsened w/WB activity, especially in sprinters and push-off athletes. Because of the difficulty in identifying these fx on plain xrays and the poor specificity of bone scans, CT Scan is the gold standard in diagnosis.
Surgical treatment should be considered for all pt who have displaced stress fx, nondisplaced complete fx with sclerotic changes or comminution, and those pt who fail conservative tx or cannot tolerate the prolonged course of recovery.

The 1st article compares the outcome of conservative and surgical tx of navicular stress fx in 86 patients followed for an average of 33 months. 19/22 (86%) pt who had initial NWB with cast immobilization greater than or equal to 6 weeks returned to sports with successful outcomes. Whereas 69% of pt had a successful outcome tx w/WB casts. 50% of pt had a successful outcome with limitation of full activity and full WB, compared to 20% with no limitation or activity restriction. Six patients were initially tx w/surgery, and 5/6 (83%) had a successful outcome. Surgery was used as a 2nd or subsequent tx for 22 fractures. When pt had not had surgery previously, (pt who failed limitation of activity w/WB or failed returning to sports) the surgical success was 68% (19/28). Bone grafting and screw fixation were the most commonly performed procedures, along with a post-op bone stimulator. The authors concluded that their tx of choice is NWB with cast immobilization for 6-8 weeks for nondisplaced fx. After 6-8 weeks the pt is re-examined. The absence of tenderness marks the beginning of functional rehab and return to activity.

62. During cemented bipolar hemiarthroplasty for treatment of a femoral neck fracture in an 80-year-old woman, the patient suddenly becomes hypotensive and hypoxic. What is the most likely reason for the change in the patients status?
1- Pulmonary embolism caused by venous trauma during hip reduction 2- Pulmonary embolism caused by preexisting deep venous thrombosis 3- Intramedullary fat embolization 4- Inadequate intraoperative fluid replacement 5- Vasodilation caused by methacrylate monomer

62. During cemented bipolar hemiarthroplasty for treatment of a femoral neck fracture in an 80-year-old woman, the patient suddenly becomes hypotensive and hypoxic. What is the most likely reason for the change in the patients status?
12345Pulmonary embolism caused by venous trauma during hip reduction Pulmonary embolism caused by preexisting deep venous thrombosis Intramedullary fat embolization Inadequate intraoperative fluid replacement Vasodilation caused by methacrylate monomer

PREFERRED RESPONSE: 3 RECOMMENDED READING: Kim YH, Oh SW, Kim JS: Prevalence of fat embolism following bilateral simultaneous and unilateral total hip arthroplasty performed with or without cement: A prospective, randomized clinical study. J Bone Joint Surg Am 2002;84:1372-1379.

This patient displays 1 of the typical findings characteristic of fat embolization syndrome hypoxia, which is the cause of the confusion/change in mentation in these patients. Other typical signs include tachycardia, tachypnea, and petechial rash. The fact that this pt is s/p cemented bipolar hemi for a femoral neck fx is the key part of the question stem which aids in answering the question. The above referenced article does not talk about fat embolization syndrome. It compares the prevalence of fat embolization in 156 pateints undergoing primary THA. 56 patients had bilat THA compared with 100 unilateral THA. It also compared the prevalence of fat embolism between cemented (n=100) and cementless stems (n=106). The results showed no statistically significant difference in prevalence of fat embolism b/w bilat vs unilat, as well as no difference b/w cemented and cementless stems. (I dont know why this was the recommended reading for this question).

63. In patients with Pseudomonas aeruginosa septic arthritis of the sternoclavicular joint, what is the most likely risk factor? 1 2 3 4 5 Rheumatoid arthritis Ankylosing spondylitis IV drug abuse Subclavian central line placement Reiters syndrome

63. In patients with Pseudomonas aeruginosa septic arthritis of the sternoclavicular joint, what is the most likely risk factor? 1 2 3 4 5 Rheumatoid arthritis Ankylosing spondylitis IV drug abuse Subclavian central line placement Reiters syndrome

PREFERRED RESPONSE: 3 RECOMMENDED READINGS: Higginbotham TO, Kuhn JE: Atraumatic disorders of the sternoclavicular joint. J Am Acad Orthop Surg 2005;13:138-145. Goldin RH, Chow AW, Edwards JE Jr, et al: Sternoclavicular septic arthritis in heroin users. N Engl J Med 1973;289:616-618.

Isolated septic arthritis of the SC joint is uncommon and frequently associated with an underlying disease or other risk factors. These include RA, sepsis, infected subclavian lines, alcoholism, HIV, dialysis and IVDU. Prompt diagnosis and treatment are crucial because untreated infection may lead to life-threatening consequences (cutaneous, extrapleural or intrathoracic abscess which could involve the retrosternal vascular structures aorta, braciocephalic vein, common carotid artery). Plain xrays may reveal sclerotic, lytic or mixed lesions but are less sensitive than CT scan. MRI may be useful in identifying softtissue involvement and abscesses. Definitive diagnosis is achieved by aspiration or open biopsy.

Although common organisms such as Staph aureus and Strep species have been reported, pt with other risk factors my have other causative organisms. Pseudomonas aeruginosa has been associated with IVDU. Neisseria gonorrhea and fungal infections with Candida have been reported with HIV. Mycobacteria TB has been reported in pt in 3rd world countries aw well as immunocompromised pt.
Although RA and sublcalvian line placement are associated with increased risk of septic arthritis of the SC joint, pseudomonas should lead you to suspect IVDU.

64. Figures 20a and 20b show the radiographs of a 45-yearold man who sustained a distal one third tibial fracture 3 months ago. The patient should be told that he is at greatest risk for the development of which of the following conditions? 1 2 3 4 5 Medial knee arthritis Contralateral medial knee arthritis Ankle pain and stiffness Low back pain Ipsilateral hip pain

64. Figures 20a and 20b show the radiographs of a 45-year-old man who sustained a distal one third tibial fracture 3 months ago. The patient should be told that he is at greatest risk for the development of which of the following conditions? 1 2 3 4 5 Medial knee arthritis Contralateral medial knee arthritis Ankle pain and stiffness Low back pain Ipsilateral hip pain

PREFERRED RESPONSE: 3 RECOMMENDED READINGS: Puno RM, Vaughann JJ, Stetten ML, et al: Long-term effects of tibial angular malunion on the knee and ankle joints. J Orthop Trauma 1991;5:247-254. Milner SA, Davis TR, Muir KR, et al: Long-term outcome after tibial shaft fracture: Is malunion important? J Bone Joint Surg Am 2002;84:971-980.

The tibial shaft is 1 of the most commonly fracured long bones in the body. A relatively frequent complication is malunion.
The 1st recommended reading analyzes the long-term effects of tibial angular malunion on the kee and ankle joints. Previous cadaveric tibia studies have showed that angular deformities of the same magnitude showed significantly more alteration of the tibiotalar contact area if the angulation is at the distal third. It has also been previosuly shown that the degree of horizontal malalignment of the ankle joint was always greater to that of the knee joint at any given position in the tibia. This could account for the more common occurrence of serious joint stiffness at the ankle following tibia fractures. In their study of 27 pateints with 28 tibia fractures, only 8 knees (19%) had pain while 14 ankles (50%) had symptoms, of which 3 (11%) were rated as severe, 5 (18%) as mo) as mild. There were 7 knees (32%) with decreased ROM, of which 6 were mild and 1 was moderate. Whereas 19 ankles (68%) had decreased ROM: 6 were mild, 8 were moderate, 4 were severe, and 1 had almost no motion.

All the contralateral knees and ankles, except for the pt with bilat fractures, were free of symptoms and had full ROM.
Furthermore, regression analyses showed that the greater the ankle malalignment, the poorer the ankle score. Knee score did not show any correlation with the degree of knee malalignment. The authors attributed this to the fact that the malalignment values of the knee were rather low compared to the ankle. This paper proved that greater degrees of joint malalignment are associated with a higher incidence of ankle morbidity. (Answer choice 3).

The 2nd article retrospectively evaluated the long-term outcome after tibial shaft fracture with a mean duration of follow-up of 36 years in 164 patients at a mean age of 61 years old. Their findings revealed that the functional outcome was good in the majority of patients, although knee pain and radiographic signs of OA were common and frequently bilateral. The OA was more prevalent on the side of the fx. The authors noted that most of the observed OA was unrelated to malunion, malalignment or any other factor related to the fx. In comparison, there was a significant increase in clinical and radiographic evidence of OA in the ipsilateral ankle and a significant increase in clinical evidence OA in the ipsilateral subtalar joint. (Consistent with answer choice 3)

65. For which of the following skeletal tumors is radiation therapy routinely used for definitive local control?

1 2 3 4 5

Osteogenic sarcoma Ewings sarcoma Chondrosarcoma Pleomorphic sarcoma Pagets sarcoma

65. For which of the following skeletal tumors is radiation therapy routinely used for definitive local control?
1 2 3 4 5 Osteogenic sarcoma Ewings sarcoma Chondrosarcoma Pleomorphic sarcoma Pagets sarcoma

PREFERRED RESPONSE: 2 RECOMMENDED READINGS: Krasin MJ, Rodriguez-Galindo C, Billups CA, et al: Definitive irradiation in multidisciplinary management of localized Ewing sarcoma family of tumors in pediatric patients: Outcome and prognostic factors. Int J Radiat Oncol Biol Phys 2004;60:830-838. Menendez LR (ed): Orthopaedic Knowledge Update: Musculoskeletal Tumors. Rosemont, IL, American Academy of Orthopaedic Surgeons, 2002, pp 195-202.

The 1st article assesses the effect of radiation dose on local control of Ewings sarcoma in 79 pediatric patients. All patients in this study received chemo as well as definitive radiatiotherapy with either low-dose (<40 Gy or standard dose >40 Gy). Some patients underwent debulking surgical procedures before RT initiation.
This study concluded that patient age >14 at time of diagnosis and tumor size >8cm were adverse prognostic factors for local treatment failure. Furthermore, pt age >14 years was also associated with worse survival. Although the radiation dose alone did not predict for local treatment failure, the cumulative incidence of local failure at 10 years was 19% when tumors <8cm were tx with <40 Gy, whereas no patient tx with standard doses (>40 Gy) developed local recurrence (p=0.084). However, large tumors, regardless of the radiation dose used, were poorly controlled locally.

The 2nd recommended reading is the OKU chapter on Ewings. Briefly, it is the 2nd most common malignant bone tumor of late childhood and early adulthood. It constitutes 1% of all childhood tumors, more common in males and is rare in nonwhite populations.
Adjuvant chemotherapy has dramatically improved the disease outcome because of its ability to control micrometastatic disease. Despite the traditional tx of chemo and radiation, several recent reports have demonstrated superior survival in pt who received surgical resection of the primary tumor combined w/chemo. However, none of these studies are randomized comparisons, so the question of superiority of 1 modality or the other remains in question.

66. A 2-year-old boy has torticollis after falling down stairs 6 months ago. A lateral radiograph and CT scans are shown in Figures 21a and 21b. Management should consist of 1- observation only. 2- physical therapy to restore neck range of motion. 3- immobilization with a soft cervical collar until the spasm resolves. 4- cervical traction, followed by immobilization in a halo brace. 5- atlantoaxial arthrodesis.

66. A 2-year-old boy has torticollis after falling down stairs 6 months ago. A lateral radiograph and CT scans are shown in Figures 21a and 21b. Management should consist of 1 2 3 4 5 observation only. physical therapy to restore neck range of motion. immobilization with a soft cervical collar until the spasm resolves. cervical traction, followed by immobilization in a halo brace. atlantoaxial arthrodesis.

PREFERRED RESPONSE: 5 RECOMMENDED READINGS: Vaccaro AR (ed): Orthopaedic Knowledge Update 8. Rosemont, IL, American Academy of Orthopaedic Surgeons, 2005, pp 767-774. Loder RT: The cervical spine, in Morrissy RT, Weinstein SL (eds): Lovell and Winters Pediatric Orthopaedics, ed 5. Philadelphia, PA, Lippincott Williams & Wilkins, 2001, pp 799-840. Subach BR, McLaughlin MR, Albright AL, et al: Current management of pediatric atlantoaxial rotatory subluxation. Spine 1998;23:2174-2179.

This pt has atlantoaxial rotatory subluxation. This is a common cause of childhood torticollis and is most often caused by trauma or infection. The most common cause is an upper respiratory infection (Grisels syndrome), but subluxation can occur after a retropharyngeal abscess, tonsillectomy, pharyngoplasty or trauma. These pt usually report neck pain and headaches and hold their head tilted to 1 side, resisting any efforts to move the head. If the deformity becomes fixed, the pain subsides but the torticollis and decreased ROM persist. Optimal management entails early diagnosis with plain xrays and dynamic CT Scan. Dynamic CT Scans show an abnormal relation between C1 and C2, marked by a fixed rotation that failed to resolve with attempts to correct the torticollis. The fixed angle between C1 and C2 is considered diagnostic.

It is classified into 4 types: I: Unilat facet subluxation with an intact transverse ligament (most common, benign)
II: Unilat facet subluxation with 305mm of anterior displacement

III: Bilat anterior facet displacement >5mm


IV: Posterior displacement of the atlas Types III and IV are rare, but neuro involvement may be present or instantaneous death can occur.

Once diagnosed, most cases will resolve with conservative measures. Treatment depends on the duration of symptoms.
1 week or less: Soft collar, NSAIDS, exercise program If sx persist after this tx: Head halter traction, muscle relaxants, analgesics

If subluxation present >1 month: Halo traction


If deformity present >3 months or reduction cannot be maintained or if neuro deficits are present: Posterior fusion is recommended to relieve muscle spasms associated with malrotation and produce normal head appearance.

John Yu

67. Discarded Question

68. What is the most predictive factor associated with successful healing of a medial collateral ligament tear?

1- Proximal rupture 2- Midsubstance rupture 3- Distal rupture 4- Concomitant anterior cruciate ligament tear 5- Concomitant posterior cruciate ligament tear

PREFERRED RESPONSE: 1
MCL has an intrinsic ability to heal. Its composed of fibroblast-type cells that have healing potential. The location of MCL injury has also been found to affect healing potential. Proximal tears, which have a more pronounced blood supply, tend to heal rapidly and may lead to stiffness. Distal tears seem to heal more slowly and patients usually do not develop problems with range of motion.

RECOMMENDED READINGS: Garrick JG (ed): Orthopaedic Knowledge Update: Sports Medicine 3. Rosemont, IL, American Academy of Orthopaedic Surgeons, 2004, pp 183-197. Shelbourne KD, Carr DR: Combined anterior and posterior cruciate and medial collateral ligament injury: Nonsurgical and delayed surgical treatment. Instr Course Lect 2003;52:413-418.

69. An 18-year-old high school football player has bilateral shoulder pain associated with bench and military press activities. He does not recall any acute injuries but reports that since age 10 years he has actively caused his shoulders to pop out of joint. Examination bilaterally shows posterior instability with jerk testing, 1 cm of inferior sulcus testing, and 1+ anterior instability with load and shift testing. Anterior apprehension and relocation tests are negative. An MRI arthrogram shows no discrete labral or capsular injury, and the glenoid is intact. Management should consist of

12345-

open posterior capsular shift. arthroscopic posterior capsular plication. rotator cuff interval closure. thermal capsulorrhaphy. physical therapy

PREFERRED RESPONSE: 5
Repetitive microtrauma as seen in swimmers, gymnasts, volleyball players, and bench pressing can lead to multidirectional instability by selectively stretching the shoulder capsule and ligaments. The symptoms develop insidiously, aggravated by repetitive minor injury when the shoulder is in the provocative position of flexion, adduction, and internal rotation. Operative treatment of posterior instability has a poor reputation. Consequently, an initial trial of physical therapy is usually recommended. Nonoperative treatment is successful in majority of athletes with multidirectional instability. Burkhead and Rockwood showed that a specific muscle strengthening program of the deltoid, RTC, and scapular stabilizers led to satisfactory results in 83% of patients with atraumatic instability. Athletes who fail nonoperative management are candidates for surgical intervention.

RECOMMENDED READINGS: Robinson CM, Aderinto J: Recurrent posterior shoulder instability. J Bone Joint Surg Am 2005;87:883-892. Levine WN, Prickett WD, Prymka M, et al: Treatment of the athlete with multidirectional shoulder instability. Orthop Clin North Am 2001;32:475-484.

70. A 25-year-old man has right knee pain after being involved in a motor vehicle accident. Radiographs reveal a distal femoral fracture with metaphyseal comminution. What is the most common missed injury diagnosed by CT?
12345Coronal fracture of the medial femoral condyle Coronal fracture of the lateral femoral condyle Sagittal fracture of the medial femoral condyle Sagittal fracture of the lateral femoral condyle Anterior cruciate ligament tear

PREFERRED RESPONSE: 2
Isolated coronal plane fractures of the distal femoral condyles (Hoffa fxs) occur uncommonly and are difficult to dx. Nork et al. reported that 38% of the supracondylarintercondylar distal femoral fxs were associated with coronal plane fxs. They also reported that 85% of the coronal fxs involving a single condyle were located laterally.

RECOMMENDED READINGS: Ostermann PA, Neumann K, Ekkernkamp A, et al: Long term results of unicondylar fractures of the femur. J Orthop Trauma 1994;8:142-146. Nork SE, Segina DN, Aflatoon K, et al: The association between supracondylar-intercondylar distal femoral fractures and coronal plane fractures. J Bone Joint Surg Am 2005;87:564-569.

71. Establishment of blood flow after replantation may lead to reperfusion injury, possibly the result of ischemia-induced hypoxanthine conversion to xanthine. What is the best adjunctive therapy agent to decrease xanthine production?

12345-

Heparin Dextran Allopurinol Aspirin Tissue plasminogen activator

PREFERRED RESPONSE: 3
Reperfusion injury refers to damage to tissue caused when blood supply returns to the tissue after a period of ischemia. The absence of oxygen and nutrients from blood creates a condition in which the restoration of circulation results in inflammation and oxidative damage through the induction of oxidative stress rather than restoration of normal function. In prolonged ischemia, hypoxanthine is formed as breakdown product of ATP metabolism. The enzyme xanthine dehydrogenase is converted to xanthine oxidase as a result of the higher availability of oxygen. This oxidation results in molecular oxygen being converted into highly reactive superoxide and hydroxyl radicals. These radicals attack cell membrane lipids, proteins, and glycosaminoglycans, causing further damage. Allopurinol is a xanthine oxidase inhibitor and can attenuate reperfusion injury in skeletal muscle by decreasing the amount of the reactive oxygen metabolite and lessening microvascular permeability.

RECOMMENDED READINGS: Trumble TE (ed): Hand Surgery Update 3: Hand, Elbow, & Shoulder. Rosemont, IL, American Society for Surgery of the Hand, 2003, pp 469-477. Waikakul S, Unnanantana A, Vanadurongwan V: The role of allopurinol in digital replantation. J Hand Surg Br 1999;24:325-327.

72. Which of the following adverse outcomes may develop following the use of bisphosphonate medications to treat children who have osteogenesis imperfecta?

1- Renal lithiasis 2- Radiographic bone changes suggestive of osteopetrosis 3- Osteonecrosis suggestive of LeggCalve-Perthes disease 4- Slipped capital femoral epiphysis 5- Hypercalcemia

PREFERRED RESPONSE: 2
Bisphosphonates are synthetic analogues of inorganic pyrophosphate. They are deposited on the surface of bone and ingested by osteoclasts during bone turnover. Its effect is anti-resorptive by inhibiting the mevalonate pathway in osteoclasts which enhances apoptosis and inhibit skeletal resorption. In adults, BPs are largely used to tx osteoporosis, pagets disease, met br. Ca., and hyperparathyroidism. In children, BPs are used through off-label applications in conditions like osteogenesis imperfecta, fibrous dysplasia, juvenile osteoporosis, Gauchers disease, and glucocorticoid induced osteoporosis. BPs have been reported to improve clinical outcomes and augment bone mass in conditions listed above. However, BPs have a prolonged half-life in bone, and their safety and efficacy have not been established in controlled trials in children. Decreased action of osteoclasts casue osteopetrosis, which is characterized by dense, poorly formed, and brittle skeletal tissue. Whyte et al. describes a case report of bisphosphonate-induced osteopetrosis in a 12 year old male. Prolonged treatment with high doses of BPs resulted in findings characteristic of osteopetrosis.

RECOMMENDED READINGS: Whyte MP, Wenkert D, Clements KL, et al: Bisphosphonate-induced osteopetrosis. N Engl J Med 2003;349:457-463. Falk MJ, Heeger S, Lynch KA, et al: Intravenous bisphosphonate therapy in children with osteogenesis imperfecta. Pediatrics 2003;111:573-578. Marini JC: Do bisphosphonates make childrens bones better or brittle? N Engl J Med 2003;349:423-426. Zeitlin L, Fassier F, Glorieux FH: Modern approach to children with osteogenesis imperfecta. J Pediatr Orthop B 2003;12:77-87.

Joshua Landa

73. A 21-year-old man reports swelling and pain in the left wrist. He denies any history of trauma or chills. Figures 22a through 22f show a radiograph, CT scan, T1- and T2-weighted MRI scans, and a biopsy specimen. What is the most likely diagnosis? 1- Telangiectatic osteosarcoma 2- Giant cell tumor 3- Synovial sarcoma 4- Chondroblastoma 5- Hemangioendothelioma

PREFERRED RESPONSE: 2
Giant cell tumor of the bone is a relatively uncommon tumor. It is characterized by the presence of multinucleated giant cells. The tumor is usually regarded as benign. In most patients, giant cell tumors have an indolent course, but tumors recur locally in as many as 50% of cases. Metastasis to the lungs may occur. Giant cell tumors typically occur in adults aged 20-40 years. Patients often complain of pain and swelling at the affected site. Pathologic fracture is present in 10% of patients. Vertebral giant cell tumors may extend into the spinal canal and compress the spinal cord, resulting in neurologic symptoms. Giant cell tumors are rarely multicentric. The plain radiographs and 3D CT show a lytic geographic lesion with cortical erosion. The location is typical for giant cell tumors (epiphysis of long bones). The histo has multinucleated giant cells. Remember that although lots of other lesions have giant cells (especially chondroblastoma, chondromyxoid fibroma, and giant cell osteosarcoma) in a giant cell tumor the giant cells look like the rest of the cells (the surrounding proliferating mononuclear cells). While important for the treatment of our patient, the recommended readings do not help with answering the question. The first JBJS article suggests that the risk of recurrence, which is high, has more to do with removing the tumor than adjuvant treatments (like cement). The second is a report of 24 patients with various tumors (20 with giant cells) of the distal radius that had reconstruction with an osteoarticular allograft. They had a low rate of recurrence of the tumor, a moderately high rate of revision, little pain in association with common activities, good function, and a moderate range of motion.

RECOMMENDED READINGS: Blackley HR, Wunder JS, Davis AM, et al: Treatment of giant-cell tumors of long bones with curettage and bone-grafting. J Bone Joint Surg Am 1999;81:811-820. Kocher MS, Gebhardt MC, Mankin HJ: Reconstruction of the distal aspect of the radius with use of an osteoarticular allograft after excision of a skeletal tumor. J Bone Joint Surg Am 1998;80:407-419.

74. A 20-year-old football player reports aching in his leg following vigorous exercise. He denies any recent trauma but does report that he sustained an untreated severe ankle sprain at age 13 years. Radiographs are shown in Figures 23a and 23b. What is the most likely diagnosis? 1- Osteochondroma 2- Parosteal osteosarcoma 3- Bizarre parosteal osteochondromatous proliferation 4- Enchondroma 5- Tibiofibular synostosis

PREFERRED RESPONSE: 5

The point of this question is to look at this XR and identify it as a tibiofibular synostosis as opposed to a tumor. You can tell by the history (he has a history of prior trauma, he is 20 years old, and it hurts more with activity) and by the appearance of the XR (the lesion is a bony connection between the tibia and fibula = synostosis). The XR is otherwise unremarkable. Also, the other responses are pretty far off an osteochondroma is generally a metaphyseal lesion, and enchondromas are intramedullary. The references are not helpful in anwering the question. In case you were curious, #3 (abreviated BPOP) is a rare subgroup of the osteochondromatous lesions that is treated with simple excision.

RECOMMENDED READINGS: Pell RF 4th, Khanuja HS, Cooley GR: Leg pain in the running athlete. J Am Acad Orthop Surg 2004;12:396-404. Henry JH, Anderson AJ, Cothren CC: Tibiofibular synostosis in professional basketball players. Am J Sports Med 1993;21:619-622.

75. In Charcot-Marie-Tooth disease, what foot deformity develops first? 1- Hindfoot varus 2- Hindfoot equinus 3- Hindfoot valgus 4- Plantar flexion of the first ray 5- Dorsal bunion

PREFERRED RESPONSE: 4

CMT is the most commonly inherited peripheral neuropathy, and can be due to an inherited defect in a myelin protein. It is also called peroneal muscular atrophy as the peroneal nerve is affected early. The tibialis anterior (and peroneus brevis) are affected with relative sparing of the peroneus longus, causing plantarflexion of the first ray. This imbalance also causes forefoot valgus (not a choice), which causes compensatory hindfoot varus. The coleman block test can be used to determine if the deformity is flexible.

RECOMMENDED READINGS: Coleman SS, Chesnut WJ: A simple test for hindfoot flexibility in the cavovarus foot. Clin Orthop 1977;123:60-62. Mosca VS: The cavus foot. J Pediatr Orthop 2001;21:423-424.

76. What method of medication monitoring has had the greatest impact on reducing medication errors? 1- Involvement of a pharmacist on rounds 2- Use of unit dosing 3- Computerized screening for allergies and interactions 4- Computerized physician order entry 5- Improved physician knowledge about drugs

PREFERRED RESPONSE: 4
In the first reference below, a total of 1111 prescribing errors were identified (62.4 errors per 1000 medication orders), most occurring on admission (64%). Of these, 30.8% were rated clinically significant and were most frequently related to antiinfective medication orders, incorrect dose, and medication knowledge deficiency. Of all verified prescribing errors, 64.4% were rated as likely to be prevented with computerized prescriber order entry (CPOE) (including 43% of the potentially harmful errors). If you got this wrong, it serves you right for not reading the archives of internal medicine like the rest of us.

RECOMMENDED READINGS: Bobb A, Gleason K, Husch M, et al: The epidemiology of prescribing errors: The potential impact of computerized prescriber order entry. Arch Intern Med 2004;164:785-792. Upperman JS, Staley P, Friend K, et al: The impact of hospital wide computerized physician order entry on medical errors in a pediatric hospital. J Pediatr Surg 2005;40:57-59.

77. Femoral nailing through the piriformis fossa is contraindicated in adolescents with open physes because of the risk of 1- osteonecrosis. 2- rotational malalignment. 3- shortening. 4- coxa vara. 5- increased intramedullary pressure.

PREFERRED RESPONSE: 1 Femoral shaft fractures in young children (<5) are usually treated in a spica. Skeletally mature teenagers get rigid IM fixation. The school-age child can benefit from several options including traction followed by casting, external fixation, flexible intramedullary nails, and plate fixation. Putting a nail through the piriformis fossa in a kiddo with an open physis has been associated with osteonecrosis (3-4%) since it interrupts the blood supply to the head. The problem appears to be related to the large diameter of the nail and its entry point in the relatively small femoral neck basis, close to the vessels supplying the femoral head. Going through the tip of the greater trochanter has been tried but occasionally causes osteonecrosis as well, and is probably not a good idea either.

RECOMMENDED READINGS: Flynn JM, Schwend RM: Management of pediatric femoral shaft fractures. J Am Acad Orthop Surg 2004;12:347-359. Sponseller PD: Surgical management of pediatric femoral fractures. Instr Course Lect 2002;51:361-365.

78. A 14-year-old girl reports right hip discomfort with activities. An AP radiograph is shown in Figure 24a, and coronal T1weighted and axial T2-weighted MRI scans are shown in Figures 24b and 24c, respectively. A biopsy specimen is shown in Figure 24d. What is the treatment of choice for this tumor? 1- Surgery only 2- Surgery and chemotherapy 3- Surgery and radiation therapy 4- Chemotherapy only 5- Chemotherapy and radiation therapy

PREFERRED RESPONSE: 2 The age and the fact that all of the answers involve doing something aggressive should key you in to something bad. Most common would probably be Ewings and Osteosarcoma. The radiology show a tumor in the right pelvis surrounding the ilium and the histo shows aggressive cells laying down osteoid. Treatment for osteosarcoma with surgery and chemo has extended life expectancies to over 70% at 5 years. RECOMMENDED READING: Menendez LR (ed): Orthopaedic Knowledge Update: Musculoskeletal Tumors. Rosemont, IL, American Academy of Orthopaedic Surgeons, 2002, pp 175-186.

Anjali Murthy

79. A 25-year-old man reports progressive wrist pain and swelling. An AP radiograph and biopsy specimen are shown in Figures 25a and 25b. Management should consist of 1- below-elbow amputation. 2- bisphosphonate treatment. 3- nonsteroidal anti-inflammatory drugs. 4- chemotherapy followed by wide surgical excision. 5- intralesional resection and reconstruction.

PREFERRED RESPONSE: 5
Radiology and histo are consistent with a giant cell tumor. The age is right (20s-40s) and there are no cystic areas visible on the slide to lead us to consider an ABC. Campanaccis staging system for giant cell tumors (based on plain radiographs): Stage I: lesions which do not distort or perforate the cortex Stage II: lesions that distort or expand the cortex but do not extend into the surrounding soft tissues Stage III: lesions that perforate the cortex and extend into surrounding soft tissues This is a Stage II lesion, and treatment for Stage I and II lesions is intralesional resection and reconstruction (often curettage and cementation or bone grafting). Recurrence is determined by adequate mechanical removal of tumor tissue, and the benefit of using cement is that it allows for good contrast with surrounding bone, facilitating detection of local recurrence. Cement also provides immediate structural stability in periarticular regions. Of note, Stage III lesions are sometimes treated with wide en bloc excision rather than an intralesional procedure to reduce the likelihood of local recurrence.

RECOMMENDED READING:
Menendez LR (ed): Orthopaedic Knowledge Update: Musculoskeletal Tumors. Rosemont, IL, American Academy of Orthopaedic Surgeons, 2002, pp 305-312. Athanasian EA: Aneurysmal bone cyst and giant cell tumor of bone of the hand and distal radius. Hand Clin 2004;20:269-281.

80. A 4-year-old boy has a fever and has been unable to ambulate for the past 48 hours. Examination reveals tenderness to palpation over the distal aspect of the right femur and no appreciable joint effusion. Laboratory studies show a WBC count of 13,200/mm3 (normal 3,500 to 10,500/mm3) and an erythrocyte sedimentation rate of 54 mm/h (normal up to 20 mm/h). Which of the following radiographic findings is most likely to be present?
12345Well-circumscribed lytic lesion Periosteal elevation Sequestrum Deep soft-tissue swelling Cortical erosions

PREFERRED RESPONSE: 4
Fever, tenderness to palpation over the distal femur, unable to ambulate for 2 days, NO joint effusion, elevated WBC, elevated ESR. They are describing the presentation for an acute infectious process, specifically ACUTE hematogenous osteomyelitis. Both references cited are very clear to state that radiographs do not show osseous changes for 7-10 days, but deep soft tissue swelling is a key finding early in the process. This swelling is better seen with CT scans than plain radiographs. Radiology for a patient with subacute hematogenous osteomyelitis may show bony changes at time of presentation that include: well-circumscribed lytic lesion (choice 1), a sequestrum (choice 2) and/or cortical erosions (choice 5). Clinically, these children tend to be less symptomatic with less pain, often no fever, and generally normal lab values. Periosteal elevation points more to a tumor process rather than an infectious one, although it can be seen in later stages of infection.

RECOMMENDED READING:
McCarthy JJ, Dormans JP, Kozin SH, et al: Musculoskeletal infections in children: Basic treatment principles and recent advancements. J Bone Joint Surg Am 2004;86:850-863. Morrissy RT: Bone and joint sepsis, in Morrissy RT, Weinstein SL (eds): Lovell and Winters Pediatric Orthopaedics, ed 5. Philadelphia, PA, Lippincott Williams & Wilkins, 2001, pp 459-505.

81. A terrible triad elbow injury that includes a comminuted nonreconstructable radial head fracture and a type III coronoid fracture associated with an elbow dislocation is best treated with acute radial head 1- resection and lateral collateral ligament repair. 2- resection, coronoid open reduction and internal fixation, and medial collateral ligament repair. 3- arthroplasty alone. 4- arthroplasty and coronoid open reduction and internal fixation. 5- arthroplasty, coronoid open reduction and internal fixation, and lateral collateral ligament repair.

PREFERRED RESPONSE: 5
Terrible triad injuries have a reputation for complications and poor clinical outcome due to problems attaining stability. Ring described unsatisfactory results in seven of eleven patients with this injury. In his study, all four patients with satisfactory results had retained the radial head and two had undergone repair of the lateral collateral ligament. Of note, none of the coronoid fractures were repaired. His recommendations were to restore elbow stability by providing radiocapitellar contact (fixing the radial head or replacing it), repairing the lateral collateral ligament and performing internal fixation of the coronoid fracture. McKee retrospectively reviewed results of this protocol for 36 consecutive patients and found that 28 patients had good or excellent results measured radiographically and clinically. Additionally, in his study the medial collateral ligament was repaired or a hinged ex-fix was placed for those patients who demonstrated residual instability. Of note, this is the order that the injuries were addressed: 1) coronoid, 2) radial head, 3) lateral ligamentous complex. Choice 5 is the only complete answer.
RECOMMENDED READING:
Ring D, Jupiter JB, Zilberfarb J: Posterior dislocation of the elbow with fractures of the radial head and coronoid. J Bone Joint Surg Am 2002;84:547-551. McKee MD, Pugh DM, Wild LM, et al: Standard surgical protocol to treat elbow dislocations with radial head and coronoid fractures: Surgical technique. J Bone Joint Surg Am 2005;87:22-32.

82. An otherwise healthy 38-year-old woman has numbness over the medial border of the left forearm and hand following a low-impact motor vehicle accident 4 weeks ago. Examination reveals difficulty with fine motor function in the left hand and weakness in long flexor function in all her digits and thumb. What is the most likely diagnosis?

1- C8 radiculopathy 2- Incomplete spinal cord injury 3- Secondary gain because the findings do not fit a consistent anatomic pattern 4- Ulnar nerve palsy at the elbow 5- Ulnar nerve palsy in Guyons canal

PREFERRED RESPONSE: 1
According to the reading, C8 radiculopathy often presents with numbness extending down the medial aspect of the arm and forearm and into the medial border of the hand and the ulnar 2 digits. Patients tend to have difficulty using the hands for routine daily activities. In contrast to ulnar nerve palsies (choices 4 and 5), the function of the FDP in the index and long fingers and of the FPL may be compromised in C8 radiculopathy. These muscles are not affected with ulnar nerve problems. Choices 2 and 3 are out because the scenario fits a very consistent anatomic pattern involving only C8.
RECOMMENDED READING:
Rao R: Neck pain, cervical radiculopathy, and cervical myelopathy: Pathophysiology, natural history, and clinical evaluation. J Bone Joint Surg Am 2002;84:1872-1881.

83. Examination of a 24-year-old man with hereditary motor sensory neuropathy reveals a cavovarus hindfoot correctable with Coleman block testing. Treatment should consist of 1- plantar fascial release, dorsal closing wedge osteotomy of the first metatarsal, and peroneus longus to peroneus brevis tendon transfer. 2- split anterior tibial tendon transfer. 3- first metatarsophalangeal arthrodesis and Achilles tendon lengthening. 4- Dwyer calcaneal osteotomy and posterior tibial tendon transfer. 5- triple arthrodesis.

PREFERRED RESPONSE: 1
When considering surgery for patients with CMT disease, it is essential to determine whether the deformity is fixed or flexible. Triple arthrodesis (choice 5) and calcaneal osteotomies (choice 4) are reserved for rigid deformities. Our patients deformity is still correctable with Coleman block testing and therefore flexible. The cavus deformity is due to imbalances caused by a weak anterior tibialis muscle and a strong peroneus longus muscle. This muscle imbalance results in plantar flexion of the first ray and secondary shortening and contracture of the plantar fascia. Compensatory hindfoot varus is achieved when the foot, which functions as a tripod, supinates and brings the midfoot and hindfoot into varus as a compensatory response to the plantar flexed first ray striking the ground first. In light of this mechanism, it just doesnt make sense to further weaken the anterior tib (choice 2). According to the OKU reference, in patients with a flexible foot like our patient, the treatment indicated is release of the plantar fascia, dorsal closing wedge osteotomy of the first metatarsal and transfer of the peroneus longus to the peroneus brevis (choice 1). IP (not MTP) fusions and Achilles tendon lengthenings (choice 3) have been described to treat these foot deformities, but both authors emphasize the importance of addressing the primary muscle imbalance, especially in flexible deformities. Choice 1 is the only option that does this. RECOMMENDED READING:
Richardson EG (ed): Orthopaedic Knowledge Update: Foot and Ankle 3. Rosemont, IL, American Academy of Orthopaedic Surgeons, 2004, pp 135-143. Holmes JR, Hansen ST Jr: Foot and ankle manifestations of Charcot-Marie-Tooth disease. Foot Ankle 1993;14:476-486.

84. A 57-year-old man was diagnosed with localized prostate carcinoma 3 years ago, with negative margins and negative lymph nodes. He now reports a 3-week history of severe right hip pain that is worse with weight bearing. Radiographs are shown in Figures 26a and 26b. CT scans of the chest, abdomen, and pelvis, as well as a whole body scan, are negative for other lesions. What is the next most appropriate step in treatment?
12345Right hip hemiarthroplasty Locked intramedullary rod placement in the right femur Biopsy of the right femur Hip disarticulation Curettage, cementation, and plate fixation

PREFERRED RESPONSE: 3
We need a diagnosis. Enough said. The article is largely irrelevant in helping to answer the question, but does comment on the importance of knowing the pathologic diagnosis before recommending intervention. The author points out that placing an intramedullary device through a primary bone tumor contaminates soft tissues and the entire medullary canal which could place the limb at risk for limb salvage and may compromise survival of a patient who may otherwise have been curable. In metastatic lesions, life expectancy comes into play when determining treatment and varies widely depending on the primary. We need to know what the tumor is before planning a course of action.
RECOMMENDED READING:
Rougraff B: Indications for operative treatment. Orthop Clin North Am 2000;31:567-575.

Dan Alfonso

85. A patient sustained a distal fibular fracture that was treated nonsurgically. Current radiographs reveal an increased talocrural angle. What is the most likely long-term abnormality related to this malunion? 1- Lateral talar tilt 2- Medial talar tilt 3- Anterior talar subluxation 4- Posterior talar subluxation 5- Medial osteochondral defect

PREFERRED RESPONSE: 1
The talocrural angle is measured off the mortise view. A line is drawn parallel to the distal articular surface of the tibia and a second line is drawn connecting the two malleoli. The angle is normally 815 degrees. Another method to measure the talocrural angle is using the angle formed by a line perpendicular to the distal tibial articular surface and the intermalleolar line. The normal values quoted for this is second method are between 75 and 87 degrees. Whichever method is used, the angle shoud be within 2 - 5 degrees of opposite side. A difference of greater than 2 to 5 degrees indicates fibular shortening.

In the study by Phillips et al, 138 SER or PERs were randomized to different treatment groups. There were a number of aspects to the study, but one conclusion that the authors came to was that comparing talocrural angle between injured and non injured side was the only radiographic marker that was useful for predicting prognosis. As talocrural angle increases, the fibula shortens and the effective joint space of the tibiotalar joint increases. The talus usually slides over with the distal fractured fibula causing increased lateral talar tilt.

RECOMMENDED READINGS: Phillips WA, Schwartz HS, Keller CS, et al: A prospective, randomized study of the management of severe ankle fractures. J Bone Joint Surg Am 1985;67:67-78. Pettrone FA, Gail M, Pee D, et al: Quantitative criteria for prediction of the results after displaced fracture of the ankle. J Bone Joint Surg Am 1983;65:667-677.

86. What is the direct intracellular target for nitrogen-containing bisphosphonates? 1- Farnesyl diphosphate synthase 2- Glutathione reductase 3- Epoxidase 4- Parathyroid hormone-related protein 5mRNA reverse transcriptase

PREFERRED RESPONSE: 1
Bisphosphonates are currently the most important class of antiresorptive drugs used for the treatment of metabolic bone diseases. Examples of nitrogen-containing bisphosphonates include alendronate, ibandronate, and risedronate. These compounds inhibit bone resorption by mechanisms that can lead to osteoclast apoptosis. More specifically, nitrogen containing bisphosphonates inhibit posttranslational modification (prenylation) of proteins, including the GTPbinding protein Ras, with farnesyl or geranylgeranyl isoprenoid via farnesyl diphosphate synthase. Glutathione reductase is a free radical scavenger. Glutathionereductase activity is related to bone density measurements. In one study, all the subjects with reduced densitometric values showed reduced glutathione-reductase levels. In that same study a strict relationship between low activity of antioxidant systems and demineralization process of the bone was found. Although glutathione reductase may play a role in osteoporosis, it is not the direct intracellular target for nitrogen-containing bisphosphonates.

Parathyroid hormone-related protein (PTHrP) is actually a family of protein hormones produced by most if not all tissues in the body. A segment of PTHrP is closely related to parathyroid hormone, and hence its name, but PTHrP peptides have a much broader spectrum of effects. Parathyroid Hormone and some of the PTHrP peptides bind to the same receptor, but PTHrP peptides also bind to several other receptors. PTHrP was discovered as a protein secreted by certain tumors that caused hypercalcemia. It was soon shown that the uncontrolled secretion of PTHrP by many tumor cells induces hypercalcemia by stimulating resorption of calcium from bone and suppressing calcium loss in urine, similar to what is seen with hyperparathyroidism. However, it quickly became apparent that PTHrP had many activities not seen with parathyroid hormone. PTHrP has not been implicated in the mechanism of bisphosphonates. mRNA reverse transcriptase and epoxidase do not appear have a direct relationship with osteoporosis or bisphosphonates.

RECOMMENDED READINGS: Reszka AA, Rodan GA: Mechanism of action of bisphosphonates. Curr Osteoporos Rep 2003;2:45-52.

87. The inhibitory effect of quinolone-class antimicrobials on early fracture healing is most directly linked to toxic effects on which of the following structures? 12345Vascular endothelial cells Osteoblasts Osteoclasts Chondrocytes Macrophages

PREFERRED RESPONSE: 4
The toxic effect of fluoroquinolones on articular cartilage has been described in a number of papers. Cellular changes in articular cartilage, such as chondrocyte death and matrix degeneration, have also been reported following toxic doses of fluorinated quinolone antibiotics. Several laboratory investigations have demonstrated articular cartilage erosions, fissure formation, and cartilage fibrillation associated with fluoroquinolone toxicity. Use of fluoroquinolones in children is contraindicated because of these adverse effects on growing cartilage and bones. The authors Huddleston et al attempted to elucidate the mechanism which flouroguinolines may prevent fracture healing by using a rat model. What the authors found was that fracture calluses in the animals treated with ciprofloxacin showed abnormalities in cartilage morphology and endochondral bone formation and a significant decrease in the number of chondrocytes compared with the controls.

RECOMMENDED READINGS: Perry AC, Prpa B, Rouse MS, et al: Levofloxacin and trovafloxacin inhibition of experimental fracture healing. Clin Orthop 2003;414:95-100. Huddleston PM, Steckelberg JM, Hanssen AD, et al: Ciprofloxacin inhibition of experimental fracture healing. J Bone Joint Surg Am 2000;82:161-173.

88.

A 17-year-old male hockey player twists his knee and is now unable to passively extend it beyond 20 degrees. An MRI scan shows a displaced bucket-handle tear of the lateral meniscus, and arthroscopy confirms a displaced bucket-handle tear of the normally configured lateral meniscus with a 3-mm peripheral rim. What is the best course of action at this time? 1- Partial meniscectomy 2- Reduction of the meniscus and an inside-out repair with vertical mattress sutures 3- Reduction of the meniscus and an outside-in repair using mulberry knots 4- Reduction of the meniscus and repair with bioabsorbable arrows 5- Reduction of the meniscus and repair with nonabsorbable horizontal mattress sutures

PREFERRED RESPONSE: 2
In the 1980s, Arnoczky and Warren defined the limits of meniscal vascularity. The peripheral 1/3 is vascular and can be repaired, the central 2/3 is nourished by the synovial fluid. Since this meniscal tear is in the peripheral 3-mm rim it can be repaired. Meniscal repair can be performed through an arthrotomy or by various arthroscopic techniques, including the inside-out technique, the outside-in technique, and the all-inside technique. Excellent results have been obtained in the repair of peripheral vertical tears. So our choices are 2 through 5 reduction and some method of repair.

The second paper by Rankin et al. compared four methods of meniscal repair in a cow meniscus model. The four methds they compared were: a biodegradeable meniscal implant without sutures (Biofix meniscus arrow), a suture anchor device, a horizontal mattress suture and a vertical mattress suture. The study found that inside out repair with vertical mattress sutures was superior with respect to residual displacement after repair and overall strength of repair. In fact, the force required to generate 2 mm of tear displacement was over three times greater for the vertical sutures (143 N) than for the bioabsorbable arrows (43.6 N). The first paper listed in the references found similar findings, vertical sutures were stronger than horizontal sutures and absorbable arrows in both pullout strength and linear stiffness. Mulberry knots are an outside in technique that have been found to be inferior to inside out vertical mattress.

RECOMMENDED READINGS: Boenisch UW, Faber KJ, Ciarelli M, et al: Pull-out strength and stiffness of meniscal repair using absorbable arrows or Ti-Cron vertical and horizontal loop sutures. Am J Sports Med 1999;27:626-631. Rankin CC, Lintner DM, Noble PC, et al: A biomechanical analysis of meniscal repair techniques. Am J Sports Med 2002;30:492-497.

89. Compared with a standard incision for a total hip arthroplasty, a minimally invasive approach using a single incision that is less than 4 inches in length is likely to result in 1- a shorter length of stay in the hospital. 2- less postoperative pain. 3- cosmetic improvement only. 4- better Harris hip scores at 6 weeks. 5- fewer complications.

PREFERRED RESPONSE: 3
Ogonda et al out of Belfast Ireland looked at 219 primary unilateral total hip arthroplasties that were randomized to less than 10 cm incisions and greater than 16 cm incisions. All patients and staff were blinded to the size of the incision and the anesthetic, analgesic, and postoperative physiotherapy protocols were standardized. The surgery was completed by one surgeon. The two groups were matched for age, grade according to the system of the American Society of Anesthesiologists, and body mass index. No significant difference was detected with respect to postoperative hematocrit, blood transfusion requirements, pain scores, or analgesic use. There was no difference in early walking ability or length of hospital stay and no difference in component placement, cement-mantle quality, or functional outcome scores at six weeks. The patient variables significantly associated with a probability of early discharge independent of incision length were patient age and preoperative hemoglobin levels. Minimally invasive total hip arthroplasty performed through a single-incision posterior approach by a high-volume hip surgeon with extensive experience in less invasive approaches to the hip is safe and reproducible. However, it offers no significant benefit in the early postoperative period compared with a standard incision of 16 cm. The only difference is cosmetic.

RECOMMENDED READINGS: Ogonda L, Wilson R, Archbold P, et al: A minimal-incision technique in total hip arthroplasty does not improve early postoperative outcomes: A prospective, randomized, controlled trial. J Bone Joint Surg Am 2005;87:701-710.

90. Figures 27a and 27b show the radiographs of the right lower extremity of a 1-year-old girl. She has normal knee extension strength. The right foot is at the level of the left midtibia. Treatment should consist of 1- tibiofibular synostosis, Syme amputation, and prosthetic fitting. 2- clubfoot release, followed by bracing and application of a lift. 3- a Van Ness rotationplasty and prosthetic fitting. 4- surgical foot realignment, followed by staged tibial lengthenings. 5- knee disarticulation and prosthetic fitting.

Images

PREFERRED RESPONSE: 1
This patient has a congenital deficiency of the tibia at the level of the midtibia. These patients often have a spectrum of lower extremity deformity including knee and ankle instability and equinovarus foot deformity with or without the loss of medial rays. The classification scheme most often used for longitudinal deficiency of the tibia was developed by Kalamchi and Dawe. In this classification system, there are three types: type I represents complete absence of the tibia, type II deficiencies are associated with absence of the distal tibia and type III have a distal tibia hypoplasia with diastasis of the distal tibiofibular articulation. This 1 year old girl has a type II longitudinal deficiency of the tibia. In Orthopaedic Knowledge Update 8, the authors state that the treatment for type II deformities entails tibifibular synostosis to attain stability, and either amputation (Syme or Boyd) or foot salvage depending on local anatomy. Syme amputation includes ankle disarticulation, removal of malleoli, and anchoring of the heel pad to the weight bearing surface. The Boyd amputation preserves the calcaneus, and the calcaneus is fused to the tibia. This obviates the problem of migration of the heel pad, but a longer waiting period before prosthetic fitting is usually necessary, and the extra length of the residual limb can sometimes make prosthetic foot fitting difficult. According to OKU 8 the answer would be tibiofibular synostosis, Syme amputation and prosthetic fitting.

As for the other answers, this patients main problem is not clubfoot (given the obiquity of the foot films it is hard to determine what is going on with the foot.) The third answer, the van ness procedure is not mentioned as an option for longitudinal deficiency of the tibia. Tibial lengthenings require a multitude of surgeries with arduous rehabilitation, but is an option. Lastly, type I (and not type II) longitudinal deficiency of the tibia is usually treated with through the knee amputation.

RECOMMENDED READINGS: Kalamchi A, Dawe RV: Congenital deficiency of the tibia. J Bone Joint Surg Br 1985;67:581-584. Schoenecker PL, Capelli AM, Millar EA, et al: Congenital longitudinal deficiency of the tibia. J Bone Joint Surg Am 1989;71:278-287. Vaccaro AR (ed): Orthopaedic Knowledge Update 8. Rosemont, IL, American Academy of Orthopaedic Surgeons, 2005, pp 745-755.

Basil Alwattar

91.

Figures 28a and 28b show the clinical photographs of a 23-year-old man who sustained a gunshot wound to the left wrist from a handgun. The muzzle velocity is approximately 1,500 ft/sec. Dorsal and volar wounds measuring 3 mm in diameter are visible on the wrist but are not actively bleeding. The patients vascular examination is normal, and he has normal motor function and sensation in the median, ulnar, and radial nerve distributions. Treatment should consist of 1- irrigation, debridement in the operating room, and IV antibiotics for 7 days. 2- irrigation, debridement in the operating room, and IV antibiotics for 7 days, followed by oral antibiotics for 7 days. 3- irrigation, debridement, external fixation in the operating room, and IV antibiotics for 3 days. 4- local wound care in the emergency department and IV antibiotics for 7 days. 5- local wound care in the emergency department and oral antibiotics for 3 days.

PREFERRED RESPONSE: 5 RECOMMENDED READINGS: Geissler W, Teasedall RD, Tomasin JD, et al: Management of low velocity gunshot-induced fractures. J Orthop Trauma 1990;4:39-41. Dickey RL, Barnes BC, Kearns RJ, et al: Efficacy of antibiotics in low-velocity gunshot fractures. J Orthop Trauma 1989;3:6-10. Bartlett CS, Hecht DL, Hausman MR, et al: Ballistics and gunshot wounds: Effects on musculoskeletal tissues. J Am Acad Orthop Surg 2000;8:21-36.

A low velocity wound is associated with a muzzle velocity of between 1,000-2,000 fps (feet per second) which is generally what is seen with most handguns (actually most are even lower aroun 850 fps). The XRs demonstrate no distinct fracture that needs fixation and the patient is neurovascularly intact with no gross bleeding. Contrary to popular belief bullets are not sterilized on discharge. Other sources of contamination include the clothing and skin flora which may be tracked in. This being said most low velocity gunshot wounds may be safely treated nonoperatively with simple local wound care (superficial irrigation and sterile dressing- never close the wound). Give a tetanus booster. As for antibiotics the answer is not clear. The first source states that one IM dose of Abx is equivalent to 48 hours of IV Abx. The second study states that no Abx are necessary (equal infxn rates in 2 cohorts IV Abx vs. none) and the review paper states antibiotics are probably only required for grossly contaminated wounds. However, because contamination is not always apparent, most authors still recommend routine prophylaxis. One study in JBJS from 1996 (Knapp et al) showed that a 3 day course of PO antibiotics led to a 2 % infection rate in patients with minor fractures due to a GSW. From all this they gave the answer of 3 days PO antibiotics. Also the review paper is actually well written and very interesting- highly recommend it.

92. A 25-year-old professional boxer reports pain and persistent swelling over the metacarpophalangeal (MCP) joint of his middle finger. Radiographs, including Brewerton views, are normal. Nonsurgical management, consisting of a 3-month course of activity modification, extension splinting, and antiinflammatory drugs, has failed to provide relief. Management should now consist of 12345reassurance and continued nonsurgical care. debridement of the MCP joint. cortisone injection into the flexor sheath. curettage and bone grafting of the third metacarpal head. repair of the extensor hood.

PREFERRED RESPONSE: 5 RECOMMENDED READINGS: Hame SL, Melone CP Jr: Boxers knuckle: Traumatic disruption of the extensor hood. Hand Clin 2000;16:375-380. Araki S, Ohtani T, Tanaka T: Acute dislocation of the extensor digitorum communis tendon at the metacarpophalangeal joint. J Bone Joint Surg Am 1987;69:616-619.

The extensor hood of the finger MP joint comprises the stout, longitudinal central tendon and the smaller, transverse- sagittal bands of the extensor digitorum comminus. Injury to the extensor hood basically is a tear of the sagittal band which results in a subluxatable or dislocatable extensor tendon. This is often described as a snapping with flexion of the MP joint. This condition usually arises in Rheumatoids but can also be traumatic often seen in boxers. Acute physical findings also include swelling, dec ROM, extensor lag and TTP at the sagittal band. Non-operative treatment as stated in first reference consists of extension splinting for 3-4 weeks and has been reported to have varying degrees of success. The first source states that splinting should be done within 10 days of the injury otherwise scarring will prevent a satisfactory result. Surgical repair has excellent results and consists of repair of the sagital band and reduction and suturing of the central tendon.

93. A 65-year-old man sustained an L1 burst fracture after falling from a tree 1 month ago. He reports back pain, increasing difficulty in voiding, and diffuse numbness in both feet. Examination reveals weakness in the long toe flexors and decreased perianal sensation. MRI scans show conus medullaris compression from a retropulsed fracture fragment. What is the next most appropriate step in management? 12CT myelogram Anterior decompression with grafting, with or without instrumentation 3- High dose corticosteroids 4- Decompressive lumbar laminectomy without fusion 5- Posterior fusion from T12 to L2 with instrumentation

PREFERRED RESPONSE: 2 RECOMMENDED READINGS: Clohisy JC, Akbarnia BA, Bucholz RD, et al: Neurologic recovery associated with anterior decompression of spine fractures at the thoracolumbar junction (T12-L1). Spine 1992;17:S325-S330. Krengel WF III, Anderson PA, Henley MB: Early stabilization and decompression for incomplete paraplegia due to a thoracic-level spinal cord injury. Spine 1993;18:2080-2087.

The first source is the only one that really addresses the issues put forth. The Clohisy study looked at 20 patients treated with immediate or delayed anterior decompression for patients with incomplete neurologic deficit with fractures at the thoracolumbar junction. They found that immediate anterior decompression had better results but that an improvement was obtained from late decompression as well but neither group had complete improvement in bladder or bowel function.

95. In mesenchymal neoplasia, an antioncogene refers to what function in a gene?


12345Reverses an oncogene to a proto-oncogene Loss of gene function leads to a malignancy Antagonizes a tumor suppressor gene Facilitates cell proliferation Opposite function to p53

PREFERRED RESPONSE: 2 RECOMMENDED READING: Buckwalter JA, Einhorn TA, Simon SR (eds): Orthopaedic Basic Science: Biology and Biomechanics of the Musculoskeletal System, ed 2. Rosemont, IL, American Academy of Orthopaedic Surgeons, 2000, pp 19-76.

Since we dont have the above listed reference available here is the definition of an antioncogene- a gene thought to inhibit cancerous growth: a recessive gene that is thought to suppress cancers by limiting cell multiplication. As opposed to an oncogene which is- a cancer-causing gene: a gene that can cause a cell to become malignant. Oncogenes are thought to be derived from normal cellular counterparts that have been taken up by viruses and altered so they malfunction when returned to the cell. So the loss of an antioncogene would lead to loss of inhibition of cancerous growth and eventual malignancy.

96. A 12-year-old boy sustained a knee injury in a fall off his bike. The patient reports that his knee pain improved the day after injury. Radiographs shown in Figures 30a and 30b reveal a lesion in the distal femur. Management should consist of
12345open biopsy and intralesional excision. CT-guided needle biopsy and antimicrobial therapy. curettage and autogenous iliac crest bone grafting. anti-inflammatory drugs as needed. observation and follow-up.

PREFERRED RESPONSE: 5 RECOMMENDED READING: Dunham WK, Marcus NW, Enneking WF, et al: Developmental defects of the distal femoral metaphysis. J Bone Joint Surg Am 1980;62:801-806.

The posteromedial aspect of the femur is where the adductor magnus inserts. It is also the site of occurrence of a developmental cortical defect. It is almost always asymptomatic and is most often an incidental finding on XR after trivial trauma. It is thought to be due to multiple minor traumas or the avulsion of small bits of bone at the site of the adductor insertion. Histology can resemble an osteosarcoma but the differentiation can generally be made via XRs to avoid biopsy. It is most common in 10-15 year olds and occurs in 11.5% of males and 3.6% of females. XRs show a small radiolucent lesion with ill defined margins on AP and on lateral an area of cortical thickening with a more ragged central defect. It is seen best on an oblique film. Bone scan is usually cold. The source cited recommends biopsy if the bone scan demonstrates more than minimal uptake. The authors of the paper suggest that this is a related entitiy to fibrous cortical defects and NOFs although most likely represents a different maturity level of the same lesion. This is a developmental defect of the distal femur and the correct treatment is observation and follow up.

Keith Chan

97. A 7-year-old boy underwent closed reduction and pinning of a supracondylar humeral fracture 8 hours ago. He now requires an increasing amount of pain medication to control his symptoms. His forearm is in a splint. His fingers are warm and pink with intact sensation and a palpable radial pulse is present; however, he has difficulty fully extending his fingers actively and has increased pain when they are passively extended. Management should now include
1- elevation and ice with reassessment the next day. 2- an increased dosage of pain medication. 3- additional radiographs for suspected loss of reduction. 4- measurement of compartment pressures. 5- measurement of prothrombin and partial thromboplastin times.

The question describes a patient who has several of the signs and symptoms of compartment syndrome pallor, parasthesias, paralysis, pulselessness, pain out of proportion, pain with passive ROM. In this case, the patient still has a palpable pulse and has a warm hand, however he has two of the hallmarks of compartment syndrome increasing pain out of proportion and pain with passive ROM. These two symptoms alone should raise enough suspicion to suspect compartment syndrome and hence, the correct answer #4: measurement of compartment pressures. PREFERRED RESPONSE: 4

RECOMMENDED READINGS: Whitesides TE, Heckman MM: Acute compartment syndrome: Update on diagnosis and treatment. J Am Acad Orthop Surg 1996;4:209-218. Bae DS, Kadiyala RK, Waters PM: Acute compartment syndrome in children: Contemporary diagnosis, treatment, and outcome. J Pediatr Orthop 2001;21:680-688.

98. What is the most frequent cause of longterm physical morbidity in the abused child? 12345Intra-abdominal trauma Head injury Spinal fracture Long bone fracture Ophthalmologic injury

Head trauma is the most frequent cause of morbidity and mortality in abused children. Head injuries may result from direct blows, dropping, shaking, or throwing. Multiple skull fractures, bilateral fractures, skull-base fractures, fractures crossing suture lines, and depressed fractures occur more frequently in abuse than in accidental injury. The infant brain is particularly vulnerable to accelerationdeceleration injuries. Subdural hematomas and retinal hemorrhages may be present without skull fractures in the shaken baby. Physical abuse should be suspected in any child with unexplained altered mental status, subdural hematoma, retinal hemorrhage, or skull fractures. Long-term sequelae of neurologic injuries from child abuse include cognitive disabilities, developmental delays, seizure disorders, and motor disabilities. PREFERRED RESPONSE: 2 RECOMMENDED READINGS: Kocher MS, Kasser JR: Orthopaedic aspects of child abuse. J Am Acad Orthop Surg 2000;8:10-20. Akbarnia BA, Campbell RM: The role of the orthopaedic surgeon in child abuse, in Morrissy RT, Weinstein SL (eds): Lovell & Winters Pediatric Orthopaedics, ed 5. Philadelphia, PA, Lippincott Williams & Wilkins, 2000, pp 1423-1445.

99. A 39-year-old laborer reports pain over the medial compartment of his knee joint. History reveals that he underwent nonsurgical management of a torn posterior cruciate ligament 20 years ago. Radiographs are shown in Figures 31a and 31b. What surgical option is most compatible with his occupation?
1- Lateral closing wedge proximal tibial osteotomy 2- Medial opening wedge proximal tibial osteotomy 3- Unicompartmental knee arthroplasty 4- Total knee arthroplasty 5- Arthroscopic-assisted insertion of a unispacer into the medial compartment

From the history, we see this is a young patient with medial joint pain and a nonoperatively treated PCL tear. The radiographs show severe medial compartment narrowing and a relatively unaffected lateral compartment. We can exclude answers 3-5 immediately from consideration. That leaves the two osteotomies. Closing wedge osteotomies have a more predictable union rate which would be beneficial in this young laborer. However, reading the reference article, I think the authors of the question want us to be aware that increasing tibial slope would aid in this patients PCL-deficient knee as it resists posterior translation of the tibia. Thus, the correct answer is an anteromedial opening wedge HTO. PREFERRED RESPONSE: 2 RECOMMENDED READING: Giffin JR, Vogrin TM, Zantop T, et al: Effects of increasing tibial slope on the biomechanics of the knee. Am J Sports Med 2004;32:376-382.

100. A 35-year-old woman has foot pain after tripping down a flight of stairs 2 weeks ago. The midfoot is swollen and tender, and she walks with a limp. Figures 32a and 32b show non-weight-bearing and weightbearing radiographs. Treatment should consist of 12345closed reduction and a short leg cast. closed reduction and percutaneous pinning. open reduction and internal fixation. weight bearing as tolerated in a postoperative shoe. external fixation.

The radiographs show NWB and WB views of the painful foot. On the WB view, the Lisfranc ligament appears injured as the Lisfranc joint between the base of the first and second metatarsals is widened. According to the reference, Dr. Tejwani found that stable anatomical reduction through open reduction and internal fixation of fracture-dislocations of the Lisfranc joint leads to the best long-term outcomes as patients so treated have less arthritis as well as better AOFAS midfoot scores. PREFERRED RESPONSE: 3

RECOMMENDED READINGS: Kuo RS, Tejwani NC, DiGiovanni CW, et al: Outcome after open reduction and internal fixation of Lisfranc joint injuries. J Bone Joint Surg Am 2000;82:1609-1618. Thordarson DB: Fractures of the midfoot and forefoot, in Myerson MS (ed): Foot and Ankle Disorders. Philadelphia, PA, WB Saunders, 2002, pp 1265-1296.

101. A 20-year-old man with a traumatic transtibial amputation reports buckling of his knee in the heel strike phase. What is the most likely cause of his symptoms? 12345Socket extension Prosthetic foot too firm Prosthetic foot placed too forward Prosthetic foot plantar flexed Excessive prosthetic length

The Miller review text provides a succinct answer to most prosthetic questions. A prosthetic foot that is too firm will cause knee flexion/buckling and lateral rotation of the toes whereas a soft foot causes excessive knee extension. Other common problems: prosthetic foot placed too forward causes increased knee extension (patellar pain) but stable, plantar flexed prosthetic foot causes increased patellar pressure and drop off. Socket extension would likely cause pistoning of the prosthesis and excessive prosthetic length would result in circumduction gait or lateral trunk bending. PREFERRED RESPONSE: 2 RECOMMENDED READINGS: Transtibial amputation, in Bowker J, Michael J (eds): Atlas of Limb Prosthetics. St Louis, MO, Mosby-Year Book, 1992. Vaccaro AR (ed): Orthopaedic Knowledge Update 8. Rosemont, IL, American Academy of Orthopaedic Surgeons, 2005, pp 645-654.

102. A 42-year-old woman who was treated with cast immobilization for a nondisplaced distal radius fracture now reports the sudden inability to extend her thumb. What is the most likely cause of this problem? 12345Entrapment of the flexor pollicis longus tendon Entrapment of the extensor pollicis longus tendon Rupture of the extensor pollicis longus tendon Posterior interosseous nerve palsy C-7 disk herniation

Rupture of the extensor pollicis longus (EPL) tendon after a distal radius fracture is an uncommon event; the incidence is 3%,according to a review of treatment of 200 consecutive patients with Colles fractures. Diagnosis is based on persistent dorsal wrist pain and a positive retroflexion sign. Recommended treatments in the prerupture setting include a third dorsal compartment release with or without an extensor retinacular patch graft. Also recommended are a palmaris longus graft in the acute rupture setting and a transfer from the extensor indicis proprius to the EPL tendon in the subacute or chronic setting. Results of all treatments seem to be clinically satisfactory. PREFERRED RESPONSE: 3 RECOMMENDED READINGS: Flatt AE: The Care of the Arthritic Hand. St Louis, MO, Quality Medical Publishing, 1995, pp 209-211. Gelb RI: Tendon transfer for rupture of the EPL tendon. Hand Clin

Yee Doung

103. A 30-year-old recreational jogger reports gradually worsening plantar heel pain that is most pronounced with the first few steps after getting out of bed and when getting up after sitting all day at a desk. Examination reveals tenderness to palpation over the medial calcaneal tuberosity. What is the most likely diagnosis? 1- Calcaneal stress fracture 2- Entrapment of the first branch of the lateral plantar nerve 3- Plantar fasciitis 4- Posterior ankle impingement 5- Flexor hallucis longus tendinitis

PREFERRED RESPONSE: 3
This answer is the classic presentation for plantar fasciitis: pain worst with the first few steps in the morning or after sitting a long time and pain at the insertion of the plantar fascia the medial calcaneal tuberosity. Calcaneal stress fractures usually present with pain over the lateral calcaneal tuberosity (positive squeeze test).

RECOMMENDED READINGS: Garrick JG (ed): Orthopaedic Knowledge Update: Sports Medicine 3. Rosemont, IL, American Academy of Orthopaedic Surgeons, 2004, pp 249-261. DeLee JC, Drez D Jr, Miller MD: Heel pain, in DeLee JC, Drez D Jr, Miller MD (eds): Orthopaedic Sports Medicine, ed 2. Philadelphia, PA, WB Saunders, 2003, pp 2446-2474.

PREFERRED RESPONSE: 3
Entrapment of the first branch of the lateral plantar nerve (Baxters nerve) presents with activity related pain and tenderness over the site of nerve entrapment instead of the plantar fascia origin. The site is usually between the abductor hallucis fascia and quadratus plantae muscle. Posterior ankle impingement presents with posterior heel/ankle pain, worse with plantar flexion. FHL tendinitis has pain over the FHL tendon and has clicking/locking symptoms of the hallux.
RECOMMENDED READINGS: Garrick JG (ed): Orthopaedic Knowledge Update: Sports Medicine 3. Rosemont, IL, American Academy of Orthopaedic Surgeons, 2004, pp 249-261. DeLee JC, Drez D Jr, Miller MD: Heel pain, in DeLee JC, Drez D Jr, Miller MD (eds): Orthopaedic Sports Medicine, ed 2. Philadelphia, PA, WB Saunders, 2003, pp 2446-2474.

104. A 65-year-old man sustained a proximal humeral fracture 4 months ago to his nondominant arm. He was treated with closed reduction and percutaneous fixation as shown in Figures 33a and 33b. At 6 weeks, he started active-assisted range of motion, and subsequently reported disabling pain both with activity and at rest. Current radiographs are shown in Figures 33c and 33d. Management should now consist of
12345shoulder arthroplasty. conversion to an intramedullary implant. removal of hardware. open reduction and revision internal fixation. activity modification.

Figure 33a

Figure 33b

Figure 33c

Figure 33d

PREFERRED RESPONSE: 1
This patient has a proximal humerus (surgical neck) nonunion 4 months from the initial fracture. There is also significant bone resorption and bone loss. According to the 2 referenced articles, patients do well with a shoulder arthroplasty. Antuna et. al. had a case series of 27 patients who underwent a shoulder arthroplasty after nonunion of 2-part (64%) or 3-part with greater tuberosity (28%) fractures. They had significant pain relief and better ROM (FE 41 88, ER 22 38).

RECOMMENDED READINGS: Antuna SA, Sperling JW, Sanchez-Sotelo J, et al: Shoulder arthroplasty for proximal humeral nonunions. J Shoulder Elbow Surg 2002;11:114-121. Galatz LM, Iannotti JP: Management of surgical neck nonunions. Orthop Clin North Am 2000;31:51-61.

PREFERRED RESPONSE: 1
Galatz and Iannotti had a review article about this topic. In it: 1. Intramedullary implants can be used but theres a high incidence of impingement and continued shoulder pain. There remains a high rate of continued nonunion. 2. ORIF with bone graft, using a blade plate or T-plate, is a good option only in cases of good bone stock. 3. Hemiarthroplasty should be used in cases of bone loss or poor bone quality. 4. Nonoperative management or removal of hardware have bad results.
RECOMMENDED READINGS: Antuna SA, Sperling JW, Sanchez-Sotelo J, et al: Shoulder arthroplasty for proximal humeral nonunions. J Shoulder Elbow Surg 2002;11:114-121. Galatz LM, Iannotti JP: Management of surgical neck nonunions. Orthop Clin North Am 2000;31:51-61.

105. What is the mode of inheritance for the condition shown in Figures 34a and 34b?
12345Autosomal-dominant Autosomal-recessive X-linked dominant X-linked recessive Non-Mendelian

Figure 34a

Figure 34b

PREFERRED RESPONSE: 1
This patient has hereditary multiple exostoses (osteochondromatosis). This is an autosomal dominant disorder with full penetrance. The genes involved are EXT1, 2, and 3. Patients present with short stature, limb length discrepancies, genu valgum, assymetry of the pelvic girdle (Figure 34A), bowing of the radius w/ ulnar subluxation of the carpus (Figure 34B), subluxation of the radial head, and relative shortening of the metatarsals, metacarpals, and phalanges.

RECOMMENDED READINGS: Buckwalter JA, Einhorn TA, Simon SR (eds): Orthopaedic Basic Science: Biology and Biomechanics of the Musculoskeletal System, ed 2. Rosemont, IL, American Academy of Orthopaedic Surgeons, 2002, pp 111-131. Pierz KA, Stieber JR, Kusumi K, et al: Hereditary multiple exostoses: One centers experience and review of etiology. Clin Orthop 2002;401:49-59.

106. A 24-year-old man with grade III (greater than 50% slip) isthmic spondylolisthesis of L5 on S1 is scheduled for surgery. The procedure should include 1- posterior lumbar fusion. 2- L5 corpectomy. 3- laminectomy alone. 4- stand-alone anterior lumbar interbody fusion. 5- complete reduction of the deformity.

PREFERRED RESPONSE: 1
Grade 3 spondylolisthesis is considered a high grade spondylolisthesis. Of all the options presented in the ICL from 2003, all of them include a posterior spinal fusion. The rest of the article talks about whether or not to instrument them, do an anterior spinal fusion, and/or do a reduction. Complete reduction holds a risk of nerve damage, especially to the L5 root. Partial reduction is indicated in those with poor sagittal balance and large slip angles. The second article reiterates the first article: posterior spinal fusion is a key part of the operation and that partial reduction is recommended, not complete reduction.
RECOMMENDED READINGS: Lenke LG, Bridwell KH: Evaluation and surgical treatment of high-grade isthmic dysplastic spondylolisthesis. Instr Course Lect 2003;52:525-532. DeWald CJ, Vartabedian JE, Rodts MF, et al: Evaluation and management of highgrade spondylolisthesis in adults. Spine 2005;30:S49-S59.

107. An obese 72-year-old woman with isolated medial knee osteoarthritis and 100 degrees of passive motion is considering undergoing unicompartmental knee arthroplasty. Which of the following is considered the greatest advantage of unicompartmental knee arthroplasty compared with a total (tricompartmental) knee arthroplasty? 12345Improved range of motion postoperatively Greater prosthetic longevity Greater relief of postoperative knee pain Faster early rehabilitation Better correction of preoperative deformity

PREFERRED RESPONSE: 4
The article quoted showed 1. 10 year survival at 91.4%, 12 year survival at 81.6% with women showing a worse survival rate than men (3.9% vs. 2.4%), 2. postop ROM to be equal or less than preop ROM, 3. only 1/291 blood transfusion requirement, 4. good postop knee society score: 93, or HSS score 84. Based on these results, by process of elimination, the only answer that makes sense is number 4: faster early rehab. However, that answer is not directly addressed by the reading. Go figure. But there is another article that says that UKAs have 2 day shorter hospital stays. (Robertsson et al. Acta Orthop Scand. 1999 Apr;70(2):170-5)

RECOMMENDED READING: Heck DA, Marmor L, Gibson A, et al: Unicompartmental knee arthroplasty: A multicenter investigation with long-term follow-up. Clin Orthop 1993;286:154-159.

108. According to the arithmetic method of determining limb-length discrepancy, what is the rate of growth per year of the distal femoral physis? 123454 mm 6 mm 10 mm 14 mm 18 mm

PREFERRED RESPONSE: 3
The growth rate mnemonic is 3, 9, 6, 3 for proximal femur, distal femur, proximal tibia, distal tibia. Officially in the readings, the growth of the distal femur per year is 10 mm, and the growth of the proximal tibia is 6 mm.

RECOMMENDED READINGS: Stanitski DF: Limb-length inequality: Assessment and treatment options. J Am Acad Orthop Surg 1999;7:143-153. Westh RN, Menelaus MB: A simple calculation for the timing of epiphysial arrest: A further report. J Bone Joint Surg Br 1981;63:117-119.

Phil Glassner

109.A 72-year-old man with a history of smoking 40 packs of cigarettes per year underwent successful left total hip arthroplasty 10 years ago. He now reports a 2-month history of progressive right hip pain. An AP pelvic radiograph and CT scan are shown in Figures 35a and 35b. What is the next most appropriate step in management? 12Immediate cemented right total hip arthroplasty Open reduction and internal fixation of the acetabular fracture 3- Activity modification, IV bisphosphonates, and follow-up in 6 weeks 4- Technetium Tc 99m scan and CT of the chest, abdomen, and pelvis 5- Radiation therapy

PREFERRED RESPONSE: 4 RECOMMENDED READINGS: Rougraff BT, Kneisl JS, Simon MA: Skeletal metastases of unknown origin: A prospective study of a diagnostic strategy. J Bone Joint Surg Am 1993;75;1276-1281. Menendez LR (ed): Orthopaedic Knowledge Update: Musculoskeletal Tumors. Rosemont, IL, American Academy of Orthopaedic Surgeons, 2002, pp 323-329. Explanation: Any 72 y/o M smoker w/ bony pain and a lesion on imaging has metastatic cancer until proven otherwise. Once you have this presentation, you need to try to find out what the lesion is before any intervention. The only choice that does this is answer 4. Further this is the preferred work-up for any bony lesion. The only thing they left out on initial work-up would be to get a chest xray, as well as plain films of any other area that lit up on bone scan or that was painful. The Rougraff article layed all this out in a prospective stucy of 40 pts who had skeletal mets of unknown origin. The stategy was H&P, routine labs (CBC, ESR, lytes, LFTs, alk phos and serum electrophoresis), plain films of chest and involved bone, tech 99 bone scan, and CT chest/abd/pelvis; followed by biopsy of most accesible osseous lesion. Percentage that these were diagnostic was as follows: H&P: 8%; labs: non-specific (although ESR >100 in MM); plain CXR: 43%; CT chest: additional 15% above CXR; CT abd/pelv: 13%; biopsy: added only 8% additional, further biopsy alone could not identify primary in 65% of patients. Overall 85% of primary tumros identified w/ above techinque. Other reference is a nice concise review, mostly the same info. Few points: Mean survival for pts presenting w/ metastatic CA of unknown origin is less than 12 months, and survival > 1yr is only 20%. Most common osseous mets: lung, breast, prostate, kidney, thyroid. Also stresses NOT to jump to biopsy w/o thorough work-up as it could be a primary lesion (ie sarcoma) and you could compromise surgical options, or a vasculat tumor (ie renal cell),

110.Which of the following is considered the most common problem that limits active overhead shoulder motion after hemiarthroplasty performed for a four-part proximal humeral fracture?
12345Retroversion of the prosthesis Varus alignment of the prosthesis Acromioclavicular arthritis Greater tuberosity nonunion Use of a cemented stem

PREFERRED RESPONSE: 4 RECOMMENDED READINGS: Frankle MA, Mighell MA: Techniques and principles of tuberosity fixation for proximal humeral fractures treated with hemiarthroplasty. J Shoulder Elbow Surg 2004;13:239-247. Frankle MA, Ondrovic LE, Markee BA, et al: Stability of tuberosity reattachment in proximal humeral hemiarthroplasty. J Shoulder Elbow Surg 2002;11:413-420. Bosch U, Skutek M, Fremerey RW, et al: Outcome after primary and secondary hemiarthroplasty in elderly patients with fractures of the proximal humerus. J Shoulder Elbow Surg 1998;7:479-484. Explanation: Amazingly only one of these articles really addresses this question directly, and it is in the introduction: tuberosity complications account for the majority of failures and poor outcomes. Doesnt specifically address overhead motion. And one even states that GT non-union was uncommon in their study. That being said, they do all stress the importance of proper positioning of the tuberosities in regards to obtaining optimal function. The first is a good review of techniques of tuberosity fixation and anatomy in regards to position of the greater tuberosity relative to the humeral head ( ie head-to-tuberosity distance being on avg 8mm) and the importance of proper lateral offset for cuff biomehanics. The 2nd is a bionmechanical study that basically says using a circumferential medial cerlage wire is when doing a hemi in 4 part fxs, increases stability and decreases interfrag motion and strain. The last article is an outcome study of elderly pts w/ 3 or 4 part prox humerus fxs; conclusion being that old people w/ these fxs have better outcomes when you do the surgery sooner than later ( ie < 4weeks, esp in regards to forward flexion).

111.A 28-year-old woman who previously underwent excision of the lateral (fibular) sesamoid for a painful intractable plantar keratosis now has a painful hypertrophic callus under the medial (tibial) sesamoid. Shoe modification and symptomatic treatment have failed to provide relief. She is now requesting excision of the remaining sesamoid. What is the most common surgical complication from this procedure?
12345Claw toe deformity Painful neuroma Painful scar Hallux varus Hallux valgus

PREFERRED RESPONSE: 1 RECOMMENDED READINGS: Richardson EG (ed): Orthopaedic Knowledge Update: Foot and Ankle 3. Rosemont, IL, American Academy of Orthopaedic Surgeons, 2004, pp 17-25. Richardson EG: Hallucal sesamoid pain: Causes and surgical treatment. J Am Acad Orthop Surg 1999;7:270-278. Explanation: The OKU chapter is a good review of disorders of the first ray, including hallux rigidus, turf toe, and sesamoid disorders. The Richardson article is only on sesamoid disorders, and is the same info with more detail, including surgical technique. Brief summary below. As far as the answer to the question, just one line saying avoid resection of both sesamoids to avoid claw toe deformity. The two sesamoids of the metatarsophalangeal joint are embedded in the tendons of the short flexor of the great toe. They are held together by the intersesamoid ligament and the plantar plate, which inserts on the base of the proximal phalanx of the hallux. The medial (tibial) sesamoid, which usually is larger than the lateral (fibular) sesamoid, rests in the medial facet sulcus) of the first metatarsal head and is more impacted by weight bearing than the lateral, which rests in the lateral facet. This anatomic arrangement leads to a higher incidence of traumatic injuries to the tibial sesamoid. Traumatic injuries to the sesamoids are easily recognized, but chronic inflammatory conditions, infections, and arthritis may be less obvious. Inflamed bursae, intractable plantar keratoses, or diffuse callus may indicate an underlying condition. Surgical treatment should not be considered until all conservative options have been exhausted, including orthotic management, shoe modifications, decreased weight bearing or avoidance of weight bearing, and cast immobilization. Excision of both sesamoids should be avoided because of the high postoperative incidence of hallux valgus or cock-up deformity (Claw toe) of the toe.

112. A 21-year-old man sustained a pivoting injury to his left knee in a flag football game 3 days ago. Examination reveals an effusion, and he is unable to reach terminal extension or flex past 45 degrees. MRI scans are shown in Figures 36a through 36c. Initial management should consist of
12345immobilization with a long leg splint. acute surgical reconstruction. preoperative physical therapy for mobilization. prophylactic bracing and a return to normal activity. arthrocentesis and rest.

PREFERRED RESPONSE: 3 RECOMMENDED READINGS: Shelbourne KD, Patel DV: Timing of surgery in anterior cruciate ligament-injured knees. Knee Surg Sports Traumatol Arthrosc 1995;3:148-156. Sterett WI, Hutton KS, Briggs KK, et al: Decreased range of motion following acute versus chronic anterior cruciate ligament reconstruction. Orthopedics 2003;26:151-154. Explanation: The patient in the question has an acute ACL tear. These studies primarily address the timing of surgery and what factors lead to the best outcomes. The Shelbourne article has a brief historical review of how ACL surgery has evolved, but is primarily a review of the authors experience with ACL recon from 1982-94, giving examples of how their pre/post-op protocol has changed. Stressing that pre-op ROM, including full extension, and quad strength is an important determinant of post-op outcome. The Sterett study is of 80 patients who underwent acute (<3 weeks from injury) recon vs chronic (>3 weeks) of their ACL tears to see if it mattered, as the thinking has been to wait >3 weeks fro a better outcome. All patients had to have pre-op ROM of 0-120 actively, be able to perform a straight leg raise and demonstrate good quadriceps control. With this criteria met, the patients then underwent there ACL recon acutely or chronically. They found no significant difference between the 2 groups at 1 year, and only one patient (who was in the chronic group) had decreased ROM at 1 yr ( decr if had >10 deg loss of extension or <120 deg flexion, or both). They concluded that pre-op ROM and strength were the primary factors for good outcome, and not the timing itself. Interestingly, no patient wa d/ced from the hospital until they had ROM of at least 5-90 post-op. This meant in house stays of 1-3 days. The sports rotation would be a lot less enjoyable if there were btwn 5-10 post-op ACLs to round on every day.

113.A 9-year-old girl sustains the fracture shown in Figure 37. What is the most likely complication following open reduction and internal fixation of this fracture?
12345Sciatic nerve palsy Nonunion Osteonecrosis Chondrolysis Coxa valga

PREFERRED RESPONSE: 3 RECOMMENDED READINGS: Blasier DR, Hughes LO: Fractures and traumatic dislocations of the hip in children, in Beaty JH, Kasser JR (eds): Rockwood and Wilkins Fractures in Children, ed 5. Philadelphia, PA, Lippincott Williams & Wilkins, 2001, pp 913-939. Hughes LO, Beaty JH: Fractures of the head and neck of the femur in children. J Bone Joint Surg Am 1994;76:283-292. Explanation: The patient in question has a Type 3 femoral neck fracture. The article and the chapter have the same info, except the chapter has more pictures. In summary, these are rare fxs, fewer than 1 % of all peds fxs; usually caused by high energy trauma. Classified By Delbet into 4 types. 1. transepiphyseal (a: non-dislocated, b. dislocated), 2. transcervical, 3. cervicotrochanteric, 4. intertroch. Type 2 are the most common (46%) followed by type3 at 34 %, then type 4, then 1. Displaced fxs have a higher risk of AVN, and the incidence is related to the initial trauma/displacement, and not to the type of treatment. AVN is the most serious and frequent complication, overall prevalence of 30%; the highest in type 1B, 2 and 3; very low in type 4.Due to disruption of blood supply; some thought that evacuation of hemarthrosis may decrease AVN rate by decreasing intracapsular pressure and improving blood flow. 3 types of AVN, 1. whole head involved, 2. partial head, 3. area of AVN from fracture to physis. The second most common complication is coxa vara, and in some studies also 30% incidence. (not valga as in the answer choices). Usually Rx for the fracture in the question is anatomic reductin w/ internal fixation, and usually follwed by spica casting up to age 10.

114.Which of the following tumors has the greatest potential to metastasize to the lung?
12345Osteoblastoma Enostosis Desmoplastic fibroma Giant cell tumor Enchondroma

PREFERRED RESPONSE: 4 RECOMMENDED READINGS: Menendez LR (ed): Orthopaedic Knowledge Update: Musculoskeletal Tumors. Rosemont, IL, American Academy of Orthopaedic Surgeons, 2002, pp 113-118. Feigenberg SJ, Marcus RB Jr, Zlotecki RA, et al: Whole lung radiotherapy for giant cell tumors of bone with pulmonary metastases. Clin Orthop 2002;401:202-208. Explanation: Giant cell tumor of bone is one of 2 benign bone tumors that can metastasize to the lung, the other being chondroblastoma. The OKU chapter is a good concise review of GCT of bone. The high points: GCT of bone is a mesenchymal tumor; high incidence of p53 protein and c-myc oncogene in aggressive GCTs that metastasize to the lung; location in epiphysis of long bones in adults ages 20-40; more often in females; most common sites: 1. distal femur, 2. proximal tibia, 3. sacrum, 4. distal radius, also can be in vertebral body; early stages present w/ pain, can have swelling in the joint; grossly is a soft, friable tumor w/ hemorrhagic areas; micro has uniform field of polyhedral to short spindle cells with a benign nuclear pattern similar to the nuclei of the GCTs; xrays show lytic lesion; campanacci staging: 1. no evidence of cortical expansion, 2. cortical thinning w/o soft tissue involvement, 3. cortical destruction w/ soft tissue extension; stage 3 more likely to recur and thus metastasize; mngmnt by extended curettage and application of bone cement, some use adjuvants such as phenol, H2O2, or liquid nitrogen; radiation therapy should not be used unless difficult lesion in spine or pelvis due to 15% incidence of late conversion to high grade osteosarc, malignant histiocytoma, or malignant GCT; mets to lungs in 2% of all cases and 6% of recurrent cases; presence of matrix metalloproteinases facilitate the local and metastatic spread of the tumor; distal radius shown to be sight most common to metastasize to the lung; lung lesions usually treated by surgical resection with close to 80% disease-free outcome.

The 2nd article is basically a review of a whole bunch of case reports including very few people who had mets to the lungs and different treatment. In the authors series of 3 patients who received whole lung radiotherapy to treat the lung mets 2 of them did well and were long-term survivors and the other one died. They concluded that their regimen of radiation of 16Gy in 10 fractions delivered to the whole lung, followed by a boost of 35 to 45 Gy should be used in patients with pulmonary mets who are poor surgical candidates, refuse thoracic surgery, mets are technically unresectable, or who have disease recurrence after surgery or chemo If anyone wants copies of any of the articles or chapters referenced in Qs 109-114; I have them all digitally, and I can email them to you. I am Phil.

Reza Jazayeri

115. Which of the following is considered the most common disadvantage of using posterior antiglide plating in the treatment of lateral malleolar fractures compared to lateral neutralization plating?
123Intra-articular penetration of the distal screws Inability to stabilize the syndesmosis Peroneal irritation if the plate is placed too distally 4- Poor distal fixation 5- A biomechanically weaker construct

The posterolateral antiglide plating technique was introduced in 1981 by Brunner and Weber. This construct was found to be biomechanically stronger in a cadaveric study versus lateral plating in AO-type B lateral malleolar fractures, especially in weak bone as shown by Schaffer. In a retrospective study of 70 patients with this fracture, 30 (43%) of patients required plate removal for discomfort and signs of peroneal tendonitis. Intraoperative peroneal tendon lesions were indentified in 9 (30%) of patients. Adjunct cadaveric study of 10 ankles demonstrated that the osteosynovial part of the peroneal groove is approximately at the junction of the proximal and middle thirds of the lateral malleolus. However, contact of the distal end of the plate with the tendons did not cause tendon lesions, nor did a screw head if it was well sunk into the plate. Correlations were found to low plate placement in conjunction with protruding screw head in the most distal hole.

PREFERRED RESPONSE: 3 RECOMMENDED READINGS: Weber M, Krause F: Peroneal tendon lesions caused by antiglide plates used for fixation of lateral malleolar fractures: The effect of plate and screw position. Foot Ankle Int 2005;26:281-285. Schaffer JJ, Manoli A II: The antiglide plate for distal fibular fixation: A biomechanical comparison with fixation with a lateral plate. J Bone Joint Surg Am 1987;69:596-604.

116. Figure 38 shows the radiograph of a 2year-old boy with a unilateral bowing of the leg. What is the most likely diagnosis?
12345Toddlers fracture Focal fibrocartilaginous dysplasia Nonossifying fibroma Fibrous cortical defect Fibrous dysplasia

Focal fibrocartilaginous dysplasia (FFDP) is an uncommon, benign condition that has been known to cause deformity of the long bone in young children. In 1988 Bradish reported on five patients with tibia vara due to focal fibrocartilaginous dysplasia FFCD. All patients presented with unilateral bowing. Age of presentation varied from 9 to 28 months. Radiographs revealed a cortical defect in the metaphyseal region of the medial tibia with an area of surrounding sclerosis, a characteristic appearance seen in this case. Histological features range from purely dense fibrous tendon-like tissue to benign fibrocatilaginous tissue. In 3 patients spontaneous correction occurred by age 3, while in one of the remaining two treated by operation, a valgus deformity and neurological complication resulted. Bradish therefore recommended conservative treatment. Choi in 2000 reviewed all 46 cases of FFCDs in the literature. 38 occurred in the tibia, 5 in the femur, 2 in the ulna and 1 in the humerus. 43% of tibial and all ulnar lesion resolved spontaneously. In contrast, all femoral and humeral lesions showed an increase or persistence in deformity. Choi agrees with other authors that initial management showed be observation. However, when the deformity increases or persists despite a reasonable period of observation, excellent results can be obtained after corrective osteotomy with or without resection of lesion with little chance of recurrence.

PREFERRED RESPONSE: 2 RECOMMENDED READINGS: Bradish CF, Davies SJ, Malone M: Tibia vara due to focal fibrocartilaginous dysplasia: The natural history. J Bone Joint Surg Br 1988;70:106-108. Choi IH, Kim CJ, Cho TJ, et al: Focal fibrocartilaginous dysplasia of long bones: Report of eight additional cases and literature review. J Pediatr Orthop 2000;20:421-427.

117.What type of prescription footwear is covered per year under Medicares Therapeutic Shoe Bill for patients with diabetic neuropathy? 12-

34-

5-

One pair of extra-depth shoes and three pairs of inserts One pair of extra-depth shoes and four pairs of inserts One pair of custom-molded shoes but not inserts Two pairs of extra-depth shoes and two pairs of insoles Three pairs of inserts but not extra-depth shoes

As per the Therapeutic Shoe Bill each qualified patient is limited to one the following within one calendar year: -One pair of custom-molded shoes (includes one pair of inserts) and two additional pairs of inserts; or -one pair of depth shoes and three pairs of inserts

PREFERRED RESPONSE: 1 RECOMMENDED READINGS: Medicare Carriers Manual: Therapeutic shoes for individuals with diabetes. U.S. Department of Health and Human Services, July 1994, section 2134. Janisse DJ: The Therapeutic Shoe Bill: Medicare coverage for prescription footwear for diabetic patients. Foot Ankle Int 2005;26:42-45.

118. While performing a quadriceps-sparing minimally invasive total knee arthroplasty, the patellar tendon starts to peel off the tibial tubercle. Retraction is stopped before the integrity of the tendon is compromised. What is the best course of action? 1Insert transfixion pins into the tibial tubercle to protect the patellar tendon. 2- Repair the peeled portion of the tendon using suture anchors. 3- Augment the patellar tendon with a semitendinosus autograft. 4- Convert the quadriceps-sparing technique into a minimidvastus approach. 5- Convert the quadriceps-sparing technique into a standard medial parapatellar arthrotomy.

MIS TKA is in the early stages of development. There are many opponents who believe that the technique is nothing more than a cosmetic modification of the standard TKA that leads to more complications and less patient satisfaction. MIS surgery should not be based on the length of the incision or the cosmetic result. The term "minimally invasive" should refer to the extent of disruption of the anatomic structures about the involved joint. In the knee, the MIS approach should not violate the extensor mechanism and should not violate the suprapatellar pouch. MIS should be a capsular approach, and as such it should produce less discomfort and a faster recovery. One should convert to a standard medial parapatellar arthrotomy when inadequate exposure will compromise the case as may be seen with obese patients. Authors recommend using MIS in knees with at least 125 degrees of flexion, and malalingment limited to no more than 10 deg varus, 15 deg valgus, and 10 of flexion contracture. The MIS procedure should allow the patient to recover faster while keeping the incidence of complications at the same or lower levels as the open procedure.

PREFERRED RESPONSE: 5 RECOMMENDED READING: Tria AJ Jr: Minimally invasive total knee arthroplasty: The importance of instrumentation. Orthop Clin North Am 2004;35:227-234.

119.A 25-year-old man has had intermittent swelling in the left thenar eminence for the past few months. He describes the onset of a dull ache associated with the swelling. An AP radiograph is shown in Figure 39. What is the most likely diagnosis?

12345-

Lipoma Hemangioma Synovial sarcoma Ewings family of tumor Giant cell tumor of the tendon sheath

Benign vascular tumors are the second most common benign tumor after lipomas. Examples include hemangiomas, lymphangiomas and glomus tumors. The most common type of angiomas in the hemangioma, which can be a superficial cutaneous lesion or a deep intramuscular one as the case presented here. In most cases the, the lesion is solitary or localized. Radiographs frequently demonstrate phleboliths, which are well-circumscribed calcific densities in the tumor. Patients may complain of an aching or throbbing when the lesion is dependent. However, in some cases, patients may be asymptomatic until intralesional hemorrhage occurs, either spontaneously over after a minor trauma. Surgical resection may be necessary to improve limb function and to reduce pain. Recurrence is a possibility, so a wide margin is preferable. Because demarcating the border of the tumor may be difficult is surgery, limb exanguination should be avoided to better appreciate the tumor.

PREFERRED RESPONSE: 2 RECOMMENDED READINGS: Menendez LR (ed): Orthopaedic Knowledge Update: Musculoskeletal Tumors. Rosemont, IL, American Academy of Orthopaedic Surgeons, 2002, pp 243-253. Weiss SW, Goldblum JR: Benign tumors and tumor-like lesions of blood vessels, in Enzinger FM, Weiss SW (eds): Soft Tissue Tumors. St Louis, MO, Mosby, 2001, pp 837-890.

120. An 18-year-old woman has pain in the middle finger after jamming the finger. Radiographs are shown in Figures 40a and 40b. Treatment should consist of
12345dynamic traction. closed reduction and splinting. open reduction and internal fixation. acute volar plate arthroplasty. arthrodesis of the proximal interphalangeal joint.

Bicondylar phalangeal fractures are unstable fractures where late displacement can exceed the initial displacement in the unsecured fracture. With displaced bicondylar fractures as the case presented here, ORIF is the best method to ensure effective stabilization. Meticulous and minimal dissection, firm fixation, and early ROM may to some degree counteract the increased propensity for fibroplasias in response to surgery. Incision and approach is via the midaxial approach the interval between the central slip and lateral bands. Relatively large fragments can be treated with fluoroscopically assisted ORIF using mini condylar plate fixation.

PREFERRED RESPONSE: 3 RECOMMENDED READINGS: Freeland AE: Hand fractures: Repair, Reconstruction and Rehabilitation. Philadelphia, PA, Churchill Livingstone, 2000, pp 105-108. Dias JJ: Intra-articular injuries of the distal and proximal interphalangeal joints, in Berger RA, Weiss AP (eds): Hand Surgery. Philadelphia, PA, Lippincott Williams and Wilkins, 2004, pp 153-161.

George Kardashian

121. A 2-week-old girl has hematogenous septic arthritis of her right knee. There are no open lesions and no other sites of infection. The most common infecting organisms in this setting include Staphylococcus aureus and 12345group B Streptococcus and gram-negative bacilli. Kingella kingae and Haemophilus influenza B. Neisseria gonorrhoeae and Borrelia burgdorferi. Yersinia species and Haemophilus influenza B. Kingella kingae and Mycoplasma variants.

PREFERRED RESPONSE: 1

This never gets old, just memorize it already. In children, hematogenous bone and joint infections are most commonly caused by Staph aureus, just like in adults. However, neonates are also susceptible to infection by group B Streptococci and gram-negative bacilli due to their weak immune system. Children with sickle cell are also prone to infections with Salmonella.

RECOMMENDED READING: Vaccaro AR (ed): Orthopaedic Knowledge Update 8.

Rosemont, IL, American Academy of Orthopaedic Surgeons, 2005, pp 217-228. McCarthy JJ, Dormans JP, Kozin SH, et al: Musculoskeletal infections in children: Basic treatment principles and recent advancements. Inst Course Lect 2005;54:515-528. Sponseller PD (ed): Orthopaedic Knowledge Update: Pediatrics 2. Rosemont, IL, American Academy of Orthopaedic Surgeons, 2002, pp 27-41.

122. After repair of a torn quadriceps tendon, which of the following rehabilitation protocols provides range of motion and minimal stress on the repair? 1- Active open chain extension and active open chain flexion 2- Active open chain extension and active closed chain flexion 3- Active-assisted extension and active open chain flexion 4- Passive extension and active open chain flexion 5- Passive extension and active closed chain flexion

PREFERRED RESPONSE: 5

The articles talk about a complex interaction of joint's geometry, compressive and shear forces, weightbearing, and muscular forces, measured with EMGs and tested during closed and open chain flexion and extension exercises. In summary and relevance to this question, they concluded that the least amount of stress is placed on quads during passive extension and active closed chain flexion, that is moslty performed by hamstrings.

RECOMMENDED READING: Stuart MJ, Meglan DA, Lutz GE, et al: Comparison of
intersegmental tibiofemoral joint forces and muscle activity during various closed kinetic chain exercises. Am J Sports Med 1996;24:792-799. Wilk KE, Escamilla RF, Fleisig GS, et al: A comparison of tibiofemoral joint forces and electromyographic activity during open and closed kinetic chain exercises. Am J Sports Med 1996;24:518-527.

123. A 5-year-old girl has a history of frequent fractures, progressive hearing loss, and deteriorating eyesight. Radiographs are shown in Figures 41a and 41b. It has been determined that she is not a candidate for a bone marrow transplant. What is the pathophysiology of this disorder? 1- Parvovirus infection of bone 2- Failure of osteoclastic resorption of bone 3- Oversensitivity of osteoblasts to parathyroid hormone 4- Qualitative defect of type I collagen synthesis 5- Overproduction of calcitonin

PREFERRED RESPONSE: 2
This girl has symptoms of cranial foraminal narrowing causing progressive and gradual hearing loss and deteriorating eye sight, all consistent with osteopetrosis. On radiograph 41a, one can see increased bone density, inability to distinguish cortex from medullary canal (filled with bone), and endobones or bones within bones radiodense areas that resemble small bones inside the cortices of tubular bones. Figure 41-b shows uniformly radiodense vertebral bodies or rugger jersey appearance with endplates sclerosis and lucency in the middle. Osteopetrosis is also known as Albers-Schonberg disease. It is a rare hereditary disorder of osteoclast function in which resorption of bone is diminished. As a consequence, bone modeling and remodeling are impaired. The defect in bone turnover characteristically results in skeletal fragility despite increased bone mass, and it may also cause hematopoietic insufficiency, disturbed tooth eruption, nerve entrapment syndromes, and growth impairment. Human osteopetrosis is a heterogeneous disorder encompassing different molecular lesions and a range of clinical features. However, all forms share a single pathogenic nexus in the osteoclast
RECOMMENDED READING: Vaccaro AR (ed): Orthopaedic Knowledge Update 8. Rosemont, IL, American Academy of Orthopaedic Surgeons, 2005, pp 187-196. Herring JA: Skeletal dysplasias, in Herring JA (ed): Tachdjians Pediatric Orthopaedics, ed 3. Philadelphia, PA, WB Saunders, 2002, pp 1505-1583. Armstrong DG, Newfield JT, Gillespie R: Orthopedic management of osteopetrosis: Results of a survey and review of the literature. J Pediatr Orthop 1999;19:122-132.

124. In comparison to nonsurgical management, surgical decompression for degenerative lumbar stenosis results in
12345better functional outcomes at 2 years. worse functional outcomes at 2 years. equivalent functional outcomes at 2 years. a higher rate of cauda equina syndrome. more delayed relief of symptoms.

PREFERRED RESPONSE: 3
Two of the articles provided do not directly deal with the question asked and the other one indirectly contradicts the answer given. However, earlier studies with shorter follow up, as well as Dr. Kims lecture, report that these pts have equivalent functional outcomes in regards to back pain but have improvement in their leg pain at 2yrs.
Kornblum article: This article deals with spondylolisthesis as a cause of stenosis and thus doesnt directly pertain to the question asked, but the authors showed that in patients undergoing single-level decompression and posterolateral arthrodesis for spinal stenosis and concurrent spondylolisthesis, a solid fusion improved longterm clinical results. Benefits of a successful arthrodesis over pseudarthrosis were demonstrated with respect to back and lower limb symptomatology compared with prior shorter-term studies, which indicated no significant difference in clinical outcome between the two groups. Atlas article: This article demonstrated that for those patients with severe lumbar spinal stenosis, surgical treatment was associated with greater improvement in patient-reported outcomes than nonsurgical treatment at 4-year evaluation, even after adjustment for differences in baseline characteristics among treatment groups. The relative benefit of surgery declined over time but remained superior to nonsurgical treatment. They also mentioned that determining whether outcomes continue to converge will require longer-term evaluation. Katz article: This is a prospective multicenter observational study of the outcome of surgery for degenerative lumbar spinal stenosis. The authors showed that patients bothered predominantly by back pain preoperatively and those with greater medical comorbidity and functional disability are significantly less satisfied with the results of surgery for degenerative lumbar spinal stenosis. This article had no non-op control.

RECOMMENDED READING: Kornblum MB, Fishgrund JS, Herkowitz HN, et al: Degenerative lumbar spondylolisthesis with spinal stenosis: A prospective long-term study comparing fusion and pseudarthrosis. Spine 2004;29:726-733. Atlas SJ, Keller RB, Robson D, et al: Surgical and nonsurgical management of lumbar spinal stenosis: Fouryear outcomes from the Maine lumbar spine study. Spine 2000;25:556-562. Katz JN, Lipson SJ, Brick GW, et al: Clinical correlates of patient satisfaction after laminectomy for degenerative lumbar spinal stenosis. Spine 1995;20:1155-1160.

125. Doxorubicin, used in the treatment of sarcomas, is thought to affect cell death by inducing
12345necrosis. apoptosis. senescence. antiangiogenesis. downregulation of oncogenes.

PREFERRED RESPONSE: 2

The current view is that most chemotherapy is cytotoxic to tumor cells through induction of apoptosis. Doxorubicin induces damage to DNA by blocking Topoisomerase II action, an enzyme responsible for cutting and unwinding DNA during replication. This in turn results in programmed cell death.

RECOMMENDED READING: Menendez LR (ed): Orthopaedic Knowledge Update: Musculoskeletal Tumors. Rosemont, IL, American Academy of Orthopaedic Surgeons, 2002, pp 49-58.

126. A healthy, active 26-year-old woman sustained a displaced two-piece calcaneal fracture with subluxation of a posterolateral fragment into the subfibular recess. Recommended treatment should consist of
1- open or percutaneus reduction and internal fixation. 2- excision of the posterolateral fragment. 3- non-weight-bearing and early mobilization. 4- casting for 3 months. 5- posterior arthroscopic debridement.

PREFERRED RESPONSE: 1
As per Rammelts article current treatment concepts recommend ORIF for displaced intraarticular calcaneal fxs using lateral approach and anatomic reduction. A simple arthroscopic debridement wont be much different from non-op treatment since the fx is not reduced, making choices 3,4,5 similar to each other and incorrect. Choice 2 is incorrect because this question mentions a two-piece calcaneal fracture, assuming the piece is of significant and fixable size. Excision of articular weight-bearing surface is not indicated if you can fix it. The other 2 articles show that ORIF is better than non-op treatment. Correct answer 1. Barlas article compared the outcomes of displaced intraarticular calcaneal fractures in women treated operatively or nonoperatively. All fractures were closed injuries and had posterior facet displacement of more than 2 mm. Patients were randomly assigned to either the nonoperatively or operatively treated groups. Nonoperative treatment included ice and elevation, while operative treatment consisted of open reduction and internal fixation using a standard lateral approach. Operative treatment showed statistically significant better results when compared to nonoperative treatment in women. The authors concluded that displaced intraarticular calcaneal fractures in women should be treated by open reduction and internal fixation. Buckleys article was designed to determine whether ORIF of displaced intra -articular calcaneal fractures resulted in better general and disease-specific health outcomes at two years after the injury compared with those after nonoperative management. A standard protocol, involving a lateral approach and rigid internal fixation, was used for operative care. Nonoperative treatment involved no attempt at closed reduction, and the patients were treated only with ice, elevation, and rest. After unmasking the data by removal of the Workers Comp patients, the outcomes were significantly better in surgically treated patients.

RECOMMENDED READING: Rammelt S, Zwipp H: Calcaneus fractures: Facts, controversies and recent developments. Injury 2004;35:443-461. Barla J, Buckley R, McCormack R, et al: Displaced intraarticular calcaneal fractures: Long-term outcome in women. Foot Ankle Int 2004;25:853-856. Buckley R, Tough S, McCormack R, et al: Operative compared with nonoperative treatment of displaced intraarticular calcaneal fractures: A prospective, randomized, controlled multicenter trial. J Bone Joint Surg Am 2002;84:1733-1744.

Joe Park

127. AP and lateral radiographs of the foot of a 2year-old child are shown in Figures 42a and 42b. Which of the following modalities will best confirm the diagnosis?
12Lateral radiographs in maximum plantar flexion CT with three-dimensional reconstruction of the subtalar joint 3- MRI of the foot and ankle 4- Muscle biopsy 5- Electromyography and nerve conduction velocity studies

PREFERRED RESPONSE: 1 RECOMMENDED READINGS: Mazzocca AD, Thomson JD, Deluca PA, et al: Comparison of the posterior approach versus the dorsal approach in the treatment of congenital vertical talus. J Pediatr Orthop 2001;21:212-217. Kodros SA, Dias LS: Single-stage surgical correction of congenital vertical talus. J Pediatr Orthop 1999;19:42-48. Vaccaro AR (ed): Orthopaedic Knowledge Update 8. Rosemont, IL, American Academy of Orthopaedic Surgeons, 2005, pp 757-765.

The radiograph in image 42 demonstrates an example of congenital vertical talus. Per OKU 8, congenital vertical talus is a dorsolateral dislocation of the talonavicular joint associated with extreme plantar flexion of the talus. The foot has a rocker-bottom appearance and the head of the talus is prominent and palpable on the plantar-medial aspect of the midfoot. The hindfoot is plantar flexed. There is a dorsiflexion contracture of the forefoot on the hindfoot that prevents the creation of a longitudinal arch by passive manipulation. 50% cases associated w/ syndromes such as arthrogryposis, myelomeningocele, sacral agenesis, 50% of cases are bilateral.

A lateral radiograph of a congenital vertical talus in maximum dorsiflexion shows fixed plantar flexion of the talus. A lateral radiograph in maximum plantar flexion shows an irreducible dislocation of the talonavicular joint and confirms the diagnosis.

The other two sources show good results from either the posterior (Cincinnati incision) or dorsal approach. In the more recent paper, the authors demonstrated shorter tourniquet times and less dissection of the talonavicular joint with the dorsal approach. In addition, the dorsal approach allowed for more efficient approach to the dorsal dislocation of the talonavicuclar joint (performed with concurrent Achilles lengthening).

128. A 21-year-old college football player reports a history of feeling his shoulder pop in and out during play. Examination reveals mild apprehension with a significant anterior shift and a positive sulcus examination. Physical therapy has failed to provide relief. A current coronal T1-weighted MRI scan is shown in Figure 43. Based on these findings, what is the most likely diagnosis? 12345Supraspinatus tendon tear of the anterior portion Humeral avulsion of the inferior glenohumeral ligament Superior labral tear (SLAP lesion) Inferior labral tear Osteochondral injury of the humeral head

RECOMMENDED READINGS: Hewitt M, Getelman MH, Snyder SJ: Arthroscopic management of multidirectional instability: Pancapsular plication. Orthop Clin North Am 2003;34:549-557. PREFERRED RESPONSE: 2 Wolf EM, Cheng JC, Dickson K: Humeral avulsion of glenohumeral ligaments as a cause of anterior shoulder instability. Arthroscopy 1995;11:600-607.

The MRI demonstrates avulsion of the inferior glenohumeral ligament from its humeral attachment. It remains attached to its origin on the glenoid via the labrum. The first source addresses multi-directional instability (none of the other choices). The second source reviewed 64 shoulder arthroscopies performed for treatment of anterior instability. In 6 out of 47 (9.3%) of cases, a HAGL lesion was found and repaired arthroscopically. There were 47 Bankart lesions (avulsion of antero-inferior ligament/labral complex at the glenoid), and 11 cases of capsular laxity.

In patients with documented anerior instability without a demonstratable primary Bankart lesion, a HAGL lesion should be ruled out. This lesion is readily recognized arthroscopically and an appropriate repair of this lesion can restore anterior stability to the patient.

129.When using hyperbaric oxygen treatment for a crush injury to the extremity, which of the following mechanisms of action is present? 12345Increased hydrostatic pressures Increased tissue edema Increased gradient for diffusion Decreased leukocyte function Decreased arteriolar vasoconstriction


1. 2.

PREFERRED RESPONSE: 3 RECOMMENDED READING: Greensmith JE: Hyperbaric oxygen therapy in extremity trauma. J Am Acad Orthop Surg 2004;12:376-384.
Pages 380-381 describe 4 mechanisms for hyperbaric oxygen minimizing secondary injury in crush injuries and compartment syndromes. Enhanced gradient for oxygen diffusion due to increased arterial oxygen tensions Decreased tissue edema by causing increased arteriolar vasoconstriction, decreased upstream arterial resistance and decreased capillary hydrostatic pressures favoring resorption. Leukocyte function is enhanced (increased oxygen which is the limiting substrate for oxidative burst-mechanism for PMN leukocyte function) Decreased effects of reperfusion injury by quenching the production of peroxidation molecules and interrupting the metabolic cascades which result from microvascular disruption.

3.
4.

130. A 67-year-old man with diabetes mellitus has undergone initial casting for a displaced midfoot neuroarthropathic collapse. Examination reveals an unstable midfoot with blanching of the skin with weight bearing. What is the next most appropriate step in management?

123-

Charcot restraint orthotic walker (CROW) Triple arthrodesis Reduction of the midfoot and application of an external fixator 4- Midfoot arthroplasty 5- Rigid foot orthosis with posting of the lateral midfoot

PREFERRED RESPONSE: 1 RECOMMENDED READINGS: Trepman E, Nihal A, Pinzur MS: Charcot neuroarthropathy of the foot and ankle. Foot Ankle Int 2005;26:46-63. Mehta JA, Brown C, Sargeant N: Charcot restraint orthotic walker. Foot Ankle Int 1998;19:619-623.

Per the Mehta et al article, the majority of Charcot feet can and should be treated by primarily nonsurgical regimens. The Charcot restrain orthotic walker (CROW) is basically a custom bivalved AFO with a rocker bottom sole (see image below). Following an initial period in a total contact cast to enable edema reduction, the CROW is fabricated and helps control edema, enable ambulation and prevent deformity as coalescence proceeds (first source) and allows for better foot hygiene as it removable.

131.A 12-year-old boy has had mild ankle pain and a limp for the past 2 months. Laboratory studies show a WBC count of 7,800/mm3 (normal 3,500 to 10,500/mm3) and an erythrocyte sedimentation rate of 40 mm/h (normal up to 20 mm/h). Radiographs and a T1-weighted MRI scan are shown in Figures 44a through 44c. Management should consist of
12345antibiotics. nonsteroidal anti-inflammatory drugs. percutaneous radiofrequency coagulation. a clamshell orthosis. observation

PREFERRED RESPONSE: 1 RECOMMENDED READINGS: McCarthy JJ, Dormans JP, Kozin SH, et al: Musculoskeletal infections in children: Basic treatment principles and recent advancements. J Bone Joint Surg Am 2004;86:850-863. Morrissy RT: Bone and joint sepsis, in Morrissy RT, Weinstein SL (eds): Lovell and Winters Pediatric Orthopaedics, ed 5. Philadelphia, PA, Lippincott Williams & Wilkins, 2001, pp 459-505.

XRs and MRI demonstrate a well-defined metaphyseal lesion with slerotic rim in the distal tibia of a child. The history of 2 months of mild pain/limp and equivocal labs are consistent with subacute/chronic hematogenous osteomyelities. Children are typically less symptomatic with less symptoms, no fever, and laboratory values are often normal. Due to the subacute nature, radiographic changes are often found. The vast majority of cases are from Staph aureus but streptococcal infections are sometimes responsible. It is important that in children, tumors are ruled out ( for example: chondroblastoma). Culture all biopsies, and biopsy all cultures (JBJS article). In chronic cases (>1 month) surgical debridement and removal of sequestrum is often required. Therefore, treatment requires appropriate antibiotic treatment as well as I+D and biopsy of the lesion

132. A 66-year-old woman reports persistent pain after falling and being diagnosed with a foot sprain 1 year ago. She also notes that she is becoming flatfooted on that side. Radiographs are shown in Figures 45a and 45b. Surgical management should consist of 12345medial displacement calcaneal osteotomy. lateral column lengthening through the calcaneocuboid joint. lateral column lengthening through the calcaneus. tarsometatarsal arthrodesis. subtalar fusion.

PREFERRED RESPONSE: 4 RECOMMENDED READINGS: Mulier T, Reynders P, Dereymaeker G, et al: Severe Lisfrancs injuries: Primary arthrodesis or ORIF? Foot Ankle Int 2002;23:902-905. Komenda GA, Myerson MS, Biddinger KR: Results of arthrodesis of the tarsometatarsal joints after traumatic injury. J Bone Joint Surg Am 1996;78:1665-1676.

The radiographs demonstrate disruption of medial column (first metatarsocuneiform joint), the middle column (second and third metatarsocuneiform joints) and lateral column (fourth and fifth metarasocuboid joints). The metatarsals are abducted, and the flat foot deformity is visible on the lateral xr. The normal AP talus-first metatarsal angle is ~0 degrees, and the normal lateral height of the first metatarsal, as measured by talus-first metatarsal angle on lateral xr and should also be ~0 degrees. In this example, the first metatarsal is dorsiflexed compared to the talus on lateral xr. The first source states that open reduction and internal fixation with screws or partial arthrodesis is the treatment of choice in severe tarsometatarsal fracture dislocations. Primary complete arthrodesis should be reserved as a salvage procedure. The second source is a retrospective review of 41 patients treated for tarsometarsal joint disruptions. All joints with 3 mm of displacement and 15 degrees of malalignment were managed with realignment and arthrodesis with 4.5 or 3.5 mm screws. They achieved 31 out of 32 fusions and improved AOFAS scores from 44 pre-op to 78 post-op. Therefore, in this question, the optimal treatment would include realignment and tarsometarsal arthrodesis.

Jeff Pope

An obese 70-year-old woman reports increasingly disabling shoulder pain. History reveals that she sustained a fracture of her dominant shoulder in a fall 3 years ago. Initial management consisted of a sling for 2 weeks, followed by a physical therapy program for range of motion to prevent a frozen shoulder. A current radiograph is shown in Figure 46. Management should now consist of 12excision of the head segment and humeral head arthroplasty. open reduction, autogenous bone grafting, and internal fixation. 3- total shoulder arthroplasty. 4- supraspinatus cuff repair. 5- surface electrical coils (bone growth stimulator).

First lets start with the xray. You can see there is a nonunion of the humeral head with sclerotic edges about the fracture. There does not appear to be too much wear of the glenoid eliminating #3. Forget about #5 when remembering that this fracture is 3 years old. Supraspinatus cuff repair would not be the sole treatment for this injury, hence it is incorrect. Answer choice #2 may be an option if it was a younger patient and not this chronic, but in this situation hemiarthroplasty is the treatment of choice.

PREFERRED RESPONSE: 1
RECOMMENDED READINGS: Dines DM, Klarren RF, Altcheck DW, et al: Post-traumatic changes of the proximal humerus: Malunion, nonunion, and osteonecrosis. Treatment with modular hemiarthroplasty or total shoulder arthroplasty. J Shoulder Elbow Surg 1993;2:11-21. Beredjiklian PK, Iannotti JP, Norris TR, et al: Operative treatment of malunion of a fracture of the proximal aspect of the humerus. J Bone Joint Surg Am 1998;80:1484-1497.

134. Which of the following antimicrobials has been shown to be most effective against intracellular phagocytized Staphylococcus aureus in macrophages? 1- Clindamycin 2- Vancomycin 3- Rifampin 4- Cefazolin 5- Gentamicin

Initially cross off answers 1 & 5 as we do not use them to treat Staph. aureus. Of answers 2-4, only rifampin is lipophilic which increases its ability to penetrate the cells. This study however stated that although rifampin could penetrate the cell well, it had poor killing ability, which is why we use it with vancomycin. Also, this study was performed in vascular endothelial tissue, not macrophages, so these question writers can SUCK IT!.

PREFERRED RESPONSE: 3
RECOMMENDED READING: Darouiche RO, Hamill RJ: Antibiotic penetration of and bactericidal activity within endothelial cells. Antimicrob Agnts Chemother 1994;38:1059-1064. If you actually look this up you need to get a life.

135. An 11-year-old boy falls down the stairs in his home and sustains an open tibial fracture. Examination reveals a 5-cm wound defect with exposed bone at the base. According to his parents, he has received no vaccinations since completion of the initial immunization series at age 5 years. The physician should now recommend 1- administration of tetanus immune globulin. 2- administration of tetanus toxoid vaccine. 3- administration of both tetanus toxoid vaccine and tetanus immune globulin. 4- no further vaccinations. 5- hyperbaric oxygen therapy.

History
Fewer than 3 doses; or 2. Uncertain number of doses; or 3. No immunization At least 3 previous doses of tetanus vaccine, but the most recent more than 10 years ago At least 3 previous doses of tetanus vaccine, but the most recent more than 5 and less than 10 years ago

Treatment Give toxoid and immune globulin (if clean, minor wound consider toxoid only)

Give toxoid only

Give toxoid only (consider no immunization if clean, minor wound)

At least 3 doses of tetanus, with the most recent 5 years ago or less
Sexual intercourse with Reza

No immunization necessary

I suggest Vanco/Levaquin/Acyclovir (and praying)

PREFERRED RESPONSE: 2
RECOMMENDED READINGS: Pickering LK (ed): Tetanus: Red Book 2003 Report of the Committee on Infectious Diseases, ed 26. Elk Grove Village, IL, American Academy of Pediatrics, 2003, pp 611-618. Tolo VT: Management of the multiply injured child, in Beaty JH, Kasser JR (eds): Rockwood and Wilkins Fractures in Children, ed 5. Philadelphia, PA, Lippincott Williams & Wilkins, 2001, pp 75-89.

136.Figures 47a and 47b show the radiographs of a 22-year-old man who sustained a wrist fracture. CT scans obtained after closed reduction and splinting are shown in Figures 47c and 47d. Treatment should now include
12345repeat closed reduction and casting for 6 weeks. closed reduction and percutaneous pin insertion. closed reduction and application of external fixation. open reduction and internal fixation through a dorsal approach. open reduction and internal fixation through a volar approach.

The xray shows a comminuted, intra-articular, dorsally angulated distal radius fracture with loss of height and an ulnar styloid fracture. There is a CT scan after supposed closed reduction (plastics must have been on call), showing a step-off of the articular surface, as well as continued dorsal angulation. Immediately eliminate answers 1-3, b/c in a 22 y/o man the goal is anatomic reduction which cannot be accomplished in this fracture type w/o a formal ORIF. Had this been Ralph Lusskin an ace bandage and acetominophen would have been appropriate. Looking at the CT scans you can see that the majority of the comminution is dorsal, so the best approach would be dorsally so that you could pack the metaphysis with bone graft to maintain height as well as regain the volar tilt and eliminate the articular step-off. Estimated time for Dr. CraigScott to finish the case: 7-8 hours, just in time for the block to wear off.

PREFERRED RESPONSE: 4
RECOMMENDED READINGS: Melone CP Jr: Open treatment for displaced articular fractures of the distal radius. Clin Orthop 1986;202:103-111. Jupiter JB: Intra-articular distal radius fractures, in Berger RA, Weiss AP (eds): Hand Surgery. Philadelphia, PA, Lippincott Williams and Wilkins, 2004, pp 277-295. Knirk JL, Jupiter JB: Intra-articular fractures of the distal end of the radius in young adults. J Bone Joint Surg Am 1986;68:647-659. Trumble TE, Schmitt SR, Vedder NB: Factors affecting functional outcome of displaced intra-articular distal radius fractures. J Hand Surg Am 1994;19:325-340.

137. Which of the following diseases of bone (when nonmetastatic at diagnosis) carries the worst prognosis for 5-year survival?
12345Lymphoma Osteosarcoma Ewings sarcoma Pagets sarcoma Conventional chondrosarcoma

Osteosarcomas associated with Pagets disease alone and osteosarcomas of the skull have a poor prognosis, with a less than 10% 5 year survival rate. Conventional chondrosarcoma is a relatively slow and stable growing tumor. No single 5 year survival rate has been quoted but they are much higher than that of Pagets sarcoma. The recommended reading referred to the 5 yr survival rate of Ewings as an unsettled and controversial issue, however it has been reported to be between 22-55% post surgical resection. Osteosarcoma 5 yr survival rates have ranged between 60-85+% between studies. Lymphoma who really cares, if you want to ask Yee you must first see the Cupertino Courier (http://www.communitynewspapers.com/archives/cupertinocourier/04.10.96/tech.scholars. html) and The Journal of Biological Chemistry (http://www.jbc.org/cgi/reprint/273/17/10317). Have fun.

PREFERRED RESPONSE: 4
RECOMMENDED READING: Simon MA, Springfield DS (eds): Surgery for Bone and Soft-tissue Tumors. Philadelphia, PA, Lippincott-Raven, 1998, p 274.

138. Which of the following tests correlates most closely with outcome after an anterior cruciate ligament tear? 1- Lachmans 2- Anterior drawer 3- Pivot shift 4- Quadriceps active 5- KT-1000

The first of the two articles compared post ACL reconstructed knees using KT 1000, Lachman, and pivotshift measures versus functional outcome and found that only the pivot-shift grade was associated with: patient satisfaction, partial giving way, full giving way, difficulty cutting, difficulty twisting, activity limitation, overall knee function, sports participation, and Lysholm score. Quadriceps active test was not evaluated. The point of the second article is that functional outcome measurements that are partially based on joint laxity measures, such as the International Knee Documentation Committee form, may artificially overestimate the disability after anterior cruciate ligament rupture.

PREFERRED RESPONSE: 3
RECOMMENDED READINGS: Kocher MS, Steadman JR, Briggs KK, et al: Relationships between objective assessment of ligament stability and subjective assessment of symptoms and function after anterior cruciate ligament reconstruction. Am J Sports Med 2004;32:629-634. Snyder-Mackler L, Fitzgerald GK, Bartolozzi AR III, et al: The relationship between passive joint laxity and functional outcome after anterior cruciate injury. Am J Sports Med 1997;25:191-195.

Dave Ruchelsman

139. Figures 48a through 48c show the radiographs of a child with dwarfism. The skeletal dysplasia is a result of a genetic defect in 1- bone morphogenetic protein-2. 2- bone morphogenetic protein-6. 3- fibroblast growth factor. 4- fibroblast growth factor receptor-3. 5- Indian hedgehog.

48A

48B

48C

PREFERRED RESPONSE: 4

139. Several chondrodysplasia phenotypes result from mutations in the FGFR3 gene, including achondroplasia, hypochondroplasia, thanatophoric dysplasia, and severe achondroplasia. Achondroplasia is the most common form of dwarfism and has an AD inheritance pattern. The single amino acid substitution leads to sustained FGFR-3 activity, which leads to growth retardation in the proliferative zone of the physis. The diagnosis of achondroplasia is based on clinical examination and radiographic features. Infants with achondroplasia typically present with rhizomelic limb shortening involving the proximal limb segments (i.e. humerus and femur), normal trunk length, genu varum, macrocephaly, frontal bossing, midfacial hypoplasia, hypotonia, and trident hands. Classic radiographic features include narrowing of the interpedicular distance in the lumbar spine (Fig 48B), shortened pedicles (Fig 48C), squared iliac wings, narrow notchlike sacrosciatic notches, chevron-shaped epiphyseal ossification centers on the metaphysic, and horizontal acetabulaall seen on AP pelvis (Fig 48A). Additional facts: The foramen magnum may be stenosis. Ventriculomegaly is common and often mild, without progression or cause for treatment with shunting. Young children should have a baseline computed tomography scan of the brain and cervical spine and/or MRI of the entire spine. Cervicomedullary cord compression causes central apnea and can be life-threatening. Symptomatic spinal stenosis occurs in the second tofourth decades and decompression is indicated when stenosis is symptomatic. Thoracolumbar kyphosis is a common in infancy, is thought to be secondary to truncal hypotonia, and typically resolves as the child achieves independent ambulation. Bracing is reserved for severe, progressive kyphosis, with apical verterbral wedging. Fusion is indicated for failure of bracing

140. According to the Lauge-Hansen classification, what is the mechanism of injury for the fracture shown in Figure 49? 1- Supination external rotation 2- Supination adduction 3- Pronation abduction 4- Pronation external rotation 5- Pronation dorsiflexion

49

PREFERRED RESPONSE: 2

140. The XR demonstrates a Lauge-Hansen supination-adduction II injury pattern (SA-II). In SA injuries, the lateral injury consists of ATFL disruption or fibula fx, which is transverse and below the level of the syndesmosis (Weber A). The medial sided injury is a vertically-oriented fx of the medial malleolus. This injury can be associated with an osteochondral impaction injury to the medial talar dome or tibial plafond. As with SER injury patterns, supination ankle injuries begin on the lateral side.

ANKLE FRACTURES

Danis Weber
Weber A SA I-II Weber B SE I-IV PA I-III Weber C PE I-IV

vs Lauge Hansen

11

141. Figure 50a shows the preoperative AP pelvic radiograph of a 58-year-old woman with osteoarthritis who is scheduled to undergo total hip arthroplasty. Six weeks after surgery, the patient reports that she has difficulty walking because the left leg is longer than the right. Figures 50b and 50c show the AP pelvic radiographs at 6 weeks and 3 months later. What is the cause of the patients early postoperative gait impairment? 1- Excessive length of the modular femoral head 2- Excessive femoral offset 3- Lateralized acetabular component 4- Apparent limb-length discrepancy 5- Incomplete seating of the femoral component

50A

50B

50C

PREFERRED RESPONSE: 4

141. Konyves and Banister (JBJS-Br 2005) assessed LLD and hip function (Oxford Hip Score) in 90 patients following primary THA at 3 and 12 months postoperatively. Following THA, hip scores improved at both postoperative time points, by a mean of 26 and 30 points, respectively. The authors noted that following primary THA. 56 (62%) limbs were long by a mean of 9 mm. This true LLD was perceived by 24 (43%) patients after 3 months and 18 (33%) after 12 months. Most importantly, the mean hip score in patients who perceived true lengthening was 27% worse than the rest of the population after three months and 18% worse after 12. In 55/56 (98%) patients, the LLD originated in the femoral component. The results reiterate the detrimental effect of LLD on clinical outcomes following primary THA. The preoperative AP pelvis (Fig 50A) demonstrates advanced degenerative changes about the left hip on both the femoral and acetabular sides. While degenerative changes are present in the right hip as well, the changes are more advanced on the left, with marked joint space narrowing superolaterally. Assessment of pre- and post-operative limb lengths may be performed utilizing a single, standard AP views of the pelvis with the hips extended and internally rotated. This method has been reported to be as reliable and reproducible, with a measurement error of 1 mm in prior studies. Reference points used to assess limb length include the inferior margin of the acetabular teardrop (or the base of the ischial tuberosity), the most prominent point of the lesser trochanter, and the centre of rotation of the femoral head. Measuring the distances between these reference points to the nearest millimeter is then performed postoperatively and compared to preoperative values. Note that this method does not take into account other discrepancies of length in the lower limb but does give an accurate assessment of the situation pre- and post-operatively. In Fig 50B (6 weeks postop) there is an apparent LLD with L>R secondary to the pelvic obliquity creating a discrepancy between the distances meausured between the inferior margin of the teardrop and the lesser trochanter on each side. This LLD is not apparent in Fig 50C when the pelvic is level. There is no evidence for excessive femoral offset, femoral head length, or incomplete seating of the femoral component. A line tangential to the the tip of the GT intersects the center of the femoral head. The acetabular component is fully medialized to the teardrop, and the theta angle looks grossly normal as does the version of the cup.

142. A 68-year-old woman has had pain in her left shoulder for the past 6 months. She reports that the pain is constant and is worse with motion. Examination reveals there is 90 degrees forward elevation and external rotation to neutral (both active and passive). An AP radiograph is shown in Figure 51. A CT scan will most likely reveal glenoid wear in what area? 1- Anterior 2- Posterior 3- Superior 4- Inferior 5- Central

51

PREFERRED RESPONSE: 2

142. The AP XR demonstrates advanced degenerative changes about the right glenohumeral joint with severe joint space narrowing, a large inferior osteophyte (goats-beard), flattening of the humeral head articular surface, and subchondral sclerosis. These radiographic features are most consistent with primary gelnohumeral osteoarthritis. Primary GH OA is associated with anterior capsular contracture manifesting as limited ER, as in this case. Additional radiographic findings may include posterior subluxation of the humeral head and posterior glenoid erosion (as seen on axillary of axial CT). Glenoid wear in the transverse plane can be assessed on the axillary radiograph or axial CT scans with use of the classification system of glenoid morphology described by Walch et al. Walch et al (J Arthroplasty 1999) reviewed 113 CT scans in osteoarthritic shoulders to derive a prognostic classification of glenoid morphology in primary GH OA. The authors defined 3 main glenoid types: Types AC. Type A (59%) glenoids were characterized by a well-centered humeral head within the glenoid. Type A is characterized by an equal balance of forces acting on the glenoid and a centralized head. Glenoid wear is either minor (type A1) or major with substantial central wear causing a glenoid cup (type A2). The symmetric erosion was explained by the absence of subluxation. In Type B (32%) glenoids, posterior subluxation of the humeral head was responsible for asymmetric loading against the posterior glenoid and manifested as a posterior wear pattern. Type B has asymmetrical force distribution on the glenoid, leading to a loss of joint space (predominantly posteriorly), subchondral sclerosis (more marked on the posterior edge of the glenoid), and/or multiple posterior geodes (type B1), or to a posterior cup that gives the appearance of a double concavity on the glenoid surface (type B2). Type C (9%) glenoid morphology was defined by glenoid retroversion >25 degrees, and represents glenoid dysplasia. Retroversion was primarily of dysplastic origin and explained early-onset OA. In primary GHOA, this classification of the glenoid can discriminate retroversion between posterior erosion and dysplasia.

Classification of glenoid morphology in osteoarthritis in the transaxial plane, according to the system of Walch et al

Additionally, the glenoid inclination angle has been described in GH OA. It is measured with use of one line drawn along the superior and inferior glenoid rim (the glenoid line) and another line drawn along the lateral base of the coracoid process (the coracoid base line) from the superior glenoid rim perpendicular to the bottom margin of the radiograph, as described by Boileau and Walch (Boileau P, Walch G. Normal and pathological anatomy of the glenoid: effects on the design, preparation and fixation of the glenoid component. In: Walch G, Boileau P, editors.Shoulder arthroplasty . New York: Springer; 1999. p127 -40.) By convention, a negative number indicates an inferiorly directed glenoid, and a positive number indicates a superiorly directed glenoid. Decentralization is defined as a deviation of the center of the humeral head of >5 mm superiorly or inferiorly from the center of the glenoid in the coronal plane.

Classification of glenoid inclination. A: In type 0, the coracoid base line (red) and the glenoid line (blue) run parallel (the brown line represents the inferior border of the radiograph). B: In type 1, the coracoid base line and the glenoid line intersect below the inferior glenoid rim. C: In type 2, the coracoid base line and the glenoid line intersect between the inferior glenoid rim and the center of the glenoid. D: In type 3, the coracoid base line and the glenoid line intersect above the coracoid base.

143. What subset of patients is expected to have a poorer outcome following open reduction and internal fixation of a displaced intra-articular calcaneal fracture? 1- Younger men 2- Women 3- Patients with a lower postfracture Bohler angle 4- Patients not receiving workers compensation 5- Patients with a lighter workload

PREFERRED RESPONSE: 3

143. Buckley et al (JBJS 2002) published a multicenter, prospective, randomized controlled trial, examining operative vs. nonoperative treatment of displaced (>2mm) intra-articular calcaneal fractures based on axial and coronal CT scans of the posterior facet. Exclusion criteria included medical contraindications to surgery, previous calcaneal injury, infection, or tumor, coexistent foot injury, extra-articular calcaneal fracture, open calcaneal fracture, injury occuring more than 14 days before presentation, or a head injury. The purpose of this study was to determine whether ORIF of displaced intra-articular calcaneal fractures results in better general and disease-specific health outcomes at two years after the injury compared with those after nonoperative management. Patients randomized to operative management underwent ORIF via lateral approach. Nonoperative treatment involved no attempt at closed reduction, and the patients were treated only with RICE. Outcome measures included radiograohic assessment of reduction, SF-36, and a disease-specific VAS. Results: 424 with 471 displaced intra-articular calcaneal fractures were enrolled. 309 (73%) were available for review at a minimum 2 yr f/u. Without stratification of the groups, the functional results after nonoperative care of displaced intra-articular calcaneal fractures were equivalent to those after operative care based on similar SF-36 and VAS scores between the 2 groups. However, after unmasking the data by removal of the patients who were receiving Workers' Compensation, the outcomes were significantly better in the following groups of patients receiving ORIF: (1) Patients not receiving Workers' Compensation (significantly higher satisfaction scores) (2) Women (higher SF-36 scores) (3) Age <29 (4) Moderately lower Bhler angle (0 to 14) (5) Comminuted fracture (6) Lighter workload (7) Anatomic reduction or step-off <2 mm following ORIF

144. Compared to normal synovium, which of the following best characterizes synovium in patients with rheumatoid arthritis?

12345-

Disruption of the basement membrane Decreased number of blood vessels Increased apoptosis Prominent intimal hyperplasia Abundant lymphocytes in the pannus

PREFERRED RESPONSE: 4

144. The normal synovial intimal layer is only 13 cells thick. See Fig below.

In RA, the synovial intimal lining is hypertrophied (8-10 cells thick). Primary cell populations in this layer are fibroblasts and macrophages. See Fig below.

The subintimal area of the synovium is where the synovial blood vessels are located and this area normally has very few cells. In RA, however, the subintimal area is heavily infiltrated with inflammatory cells, including T and B lymphocytes, macrophages and mast cells. The intense cellular infiltrate is accompanied by angiogenesis and pannus formation with progressive erosion of adjacent cartilage and subchondral bone. The pannus is characterized by thick villi with neovascularization and lymphoid aggregates. The article by Koch is a review of the various cytokines, chemokines, and growth factors present in RA patients and animal models of inflammatory arthritis that have been shown to promote angiogenesis. FGFs are well documented angiogenic factors. Basic science research has demonstrated that the synovium of patients with RA and joints from rats with adjuvant induced arthritis (AIA) contains increased amounts of FGF-2. Several groups have shown that VEGF, an endothelial selective growth factor, is prominent in the human RA joints and serum. Furthermore, IL1 and TNF, important inflammatory mediators of RA, have been shown to induce fibroblast expression of VEGF. These angiogenic cytokine and growth factors are thought to exert their effect via cellular adhesion molecules/integrins. Several studies indicate that angiogenic markers have the potential to help guide medical management of RA.

Theo Shybut

145. A Syme (through the ankle) amputation is considered most appropriate for which of the following patients?

1- A 25-year-old man with osteomyelitis of the calcaneus following open reduction of an intra-articular fracture 2- A 35-year-old man with a severe crushing and degloving injury to the forefoot with intact plantar skin 3- A 55-year-old man with diabetes mellitus and an anklebrachial index of 0.30 4- A 65-year-old woman with diabetes mellitus and gangrene isolated to the lesser toes secondary to embolism 5- A 75-year-old woman with diabetes mellitus and a failed partial calcanectomy

PREFERRED RESPONSE: 2
Indications for amputations include trauma, tumor, infection, peripheral vascular disease, congenital anomaly, and any rapidly progressive flesh-eating rash acquired from using the bathroom or scrub sink after Reza, Kaplan, Miyamoto, etc. Seriously, if you dont have the immunoglobulins, cut if off. Considerations in determining the level of amputation include the metabolic cost of walking on the amputated limb (increased with more proximal level), load transfer, and wound healing. Factors contributing to wound healing include nutrition, immune status, and delivery of oxygenated blood, which is a function of arterial supply and blood oxygen content, meaning proximal is generally better in this regard. The Symes amputation is a trans-ankle amputation. A patent posterior tibial artery is essential, as is a viable heel pad. Thusly, I would be less enthusiastic about doing this amputation on the last 3 patients because they are older diabetics and the younger 2 of those 3 have obvious vascular disease. That leaves the first two choices. The patient with calcaneal osteomyelitis and a prior calcaneal fracture has both traumatic injury and infection in the hindfoot, which would be supplying the soft tissue coverage for the stump, whereas the patient with the forefoot injury is specifically noted to have intact plantar skin, making me much more enthusiastic about the operation in that patient.

RECOMMENDED READINGS: Brodsky JW: Amputations and prostheses of the foot and ankle, in Coughlin MJ, Mann RA (eds): Surgery of the Foot and Ankle, ed 7. St Louis, MO, Mosby, 1999, pp 970-1006. Mizel MS, Miller RA, Scioli MW (eds): Orthopaedic Knowledge Update: Foot and Ankle 2. Rosemont, IL, American Academy of Orthopaedic Surgeons, 1998, pp 305-314.

146. A 58-year-old woman with rheumatoid arthritis reports progressive neck pain and difficulty with fine motor movements, including playing cards and handling coins. Examination revealed hyperreflexia without objective weakness. What is the most important radiographic factor that may predict neural recovery after decompressive surgery?

12345-

Basilar invagination of less than 1 cm Atlanto-dens interval of less than 5 mm Posterior atlanto-dens interval of greater than 13 mm Subaxial subluxation of less than 3.5 mm Rotatory subluxation of less than 10 degrees

PREFERRED RESPONSE: 3
This patient demonstrates signs and symptoms of cervical myelopathy. RA is an autoimmune disease that produces an inflammatory synovitis, resulting in pannus formation that destroys involved joints. In the cervical spine this results in subluxation, instability, and, ultimately, neural compromise. Boden et al analyzed their experience with 73 rheumatoid patients over a 20 year period with average 7 year followup and found that the posterior atlanto-odontoid interval and the diameter of the subaxial sagittal canal measured on the cervical radiographs demonstrated statistically significant correlations with the presence and severity of paralysis. All of the patients who had a Ranawat Class-III neurological deficit had a posterior atlanto-odontoid interval or diameter of the subaxial canal that was less than fourteen millimeters. In contrast, the anterior atlanto-odontoid interval, which had previously been reported, did not correlate with paralysis. The prognosis for neurological recovery following the operation was not affected by the duration of the paralysis but was influenced by the severity of the paralysis at the time of the operation. The most important predictor of the potential for neurological recovery after the operation was the preoperative posterior atlanto-odontoid interval. In patients who had paralysis due to atlanto-axial subluxation, no recovery occurred if the posterior atlanto-odontoid interval was less than ten millimeters, whereas recovery of at least one neurological class always occurred when the posterior atlanto-odontoid interval was at least ten millimeters. If basilar invagination was superimposed, clinically important neurological recovery occurred only when the posterior atlanto-odontoid interval was at least thirteen millimeters. All patients who had paralysis and a posterior atlanto-odontoid interval or diameter of the subaxial canal of fourteen millimeters had complete motor recovery after the operation.

RECOMMENDED READINGS: Boden SD, Dodge LD, Bohlman HH, et al: Rheumatoid arthritis of the cervical spine: A long-term analysis with predictors of paralysis and recovery. J Bone Joint Surg Am 1993;75:1282-1297. Monsey RD: Rheumatoid arthritis of the cervical spine. J Am Acad Orthop Surg 1997;5:240-248.

147. A 2-year-old boy has a limp and anterior bowing of the leg. A lateral radiograph is shown in Figure 52. What is the most likely diagnosis?

12345-

Unicameral bone cyst Osteofibrous dysplasia Adamantinoma Ewings sarcoma Metastatic neuroblastoma

Xray

PREFERRED RESPONSE: 2
The radiograph shows an eccentric, intracortical, lytic, expansile multilocular lesion with sclerosis of the inner cortex occurring in the tibia. There is associated bowing. Radiographically this is either osteofibrous dysplasia or adamantinoma. (At this point in the test Yee was bored by and done with this question, had mentally completed 3 Times crosswords, and was moving on to taking an out of sight nap behind her chair. With gloves, scarf, and earmuffs on, of course. Ilya, on the other hand, also bored by and done with this question, was looking forward to lifting shoulders at his posh upper west side gym and simultaneously wondering if tumors taste like chicken). Osteofibrous dysplasia typically presents in children aged 1-5. Adamantinoma occurs in the second decade. Whether or not the two represent related benign and malignant processes remains controversial. Osteofibrous dysplasia is typically managed nonoperatively until skeletal maturity after which time the lesion can be curetted and bone grafted and any deformities corrected. Earlier curettage has a high rate of recurrence. Bowing deformity is not generally associated with UBC, Ewings, or neuroblastoma. UBC occurs in older children (age 5-15, average age 9) and is metaphyseal, commonly occurring in the proximal humerus and proximal femur. Look for the fallen fragment or fallen leaf sign on radiographs. Ewings also occurs more typically in this slightly older age range. Radiographically Ewings may be predominantly lytic or sclerotic, but would characteristically show periosteal reaction and new bone formation in an onionskin pattern. The depicted lesion does not show such aggressively malignant characteristics. Finally, neuroblastoma occurs at a median age 22 months. Uncommonly it presents as Hutchinson syndromelimping and irritability due to skeletal metastases. Radiographically, though, the metastases produce diffuse osteopenia and cortical periosteal reaction.

RECOMMENDED READING: Ozaki T, Hamada M, Sugihara S, et al: Treatment outcome of osteofibrous dysplasia. J Pediatr Orthop B 1998;7:199-202.

148. A 10-year-old girl has sustained three inversion sprains of her left ankle in the past 6 months. She also reports variable activity-related midfoot pain. Radiographs and CT scans are shown in Figures 53a through 53d. If nonsurgical management fails to provide relief, surgical management should consist of resection of the
12345coalition and interposition of the extensor digitorum brevis. coalition and interposition of a split portion of the flexor hallucis longus. periarticular osteophytes at the talonavicular joint. os trigonum and stabilization of the flexor hallucis longus. Stiedas tubercle and mobilization of the flexor hallucis longus.

Images

Images

PREFERRED RESPONSE: 1
The radiographs and CT show a calcaneonavicular tarsal coalition. You should be suspicious of this diagnosis in children with recurrent ankle sprains. (Remember that the coalition may be fibrous or cartilaginous). It results from a failure of differentiation and segmentation of mesenchyme. Onset of pain occurs around 9-13 years old, thought to be around the time the coalition ossifies. Initial management is conservative, with empiric use of heel cup with a medial wedge or longitudinal arch support or short walking cast followed by UCBL orthosis. Once you have recognized this as a calcaneonavicular coalition, you have to decide if youre going to interpose EDB or FHL. FHL interposition is an option in the treatment of talocalcaneal coalitions. Calcaneonavicular coalition resection with interposition of EDB results in good pain relief in 80% or more of patients. If you look at the anatomy of the foot, the proximity of the tendons to the proposed site of interposition makes sense. The subtalar joint is more posterior, and the FHL courses just posterior to the talus. The calcaneonavicular joint is not as plantar because the anterior process of the calc projects dorsal and anterior to articulate with the navicular. Simple resection without tendon interposition produces inferior results as there is a high rate of bone reformation.

RECOMMENDED READINGS: Vaccaro AR (ed): Orthopaedic Knowledge Update 8. Rosemont, IL, American Academy of Orthopaedic Surgeons, 2005, pp 757-765. Herring JA: Disorders of the foot, in Herring JA (ed): Tachdjians Pediatric Orthopaedics, ed 3. Philadelphia, PA, WB Saunders, 2002, pp 891-1037. Cooperman DR, Janke BE, Gilmore A, et al: A three-dimensional study of calcaneonavicular tarsal coalitions. J Pediatr Orthop 2001;21:648-651.

149. Figure 54 shows a graphic representation depicting the current understanding of signaling pathways leading to the formation of multinucleated osteoclasts that can resorb bone. Which of the following is known to directly inhibit the RANKL-mediated osteoclast formation process? 1- OP-1 2- Osteocalcin 3- Osteoprotegerin 4- TNF- 5- IL-1

PREFERRED RESPONSE: 3
If you havent memorized the signaling pathways leading to multinucleated osteoclast formation you should probably drop out of residency right now. Seriously, what kind of surgeon are you going to be? Take a 10 minute time-out, go sit in the corner, and think about what youve done. Not included among the references was this gem: Lacey DL, Timnis E, Tan HL, Kelley MJ. Dunstan CR, Burgess R. Osteoprotegerin ligand is a cytokine that regulates osteoclast differentiation and activation. Cell 1998;93: 165-76. The title almost says it all. In brief, wear debris is phagocytosed by macrophages, which produce proinflammatory cytokines (i.e. TNF-a, IL-1, IL-6) in response to the particles. These cytokines promote the genesis of an erosive interfacial membrane, which stimulates osteoclasts to resorb the bone. While many different cytokines contribute to this process, it has been proposed that there are two final effector molecules that ultimately control this wear debris induced osteolysis. These are the receptor activator of NFKB-ligand, RANKL (also called OPGL, ODF, TRANCE) that binds to its cell surface receptor, RANK (also called ODAR) and its decoy receptor osteoprotegerin (OPG). RANKL is the osteoclast differentiation factor that directly stimulates the differentiation of osteoclast progenitors to mature osteoclasts by signaling through its membrane receptor RANK. Physiologically, RANKL signaling is negatively regulated by its soluble receptor antagonist protein OPG, which binds and sequesters RANKL, and thereby prevents its interaction with the membrane receptor RANK. RANK signaling is absolutely required for osteoclast differentiation, activation, and survival. Ok, good talkIll see you out there.

RECOMMENDED READINGS: Clohisy D: Cellular mechanisms of osteolysis. J Bone Joint Surg Am 2003;85:S4-S6. Goater JJ, OKeefe RJ, Rosier RN, et al: Efficacy of ex vivo OPG gene therapy in preventing wear debris induced osteolysis. J Orthop Res 2002;20:169-173..

150. What is the order of resorption rates from fastest to slowest of the following bone graft substitutes?

12345-

Calcium sulfate, tricalcium phosphate, hydroxyapatite Hydroxyapatite, calcium sulfate, tricalcium phosphate Hydroxyapatite, tricalcium phosphate, calcium sulfate Tricalcium phosphate, calcium sulfate, hydroxyapatite Tricalcium phosphate, hydroxyapatite, calcium sulfate

PREFERRED RESPONSE: 1
Most of you probably get your britches all up in a bunch in anticipation of each months hot new installment of Calcified Tissue International. In case you missed the one suggested reading, a landmark 2002 study of bone graft substitute in rabbit tibial defects, you can still reason through the question. Calcium sulfate is the simplest moleculeone calcium, one SO4. It should be turned over fastest. Tricalcium phosphate is Ca3(PO3)2more complex, more time to turn over. Hydroxyapetite is the largest molecule. It will take the most time to be resorbed. In the study rabbits were surgerized with a dental drill to create 5x15mm defects that were filled with either calcium sulfate or a hydroxyapetite/calcium carbonate hybrid created by partial conversion of calcium carbonate. Not surprisingly their radiographic and histomorphologic studies of the tibias showed faster resorption in the calcium sulfate group. Of note, while the HA/CC hybrid may seem more complicated than plain HA and hence absorb more slowly, it is actually CC that is partially converted to HA since for a given amount of graft substitute it is mostly CC (simple) as opposed to HA (complex), it absorbs faster. Also note that calcium sulfate is the mineral in plaster of paris.

RECOMMENDED READING: Jamali A, Hilpert A, Debes J, et al: Hydroxyapatite/calcium carbonate (HA/CC) versus plaster of Paris: A histomorphometric and radiographic study in a rabbit tibial defect model. Calcif Tissue Int 2002;71:172-178.

Eric Strauss

151. What is the most common malalignment after intramedullary nailing of the tibia?
1- Varus with proximal one third fractures 2- Apex posterior with proximal one third fractures 3- Rotational with distal one third fractures 5- Valgus with distal one third fractures with an intact fibula

Displaced, unstable fractures of the tibial diaphysis are commonly treated with intramedullary fixation as many studies have shown superior functional outcome and lower complication rates compared to ORIF, external fixation or cast immobilization. However, malalignment is a potential problem associated with IM nail fixation of tibia fractures. The type of malalignment that occurs depends on the location of the fracture and the status of the fibula. IM nail fixation of proximal tibia fractures is technically difficult and is prone to the development of malalignment. The most common malalignment associated with intramedullary nailing of proximal 1/3 fractures is valgus and procurvatum (eliminating choices 1 and 2). Valgus displacement/deformity occurs as a result of greater space laterally than medially in the intramedullary area of the proximal tibial metaphysis, a nail starting point that is too medial, tension on the proximal fragment from the pes anserinus and eccentric reaming in the proximal metaphyseal segment. Flexion deformity with proximal 1/3 fractures is caused by flexion of the knee during intramedullary nail insertion, the shape and location of the sagittal bend in the nail and the eccentric starting point and entrance angle of the nail. Can avoid these deformities by using a starting point that is slightly medial to the lateral tibial spine, placing the knee in a semiextended position during nail insertion or using Poller screws.

For distal 1/3 tibia fractures with an intact fibula, the fracture tends to go into varus alignment (eliminating choice 5). Puloski et al examined the incidence and severity of rotational malalignment of the tibia following reamed intramedullary nail fixation in 25 consecutive patients with tibial shaft fractures. In this study the authors obtained post-operative CT scans for each patient and compared the rotational alignment with the contralateral tibia, defining malrotation as IR/ER deformity >10 degrees. The authors found that 5 of the 22 patients (22%) had evidence of malrotation >10 degrees and 4 of these 5 patients had distal third tibia fractures. Thus the correct answer is 3 rotational malalignment with distal one third fractures. The other cited reference was a prospective randomized trial comparing IM nailing of tibia fractures using fracture table traction versus manual traction. The authors found in their review of 85 fractures in 79 patients that manual traction provided equal results with respect to operative parameters and quality of fracture reduction compared with fracture table traction while significantly reducing positioning time. Interesting, but not helpful for answering the question.
PREFERRED RESPONSE: 3 RECOMMENDED READINGS: Puloski et al. Rotational Malalignment of the Tibia Following Reamed Intramedullary Nail Fixation. J Orthop Trauma. 2004. 18:397-402 McKee et al: A Prospective, Randomized Clinical Trial Comparing Tibial Nailing Using Fracture Table Traction Versus Manual Traction. J Orthop Trauma. 1999. 13:463-469

152. A 5-year old boy was struck by a truck and sustained a posterior right hip dislocation that was reduced in the emergency department. A postreduction CT scan is shown in Figure 55. What is the next most appropriate step in treatment?
1- Arthroscopic excision of the torn labrum 2- Inspection of the joint through an anterolateral approach 3- Inspection of the joint through a posterior approach 4- Skeletal traction for 3 weeks 5- Application of a hip spica cast for 4 weeks.

Figure 55

Traumatic hip dislocation in a pediatric patient is an uncommon injury. Most cases are posterior and favorable outcomes can be expected if expedient reduction is performed. Avascular necrosis of the femoral head is the most feared complication, with studies showing that the only statistically proven risk factor is delay between dislocation and reduction. If the delay is greater than 6 hours, the risk of avascular necrosis is increased 20-fold. According to the literature, reduction of a pediatric dislocated hip is usually easy to achieve, however labral, capsule and/or osteochondral fragment interposition may prevent concentric reduction requiring surgery to achieve an anatomic reduction. Often the interposed osteochondral fragment stems from a posterior wall acetabular fracture associated with the traumatic hip dislocation. In the figure associated with question, the CT cut demonstrates joint space widening secondary to retained osteochondral fragments from the posterior wall of the acetabulum. This non-concentric reduction requires operative exploration via the posterior approach (choice 3 and the correct answer). Vialle et al retrospectively reviewed 35 hip dislocations in skeletally immature patients occurring over a 22 year period. Thirty-three of the thirty-five dislocations were posterior dislocations and in 27 of these posterior dislocations the injury was isolated without an associated acetabular lesion. Overall, 9 kiddos (including 6 with acetabular fractures) required operative intervention to remove interposed joint capsule and/or osteochondral fragments to achieve anatomic reduction. Overall, at a mean follow up of 3 years, the outcomes were good in this series with 30/35 patients having normal, pain-free ROM in the injured hip and Harris Hip Scores of 100.

In one of the cited papers (Quick et al), the authors discuss the Watts Classification of Acetabular Fractures a specific classification system to deal with the intraarticular pathology and prognosis associated with this injury. Type A B Description Small fragments associated with dislocation of the hip joint Undisplaced fractures associated with pelvic fractures

C
D

Linear fractures with hip joint instability


Central fracture dislocation of the hip

Conservative treatment is indicated in simple non-displaced fractures or those through nonweightbearing areas (Watts Type A and B) with some evidence of remodeling present with these injuries. Presence of larger intra-articular osteochondral fragments (evidenced by asymmetric reduction as present in the patient from the question) warrants an operative exploration through a posterior approach, as do injuries involving the weightbearing aspect of the acetabulum (Watts Type C). For Watts Type D injuries, the risk of severe complications is so high with operative treatment that there is no consensus on the best method of treatment.

PREFERRED RESPONSE: 3 RECOMMENDED READINGS: Vialle et al Traumatic Hip Dislocation in Childhood. J Pediatr Orthop. 2005. 25:138-144. Quick et al Pediatric Fractures and Dislocations of the Hip and Pelvis CORR. 2005. 432: 87-96

153. A 67 year old man reports pain and weakness in his dominant arm. He states that he is no longer able to play golf and finds it increasingly difficult to tuck his shirt behind his back. Radiographs reveal significant arthritis. In preparation for shoulder arthroplasty, the CT scan shown in Figure 56 is obtained. Based on the CT scan findings, the patient should be advised that
1- A destructive lesion of the glenoid needs biopsy 2- The humeral metaphysis is deficient and bone cement will be required 3- There is inadequate bone quantity to support a glenoid component 4- A rotator cuff repair will be needed in addition to joint arthroplasty 5- A metal-backed glenoid should be used because of poor glenoid bone

Figure 56

Total shoulder arthroplasty has proved to be successful and reliable in relieving the painful symptoms of an arthritic glenohumeral joint. Several studies have demonstrated excellent pain relief and improved motion in association with the procedure. However, complications do occur, most common being failure of the glenoid component and glenoid component fixation. Asymmetric glenoid wear or loss of bone stock can contribute to the development of these complications. The reported rate of glenoid component loosening (defined as dislocation, migration or a progressive radiolucent line) is approximately 10% inadequate glenoid bone stock and lack of congruent component contact have been associated with an increased rate of component loosening. Some degree of glenoid wear or resorption is common in arthritic shoulders being considered for shoulder arthroplasty. Very poor glenoid bone stock, as seen in the CT scan in Figure 56 may preclude the use of a glenoid component and treat the patient with a hemiarthroplasty (choice 3 the correct choice) or require the use of internally fixed bone grafts to restore glenoid fossa volume and version. Friedman et al (Orthop Trans 1992. 16:66) recommended bone grafting if the retroversion of the glenoid surface exceeded 15 degrees on preoperative CT scan. Hill and Norris (cited JBJS article) recommend grafting if the preop CT indicated that the glenoid component would penetrate the cortex of the glenoid neck after corrections in version are made.

In their study, Hill and Norris looked at the long-term results of internally fixed, corticocancellous bone graft in 21 shoulders undergoing TSA (17 followed up). Although they were successful in correcting glenoid version with the grafting procedure , the functional result at 6 years of follow up was excellent in 3 shoulders, satisfactory in 6 and unsatisfactory in 8 with 5/17 (29%) glenoid components failing and requiring revision. Better results from bone grafting for glenoid deficiency were reported by Steinmann and Cofield in JSES in their evaluation of 25 patients indicated for TSA. At an average of 5 years of follow up 13 shoulders were rated as excellent, 10 as satisfactory and 5 as unsatisfactory. Mean values for postoperative pain and post-operative ROM were similar to series of TSA without associated glenoid grafting. The last cited reference by Boyd et al, compared the results of total shoulder arthroplasty and hemiarthroplasty in a similar patient population. Hemiarthroplasty and total shoulder arthroplasty produced similar results in terms of functional improvement. Pain relief, range of motion, and patient satisfaction were better with total shoulder arthroplasty than hemiarthroplasty in the rheumatoid population. The conclusions were that total shoulder arthroplasty is recommended for patients with inflammatory arthropathies, and hemiarthroplasty is recommended for patients with osteoarthritis, avascular necrosis, and four-part fractures with preservation of glenoid congruity and absent synovitis.

PREFERRED RESPONSE: 3 RECOMMENDED READINGS: Steinmann and Cofield. Bone Grafting For Glenoid Deficiency in Total Shoulder Replacement. JSES. 2000. 9:361-367 Hill and Norris. Long-Term Results of Total Shoulder Arthroplasty Following Bone-Grafting of the Glenoid. JBJS. 2001. 83A: 877-883

154. A 27 year old man reports the insidious onset of low back pain. An AP radiograph is shown in Figure 57. Which of the following blood tests would best confirm his diagnosis? 1- Antinuclear antibody profile 2- Rheumatoid factor profile 3- Serum lead levels 4- Acid-fast bacillus test 5- HLA-B27

Figure 57

The question gives us a young man with low back pain and a corresponding AP view of the SI joints/lumbosacral spine demonstrating bilateral sacroiliitis. An insidious onset of pain and stiffness in the lower back are the typical leading symptoms of ankylosing spondylitis. Ankylosing spondylitis is an inflammatory disease of unknown etiology that affects primarily Caucasian males in the second through fourth decades of life. Genetic linkage to HLA-B27 has been established (Answer 5 as a confirmatory laboratory test). Ankylosing spondylitis primarily affects the axial skeleton and is characterized by inflammation and fusion of the sacroiliac joints, spine, and hips.
Some detail on Anklylosing Spondylitis (taken from Kubiak et al JAAOS 2005. 13: 267-278):

Ankylosing spondylosis (AS) is a seronegative inflammatory disease of unknown etiology characterized by inflammation in the axial skeleton. The sacroiliac joints are often involved initially, followed by enthesopathy in the paravertebral zygapophyseal joints and disk spaces, Enthesopathy is inflammation at the site of tendon insertions that predisposes to the development of fibrosis and calcification; left untreated, it leads to fusion of the zygapophyseal joints and intervertebral disk space. This fusion in turn can lead to a fixed, hyperkyphotic posture and compromised sagittal balance. Hip and knee flexion contractures occur when patients compensate for loss of sagittal balance by assuming a crouched position. Degeneration and ankylosis of peripheral joints, such as the hip, knee, and shoulder, may ensue. Deformity resulting from inflammation and fusion of the sacroiliac joints, spine, and hips leads to severe functional impairment in approximately 30% of AS patients.

Epidemiology: AS typically affects young adults, most commonly males (M:F = 3:1) in their second through fourth decades. The incidence of AS in North America is reported to be 1 to 2 per 1,000. Worldwide prevalence is up to 0.9%. Fifteen percent to 20% of patients with AS have a positive family history of the disease. AS is linked to HLA-B27; 80% to 95% of patients with AS are HLAB27positive. Carriers of this gene have a 16% to 50% increased risk of developing AS, but other genetic factors are likely involved as well. HLA-B27 is present among the inhabitants of Eurasia, North Africa, and North America; it is virtually absent among the aboriginal populations of Australia and South America. Although both HLAB27 negative and HLA-B27positive AS patients have similar articular manifestations, the former usually develop the disease at an older age and lack a positive family history. AS tends to occur at a later age in females than in males, and diagnosis is often delayed. Clinical History: Patients usually present with symptoms and physical findings consistent with the non-mechanical and inflammatory nature of AS. Onset is often insidious; patients typically cannot give the precise time of onset or even pinpoint the initially affected site. These individuals frequently present in the second through fourth decades of life with low back pain and stiffness, particularly in the morning, at night, or after prolonged periods of sitting and/or recumbency. Chest wall pain is a frequent complaint, particularly with deep inhalation. Back pain not relieved by recumbency and the persistent discomfort may compel the individual to leave the bed at night. Patients also may have buttock pain that radiates down the posterior thigh. Enthesopathy involving the Achilles and plantar tendon insertions with associated symptoms also is common. Some patients experience no pain but present with increasing stiffness in the hips and spine. As with other inflammatory diseases, symptoms often improve with exercise.

Laboratory Evaluation: Baseline laboratory investigation in patients suspected of having AS should include a basic metabolic panel, complete blood count, erythrocyte sedimentation rate (ESR), C-reactive protein (CRP) level, and rheumatoid factor. Although routine HLA-B27 status is not required to make a diagnosis of AS, it is helpful in the differential diagnosis. The clinician should be suspicious of a diagnosis of AS in patients who are HLA-B27negative. Patients in the active phase of the disease have mildly elevated ESR, CRP level, and white blood cell count. In most cases, rheumatoid factor is negative and HLA-B27 is positive. Imaging Studies: The sacroiliac joints usually are the first joints affected in patients with AS. Sacroiliitis initially presents as a widening of the sacroiliac joints with progressive sclerosis of the joint margins, leading to eventual bony fusion across the joints with subsequent loss of sclerosis. Care must be taken to disregard congenital deformities and degenerative changes as well as osteitis condensans ilii (symmetric sclerosis on the iliac sides of both sacroiliac joints without erosions, seen in women who have borne children). Standard anteroposterior radiographs of the pelvis are inadequate to fully evaluate the sacroiliac joints. For accurate diagnosis, a Ferguson view of the pelvis (anteroposterior with the x-ray tube tilted 30 cephalad) and oblique radiographs are required.

PREFERRED RESPONSE: 5 RECOMMENDED READINGS: Rudwaleit M et al. How to Diagnose Axial Spondyloarthriits Early. Ann Rheum Dis 2004; 63:535-543 Burgos-Vargas R, Clark P. Axial Involvement in the Seronegative Enthesopathy and Arthropathy Syndrome and its Progression to Ankylosing Spondylitis. J Rheumatol. 1989; 16: 192-197

155. Figures 58a and 58b show the radiographs of a 24 year-old professional baseball player who injured his foot sliding into home plate. Which of the following treatments offers the shortest time to union and the lowest risk for the development of a nonunion?

12345-

Open reduction and plate fixation Intramedullary screw fixation External fixation Short leg weight-bearing cast Short leg non-weight-bearing cast

A Jones fracture is a fracture of the 5th metatarsal at the metaphyseal-diaphyseal junction it is an intra-articular fracture with the fracture line involving the fourth-fifth intermetatarsal articulation. The injury is believed to occur when a large adduction force is applied to the forefoot with the ankle plantarflexed. Nonoperatively treated Jones fractures are often associated with a relatively high incidence of delayed union and nonunion (reported rates range from 7-28% in the literature). Clapper et al (CORR 1995; 315: 238-241) reported on 25 patients treated in a non-weight-bearing cast for 8 weeks followed by a weight-bearing cast until radiographic evidence of healing was present. Union occurred in 18 patients (72%) with casting alone, with an average time to union of 21.2 weeks. The other 7 patients had clinical and radiographic evidence of nonunion at 25 weeks and all underwent treatment with intramedullary screw fixation union was then achieved in all 7 cases at a mean of 12 weeks.

In the average patient, the recommended treatment of an acute, nondisplaced Jones fracture is non-weight-bearing short leg cast for 6-8 weeks, with advancement of weight bearing coinciding with radiographic evidence of healing. In patients with an acute, displaced Jones fracture, treatment should include early operative fixation by means of closed intramedullary screw fixation (4.5mm fully threaded lag screw).

In high performance athletes who require rapid return to play, early operative intervention with an intramedullary screw may be used leading to more rapid time to union and a lower incidence of nonunion. Quill et al (Orthop Clinic North America. 1995; 26:353-361), reported on 12 patients who were operated on acutely with IM fixation. This treatment resulted in 100% union at a mean of 7.4 weeks. In the other cited study from Porter et al, 23 consecutive athletes with Jones fractures treated surgically with a 4.5mm cannulated screw were followed. In this series 100% of the cases healed clinically with the mean return to sports occurring at 7.5 weeks.
PREFERRED RESPONSE: 2 RECOMMENDED READINGS: Porter DA, Duncan M, Meyer SJ: Fifth Metatarsal Jones Fracture Fixation with a 4.5mm Cannulated Stainless Steel Screw in the Competitive and Recreational Athlete: A Clinical and Radiographic Evaluation. AJSM. 2005; 33: 726-733 Kelly, IP et al. Intramedullary Screw Fixation of Jones Fractures. FAI. 2001; 22: 585-589 Rosenberg GA, Sferra JJ. Treatment Strategies for Acute Fractures and Nonunions of the Proximal Fifth Metatarsal. JAAOS. 2000; 8: 332-338

Jason Boyer

156. An 18-year-old woman has pain in her left shoulder after falling. At the emergency department, she was told that she sustained a contusion and was given a sling for comfort. Her pain has decreased over the past 2 weeks; however, she is unable to raise her arm above horizontal and externally rotate past neutral. Examination reveals that passive external rotation is only possible to neutral position. An AP radiograph is shown in Figure 59. In addition to obtaining additional radiographic views, what is the next most appropriate step in management? 12MRI to evaluate rotator cuff integrity Physical therapy that emphasizes stretching and range-ofmotion exercises 3- Electromyography and nerve conduction velocity studies to evaluate injury to the upper brachial plexus 4- Closed reduction of the left shoulder with immobilization in external rotation 5- Arthroscopy with labral and capsular repair

PREFERRED RESPONSE: 4 The first step in answering this question is to recognize that although you only have one film here, this is probably a posterior dislocation. In the xray, the humerus is internally rotated as manifest by the position of the tuberosities as well as the light bulb appearance of the humerus. The text tells you that both active and passive external rotation stops at neutral and that abduction is limited. All of these are suggestive of a posterior dislocation where the anterior surface of the humerus is locked against the posterior gleniod. This is the hallmark of posterior dislocation. From the xray again, there is no evidence of associated fracture. The Robinson article referenced splits these simple dislocations without fracture into acute (occuring within six weeks of presentation) and chronic (after 6 weeksare you following so far?). All acute dislocations should be closed reduced and then assesed for stability with internal rotation. By internally rotating the humerus after reduction, you are bringing the impression on the humeral head or reverse Hill-Sachs lesion in contact with the posterior lip of the glenoid again which is the position most likely to encourage dislocation. Therefore, immobilization should occur in external rotation or in the posistion the shoulder will be most stable. This is the famed gunslinger brace. Be aware that large impressions may need bone grafting. The OKU reference echoes the previous reference. RECOMMENDED READINGS: Robinson CM, Aderinto J: Posterior shoulder dislocations and fracture-dislocations. J Bone Joint Surg Am 2005;87:639-650. Norris TR (ed): Orthopaedic Knowledge Update: Shoulder and Elbow 2. Rosemont, IL, American Academy of Orthopaedic Surgeons, 2002, pp 71-81.

157.An 84-year-old woman has an intertrochanteric hip fracture. What is the best predictor of mortality risk?
12345American Society of Anesthesiologists (ASA) classification Patient age Fracture type Prefracture mobility Type of implant used

PREFERRED RESPONSE: 1 In this article from our home base, 836 of the 950 hip fractures (fem neck and intertroch of non pathologic origin) that were admitted to our very own HJD between July 1 1987 and Jan 1 1997 were retrospectively reviewed. Included were patients that were Caucasian, ambulatory, home dwelling and of reasonably sound mind. In this group of patients, mortality was highest in the first 3 months and then gradually decreased until 2 years after fracture. When this study cohort was stratified by age, the 65-84 year old group with ASA scores of 3 or 4 on presentation had a significantly higher mortality rate as compared to the general population of the same age. An ASA score of 1 or 2 did not show a significant difference in mortality. At age 85, however, the mortality rate was no different in the patients with hip fractures as compared to the general population of the same age regardless of ASA score. It just goes to show that if they make it to 85 on their own, there isnt much more we can do to them. Notice that the question specifies an 84 year old female and so ASA score applies to her. For those interested, the ASA score is determined as follows: 1 A normal healthy patient 2 A patient with mild systemic disease 3 A patient with severe systemic disease 4 A patient with severe systemic disease that is a constant threat to life 5 A moribund patient who is not expected to survive without the operation 6 A declared brain-dead patient whose organs are being removed for donor purposes

RECOMMENDED READING: Richmond J, Aharonoff GB, Zuckerman JD, et al: Mortality risk after hip fracture. J Orthop Trauma 2003;17:53-56.

158.When evaluating wound healing in patients with diabetes mellitus, calcification in the arteries can result in inaccurate readings in which of the following tests?
12Arteriography Transcutaneous oxygen pressure measurement 3- Waveform analysis 4- Bone scan 5- Doppler pressures

PREFERRED RESPONSE: 5 Questions about the diabetic foot and healing potential seem to pop up every year. When assessing the diabetic foot for healing potential, Doppler ultrasound is effective in assessing the adequacy of circulation. Current guidelines are that toe pressures should be at least 40 mm Hg, the ankle brachial index should be at least .45 and transcutaneous oxygen measurement should be above 30 mm Hg. Some authors also include protective sensation as a positive predictor for healing. This is defined as the ability to perceive a Semmes-Weinstein 5.07 monofilament. Because many diabetics have atherosclerosis and calcified vessels, the Doppler pressures can be falsely elevated. In the presence of a suspected high reading, the waveform analysis can help identify calcified vessels. A normal, elastic vessel with exhibit a triple phase signal whereas a stiff, calcified vessel will give a monophasic reading. Transcutaneous oxygen measurements are unaffected by calcified vessels. The OKU chapter is a good overview of treatment of diabetic ulcers and foot infections. RECOMMENDED READINGS: Richardson EG (ed): Orthopaedic Knowledge Update: Foot and Ankle 3. Rosemont, IL, American Academy of Orthopaedic Surgeons, 2004, pp 123-134. Wagner FW Jr: The dysvascular foot: A system for diagnosis and treatment. Foot Ankle 1981;2:64-122

What is the preferred method of treatment for the isolated injury shown in Figures 60a and 60b?
12345Sling and swathe for 6 weeks, followed by motion A sling for 2 weeks, followed by early motion Open reduction and internal fixation through a posterior approach Open reduction and internal fixation through an anterior approach Percutaneous screw fixation alone

PREFERRED RESPONSE: 2 Get out your fracture conference skills and read the xrays for me. Note that you are given 2 views that are similar to scapular y views but do a really good job of showing you a scapular neck fracture. There is no indication of another fracture and since we have no more views, we can infer that the clavicle and other structures of the shoulder suspensory complex are intact. Now you have to ask yourself how many scapula fractures you have ever seen fixed- answer very few. Classic indications for fixation of scapular neck fractures are angulation of the glenoid more than 40 degrees, open injuries and concomitant fractures of the neck and the clavicle (holy floating shoulder Batman). With an isolated neck fracture, even if the glenoid is extremely medialized, case series have shown that everyone does well. Now you know the treatment, so you have to ask yourself how long to immobilize the shoulder. Ask yourself- how many times have you seen a shoulder locked up for six weeks- answer never. Choice 2 becomes a brilliant deductive move. RECOMMENDED READINGS: Browner BD, Jupiter JB, Levine AM (eds): Skeletal Trauma. Philadelphia, PA, WB Saunders, 1992, pp 1291-1301. McGahan JP, Rab GT, Dublin A: Fractures of the scapula. J Trauma 1980;20:880-883. van Noort A, van Kampen A: Fractures of the scapula surgical neck: Outcome after conservative treatment in 13 cases. Arch Orthop Trauma Surg 2005;125:696-700.

160.Based on the CT scan shown in Figure 61, what is the typical mechanism of injury?
12345Supination-external rotation Pronation-abduction Pronation-adduction Plantar flexion-internal rotation Dorsiflexion-internal rotation

PREFERRED RESPONSE: 1 This question ia all about recognition of the image provided. A coronal cut of an ankle CT is given that clearly shows a Salter-Harris 3 fracture of the lateral aspect of the distal tibial epiphysis. Because the fibula is not present in this cut, we can assure ourselves that this is the anterolateral portion of the epiphysis and because we study and listen in our Peds fracture conferences, we know that a Salter-Harris 3 injury to the anterolateral aspect of the distal tibial epiphysis is called a Tillaux fracutre. These are rare fractures (3-5% of all pediatric ankle fracture) seen in children nearing skeletal maturity (usually 12-14 years old) during an 18 month window when the physis is closing. The anterolateral location results from the order that the physis closes- central first, then medial and lateral last. The fragment is avulsed off by the anterior inferior tibiofibular ligament and in some cases, can be assocoated with a distal fibula fracture as well. This is known as the dreaded floating anterior inferior tibiofibular ligament which I believe was studied by our own Kevin Kaplan but never caught on because it takes too long to say. The mechanism of injury is supination aand external rotation. Treatment is determined by the articular surface. For non displaced fractures, a long leg cast for four weeks followed by a short leg for 2 weeks is sufficient. For articular step off, the fragment needs to be reduced and fixed. Percutaneous screws are a lot of fun if you can stay out of both the joint and the physis. RECOMMENDED READINGS: Kay RM, Matthys GA: Pediatric ankle fractures: Evaluation and treatment. J Am Acad Orthop Surg 2001;9:268-278. Spiegel PG, Cooperman DR, Laros GS: Epiphyseal fractures of the distal ends of the tibia and fibula: A retrospective study of two hundred and thirty-seven cases in children. J Bone Joint Surg Am 1978;60:1046-1050.

Alex Golant

161
161. A 16-year-old girl has had vague right hip pain for the past 2 months. An AP radiograph of the pelvis and a T1-weighted fat suppression, gadolinium-enhanced MRI scan are shown in Figures 62a and 62b. A biopsy specimen is shown in Figure 62c. What characteristic genetic translocation is associated with this disease?

12345-

t(X;18) t(2;13) t(9;22) t(11;22) t(12;16)

161
First you have to know which translocations occur in which tumors (important ones highlighted): t(X;18) synovial sarcoma t(2;13) alveolar rhabdomyosarcoma t(9;22) extraskeletal myxoid chondrosarcoma t(11;22) Ewings (11+22=33 Patrick Ewings jersey #) t(12;16) myxoid liposarcoma a few other ones: t(12;22) clear cell sarcoma ring chromosome 17 dermatofibrosarcoma protuberans ring chromosome 12 well-differentiated liposarcoma

161
Now youve got to identify this tumor. A lesion in the pelvis is malignant until proven otherwise. Looking at the radiographs you cant see much (possibly a lucency in the right superomedial ilium). MRI shows destruction of the right iliac wing with a huge soft-tissue mass extending anteriorly and posteriorly. Histology shows many small round blue cells with mitotic figures and polymorphism. Info that gives away that this is Ewings: age (teens typical), location (pelvis), histology (small round blue cells). Ewings small round blue-cell tumor of children (>5yo) and young adults (<30yo). 7% of all primary bone malignancies. Most commonly occurs in pelvis, distal femur, prox tibia, femoral shaft, proximal humerus. Large lytic lesion in metaphysis and/or diaphysis, occasionally w/onion skinning, often w/abnormal labs (esr, wbc, anemia). Consistent chromosomal translocation (11:22). Treat with multi-modal approach: multi-agent chemo, XRT, surgery. Poor prognostic factors include spine/pelvic tumor, size >100cm3, <90% necrosis after chemo, elevated LDH. Differential Dx includes osteomyelitis, EG, osteosarc, PNET (peripheral neuroectodermal tumor) and neuroblastoma (in very young children).

161
PREFERRED RESPONSE: 4 RECOMMENDED READINGS: Douglass EC: Chromosomal rearrangements in Ewings sarcoma and peripheral neuroectodermal tumor (PNET). Semin Devel Biol 1990;1:393-396.
Dont bother.

Menendez LR (ed): Orthopaedic Knowledge Update: Musculoskeletal Tumors. Rosemont, IL, American Academy of Orthopaedic Surgeons, 2002, pp 11-20.
Read this one or review Big Jims lectures and handouts

162
162. If the posterior condylar axis is used for determining the rotation of the femoral component, which of the following errors is expected when 3 degrees of external rotation are built into the jig that controls femoral rotation? 12345Internal rotation of the femoral component if the lateral femoral condyle is hypoplastic Internal rotation of the femoral component in a varus knee Internal rotation of the femoral component when there is wear of the posterior aspect of the medial femoral condyle External rotation of the femoral component if the lateral femoral condyle is hypoplastic Femoral component rotation parallel to the epicondylar axis in all knees

162
This is a question of simple logic for us, boneheads. You want the femoral component in about 3 degrees of external rotation, which is whats built into this jig. The rotation can be measured from three different lines (see Figure 4-30, page 291 in Miller): 1) the line tangential to the posterior femoral condyles (posterior condylar axis), 2) the epicondylar axis, and 3) Whitesides line (AP axis of the femur, down the middle of the trochlea remember Lamonts TKAs?). If youre measuring from the posterior femoral condyles and the lateral condyle is hypoplastic posteriorly, your initial referencing line will be internally rotated, resulting in the relative internal rotation of the femoral component (Answer 1 correct, eliminate Answer 4). If the medial condyle is worn or hypoplastic, that will kick you into excessive external rotation of the femoral component (eliminate Answer 3). Varus/valgus should have no effect on rotation of the femoral component (eliminate answer 2), and the relationship of the component to the epicondylar axis will depend on the wear pattern of the posterior femoral condyles (eliminate Answer 5). PREFERRED RESPONSE: 1

162
The dos and the donts of TKA component placement (for improving patellar tracking) see Table 4-7, Page 290 in Miler: Do NOT internally rotate medialize internally rotate place button laterally place button inferiorly Do externally rotate about 3 deg center or lateralize center over lateral third of the tubercle medialize the button place button centrally or slightly superiorly

Femur Femur Tibia Patella

162
RECOMMENDED READING: Laskin RS: Instrumentation pitfalls: You just cant go on autopilot! J Arthroplasty 2003;18:18-22.
You may think RS Laskin is our dear ol Ralph, but hes not. Hes some dude from HSS. This is actually a pretty good article, definitely worth a read. Specifically with regards to femoral component rotation the author states that posterior condylar line is generally believed NOT to represent a neutral mediolateral axis and on average is 3 deg internally rotated to either the transepicondylar or midtrochlear [whitesides] lines. He also goes on to say that in valgus knees with lateral condyle dysplasia, aligning the guide with posterior condyles and externally rotating by 3 degrees will result in interally rotated femoral component, whereas in severe varus knees w/medial condyle wear, femoral component ends up being too externally rotated (as discussed above). Then you treat it with sugar.

163
163. Figure 63 shows the radiograph of a patient who has medial knee pain. What is the most likely cause of the pain? 12345Iatrogenic anterior cruciate ligament injury Iatrogenic posterior cruciate ligament injury Medial collateral ligament injury Nail is too distal Distal interlock is too long

163
Look at the xray and think for a minute. Identify Blumensaats line, which corresponds to the top of the intercondylar notch on the lateral view. We can see that the nail is buried proximal to that line, and therefore is not sticking out into the notch. This eliminates Answer 4 (the nail is NOT too distal). The starting point of the nail in the A-P direction (on the lateral view) is also appropriate thus decreasing the chance of ACL/PCL injury, eliminating Answers 1 and 2. Theres absolutely no radiographic or clinical info to support Answer 3 (MCL injury) they would give you more to work with, like laxity w/valgus stress, etc. This leaves us with Answer 5 (distal interlock too long). Looking at the AP view, the distal screw is noted to be too long and is probably tickling medial knee structures the way Anwar would love to tickle Dr. Ryan if he got him alone for a few minutes.
PREFERRED RESPONSE: 5

163
RECOMMENDED READINGS: Ostrum RF, Agarwal A, Lakatos R, et al: Prospective comparison of retrograde and antegrade femoral intramedullary nailing. J Orthop Trauma 2000;14:496-501.
Prospective, randomized study, 44 retrograde vs 46 antegrade IMNs for femoral shaft fxs. Both had very high union rates (100% A, 98% R), same amount of knee pain, same ROM. Antegrades healed faster by average of 4wks, had more hip/thigh pain. Retrogrades had more symptomatic distal locking screws (R= 18 vs A=4), 7 of which required ROH.

Ricci WM, Bellabarba C, Evanoff B, et al: Retrograde versus antegrade nailing of femoral shaft fractures. J Orthop Trauma 2001;15:161-169.
Restrospective study, 104 retrograde vs 94 antegrade IMNs for femoral shaft fxs. Similar union (88%, 89%) and malunion (11%, 13%) rates. Higher incidence of knee pain in retrograde group (36% vs 9%), higher incidence of hip pain in antegrade group (10% vs 4%) duh! Interestingly, the rate of distal interlocking screw removal for pain were similar between two groups (5-6%)

164
164. According to workers compensation law, when can a patient with a fracture settle his or her claim? 12345Following 1 year postinjury Following release to light duty Following release to full duty Following maximal improvement Upon fracture healing

164
I couldnt care less. But for the sake of answering the fing question, here we go. Another one for pure logical reasoning. No specific time period makes sense, so throw out Answer 1 (1 year post-injury) as we all know patients may could be take years to heal and recover from injuries. Similarly Answers 2 and 3 dont work, because patients may never get back to full, or even light, duty. The ultimate goal of our treatment of orthopaedic injuries is recovery of maximal function, or as they word it here maximal improvement, which is the time when legal claims pertaining to the injury can be settled, Answer 4. Healing of the fracture does not necessarily imply maximal functional recovery, so Answer 5 is incorrect. PREFERRED RESPONSE: 4

164
RECOMMENDED READINGS: Chaise F, Bellemere P, Fril JP, et al: Return-to-work interval and surgery for carpal tunnel syndrome: Results of a prospective series of 233 patients. J Hand Surg Br 2004;29:568-570.
Doesnt help answer our question, but shows that at least in Great Britain civil servants and patients on workers comp take way more time off work after carpal tunnel surgery (25-40 days more!) than patients who are covered by normal insurance. Yeah, I think we have the same problem here in the good ol US of A.

Davis PM, Badii M, Yassi A: Preventing disability from occupational musculoskeletal injuries in an urban, acute and tertiary care hospital: Results from a prevention and early active return-to-work safely program. J Occup Environ Med 2004;46:1253-1262.
What a waste of paper it was to print out this article. Thanks OITE writers, this was a completely useless reference.

165
165. During revision total knee arthroplasty for flexion instability, the tibial baseplate is retained. Compared to the original femoral component, the revision femoral component should be

12345-

smaller. larger. more distal. lateralized. at the same position and size, with a thicker tibial insert.

165
Identify whats given first: flexion instability, tibial component stays as is. The question is what to do with the femoral component. The answer is straight out of Miller simplest thing to do is increase the size (which is manifested as a larger A-P dimension) of the femoral component, Answer 2 is correct. (You may have to augment posterior condyles on the component to account for bone loss posteriorly, or just fill w/cement). Eliminate other answers as follows: 1 smaller component would worsen the flexion gap, increasing instability, 3 more distal femoral component would tighten the extension gap but do nothing to the flexion gap, 4 lateralized femoral component would have no effect on the flexion gap (this is more important for Q-angle optimization and patellar tracking), 5 thicker poly will tighten both the flexion and extension gaps; since only the flexion gap is loose here, this answer is incorrect

PREFERRED RESPONSE: 2 RECOMMENDED READING: Ries MD, Haas SB, Windsor RE: Softtissue balance in revision total knee arthroplasty: Surgical technique. J Bone Joint Surg Am 2004;86:81-86.

165
Here is a simplified approach to balancing flexion/extension gaps in TKA:

Gap is Symmetric Symmetric Assymetric

Problem is tight flex/ex loose flex/ext loose flexion tight flexion loose extension tight extension

Concentrate on tibia tibia posterior femur posterior femur distal femur distal femur

What to do cut more tibia larger poly insert increase femur size decrease femur size augment distal femur cut off more distally

What else can limit flexion? osteophytes What else can limit extension?

PCL, not enough tibial slope, posterior Tight posterior capsule

Also, remember that asymmetric gaps can be re-cut and converted to symmetric gaps, which then enables you to alter the size of the poly insert. All of this is well-summarized in Miller (Table 4-6, page 287) and also in the excellent (for a change) reference article by Ries et al, JBJS 2004.

Ilya Iofin

166. What anatomic region or landmark is best seen on the iliac oblique radiograph of the pelvis? 1- Posterior column 2- Sacroiliac joint 3- Iliopectineal line 4- Profile of the obturator foramen 5- Posterior wall of the acetabulum

Judet radiographs are obtained by bumping up the patient to rotate him 45 degrees. The side closest to the plate will provide the iliac oblique view and the contralateral side will be the obturator oblique view. The iliac oblique view shows best the anterior wall and posterior column. The obturator oblique view will show the other two structures, i.e. posterior wall and anterior column. Take a model of the pelvis and play with it to understand the anatomy. To answer the question during the exam you have two options. One is to rotate an imaginary model of the pelvis in your mind and derive the answer. If the imaginary model of the pelvis is too bulky to fit inside your mind, then just memorize a mnemonic. An example is IOWAiliac oblique wall anterior. Once you know one structure seen on one view, the other three are easily derived. Its eaasssy, its not hard labor. OK, conyo.

PREFERRED RESPONSE: 1 RECOMMENDED READING: Mayo KA: Fractures of the acetabulum. Orthop Clin North Am 1987;18:43-57.

167.A 2-year-old girl has a mass and erythema in her right hand. She has no recent history of fever, chills, or sore throat. Examination reveals no lymphadenopathy. Figures 64a through 64f show a radiograph, MRI scans, clinical photograph, and a biopsy specimen. Treatment should consist of 12345chemotherapy and wide resection. ray amputation. curettage and allogenic bone grafting. curettage and antimicrobial treatment. bone marrow injection.

The radiographs show an expansile geographic lytic lesion in the index metacarpal. On MRI, the lesion is bright on T2 and dark on T1, revealing a high water content. There is also a soft tissue extension seen on MR. The biopsy specimen shows a highly cellular infiltrate with a lot of very angry looking blue cells. I am sure I am not the only one who wanted to chop this poor girls finger off and do even worse things to her like giving her chemo. But I am the only guy who runs the risk of actually doing such savagery in my practice. The clue to the correct answer is the low power pathology slide which shows a granuloma. A granuloma is a group of epithelioid macrophages surrounded by a lymphocyte cuff. A caseating (translated as turning to cheese) granuloma is one that has central necrosis. These are characteristic of TB. I do not claim to be a cheese expert, but the granuloma shown is not caseating. Be it as it may, the girl has TB and the treatment is curettage and antimicrobial treatment. Those of you who do not have the JBJS issues from 1979 and 1972 should really devote your post call days to accumulating a good orthopaedic library. And not having been born at the time a case report was written is no excuse for not having read it. I read War and Peace, and it was written even before my grandparents were born. PREFERRED RESPONSE: 4 RECOMMENDED READINGS: Manzella JP, Vanvoris LP, Hruska JF: Isolated calcaneal tuberculous osteomyelitis: A case report. J Bone Joint Surg Am 1979;61:946-947. Bailey HL, Gabriel M, Hodgson AR, et al: Tuberculosis of the spine in children: Operative findings and results in one hundred consecutive patients treated by removal of the lesion and anterior grafting. J Bone Joint Surg Am 1972;54:1633-1657.

168.What is the most likely complication following open or endoscopic anterior surgery of the thoracic spine? 12345Pseudarthrosis Intercostal neuralgia Tension pneumothorax Aortic laceration Parenchymal lung injury

According to the Spine Journal article describing the thoracoscopic approach for treatment of fractures, access-related complications such as pleural effusion, pneumothorax and intercostals neuralgia were seen in 12 patients (5.7%). This is the most common complication, though there is no separation in incidence of the3 complications listed above. Of the 15 patients included in the second cited article, 3 had atelectasis and 2 had intercostals neuralgia and the other complications were experienced by one patient each. I did not major in math, but 3>2, but then again, atelectasis is not a possible answer. I guess the take home point of this question is that it hurts to have your ribs spread open or to have cameras and instruments shoved between your ribs. Who would have thunk that.

PREFERRED RESPONSE: 2 RECOMMENDED READINGS: Kim DH, Jahng TA, Balabhadra RS, et al: Thoracoscopic transdiaphragmatic approach to thoracolumbar junction fractures. Spine J 2004;4:317-328. Dickman CA, Rosenthal D, Regan JJ: Reoperation for herniated thoracic discs. J Neurosurg 1999;91:157-162.

169.A 15-month-old boy requires treatment with a halo device. What is the preferred pin configuration? 12345Four pins, with a torque of 6 to 8 in-lb Four pins, with a torque of 2 to 4 ft-lb Four pins, with a torque of 6 to 8 ft-lb Six to twelve pins, with a torque of 2 to 4 in-lb Six to twelve pins, with a torque of 2 to 4 ft-lb

The thing to remember about little kiddos is that they have thinner skulls than older kiddos (I am not even talking about the ones destined to be orthopaedic surgeons). That means that more pins need to be used to distribute the force more evenly and not penetrate the inner table of the skull and, God forbid, the dura. Therefore choices 4 and 5 are the two viable options. As far as torque, torque is the product of the force applied times the length of the lever arm perpendicular to the vector of the force application. So imagine a wrench of a certain length with a weight hanging off of it. A 4 inch wrench with a one pound weight hanging off of it used to tighten the pins sounds reasonable. A 4 foot long wrench with a one pound weight on it will result in a torque that will drive the pin through the cranium. I am not a brain surgeon, but I think thats bad. For these spine/ex-fix (so to speak) patients, infection, not surprisingly, was the most frequent complications reaching rates of about 20-60%. But all reasoning aside, the bottom line is that this is a memorization question. You either know how to put on a halo vest or you dont. But dont feel bad. I also dont know how to put on a halo vest even though I was assigned this question. PREFERRED RESPONSE: 4 RECOMMENDED READINGS: Vaccaro AR (ed): Orthopaedic Knowledge Update 8. Rosemont, IL, American Academy of Orthopaedic Surgeons, 2005, pp 767-774. Dormans JP, Criscitiello AA, Drummond DS, et al: Complications in children managed with immobilization in a halo vest. J Bone Joint Surg Am 1995;77:13701373.

170.To decrease mobidity and mortality in an elderly ambulatory patient with a displaced femoral neck fracture, optimal management should include 1234surgical intervention within 4 days of injury. open reduction and internal fixation. a delay in surgery of 7 days. early surgery irrespective of medical comorbidities. 5- transfer to a level 1 trauma center.

The first paper cited states that mortality increases if surgery is delayed by more than 4 days. No statistically significant difference was detected in people who had their fractures fixed on the first day or in the first 4 days. Waiting more than 4 days significantly increases mortality. The paper is out of Great Britain which has socialized medicine and where the government tells you when you can fix a hip fracture. Most of the delays were due to logistics, not medical comorbidities. Having medical comorbidities, not surprisingly, increases your chances of death. Overall, the chances of dying within a year of a hip fracture are about 30%. The second paper states that total hips are the best options for older patient with displaced femoral neck fractures. Thus, answer 2 is definitely wrong. As far as answer 3, dont believe the medical consult at Bellevue who doesnt want to bother with the patients medical clearance. Delaying surgery and being bed bound for days is not good for patients. Neither is operating on very sick patients, as answer 4 suggests. Transferring to a level 1 trauma center is a bit of overkill for grandma. The bottom line is, operate on hip fractures as soon as they are medically optimized. PREFERRED RESPONSE: 1 RECOMMENDED READINGS: Moran CG, Wenn RT, Sikand M, et al: Early mortality after hip fracture: Is delay before surgery important? J Bone Joint Surg Am 2005;87:483-489. Healy WL, Iorio R: Total hip arthroplasty: Optimal treatment for displaced femoral neck fractures in elderly patients. Clin Orthop 2004;429:43-48.

Gregg Jarit

171
171. During revision total knee arthroplasty with the trial components in place, the knee is stable in flexion but a 10-degree flexion contracture is present with the thinnest polyethylene liner. Treatment should consist of which of the following surgical techniques?

12345-

Increase the tibial posterior slope Use a larger femoral component Use a smaller femoral component Resection of additional distal femur Posterior displacement of the femoral component

171
PREFERRED RESPONSE: 4 Soft tissue balancing in TKA is tested every year. In this case the knee is stable in flexion but tight in extension (10 degree flexion contracture). So our treatment needs to change the extension gap without changing the flexion gap. To increase the extension gap you can either resect more distal femur (answer 4), resect more proximal tibia, or use a smaller femoral component (answer 3). However, if you resect more proximal tibia, you will also increase the flexion gap, making the knee loose in flexion. Downsizing the femoral component is the equivalent of resecting more of the posterior femoral condyles, which will also increase the flexion gap. If you imagine the knee in a flexed position, you can see that resecting more distal femur will not affect the flexion gap. So the way to increase extension gap without changing the flexion gap is to resect more distal femur.

RECOMMENDED READINGS: Ries MD, Haas SB, Windsor RE: Soft-tissue balance in revision total knee arthroplasty: Surgical technique. J Bone Joint Surg Am 2004;86:S81-S86. Baldini A, Scuderi GR, Aglietti P, et al: Flexion-extension gap changes during total knee arthroplasty: Effect of posterior cruciate ligament and posterior osteophytes removal. J Knee Surg 2004;17:69-72.

172
172. What is the proper order for screw insertion to obtain fracture compression and lag screw fixation through the dynamic compression plate shown in Figure 65? 12345B, E, C C, D, A C, B, E D, B, C D, A, C

172
PREFERRED RESPONSE: 5 Principles of internal fixation. For application of a compression plate, after fracture reduction, the plate is fixed to one of the fragments (D). Tension is then applied to the opposite side of the plate (A), creating compression at the fracture site. You can see that placing the second screw at A eccentrically in that hole will pull the fragment on the right side towards it. With this technique it is helpful to initially affix the plate to the fragment that creates an axilla between the plate and the screw (in this case the right side). If you fix to the left side first, then as you tighten the second screw, you can get over reduction as the fragments slide over each other. After these two screws are placed, you can then gain additional compression at the fracture site by placing a lag screw through the plate. Last point: DCP = dynamic compression plate. LCDCP = limited contact dynamic compression plate. So you can see that saying DCP plate or LCDCP plate makes you sound stupid. RECOMMENDED READINGS: Browner BD, Jupiter JB, Levine AM, et al: Skeletal Trauma. Philadelphia, PA, WB Saunders, 1992, pp 247-252. Chapman MW (ed): Chapmans Orthopaedic Surgery, ed 3. Philadelphia, PA, Lippincott Williams & Wilkins, 2001, pp 348-351.

173
173. A 19-year-old basketball player sustains a fracture to the fifth metatarsal that is treated with screw fixation. At his 6-week postoperative visit, he is asymptomatic and has no pain with running or jumping. A radiograph is shown in Figure 66. Which of the following is directly related to the risk of potential treatment failure? 12345Use of a bone stimulator postoperatively Return to full activity prior to radiographic union Patients age Time from injury to surgical fixation Size of screw used

173
PREFERRED RESPONSE: 2
In the Larson study they looked at 15 patients who underwent cannulated screw fixation of a Jones fracture. Return to full activity, especially among elite athletes, before radiographic union was predictive of failure. Only 1/6 (17%) patients with failure showed radiographic union. There were no significant differences in age, sex, screw diameter, use of bone graft, or age of fracture. The Glasgow study also showed that early return before radiographic union correlated with failure. There was no mention of using bone graft immediately after surgical fixation of these fractures (answer 1). The lesson to be learned from this question is that just because the magic 6 weeks has passed doesnt mean patients should return to full activity. Its why we check x-rays at this time. RECOMMENDED READINGS: Larson CM, Almekinders LC, Taft TN, et al: Intramedullary screw fixation of Jones fractures: Analysis of failures. Am J Sports Med 2002;30:55-60. Glasgow MT, Naranjia RJ Jr, Glasgow SG, et al: Analysis of failed surgical management of fractures of the base of the fifth metatarsal distal to the tuberosity: The Jones fracture. Foot Ankle Int 1996;17:449-457.

174
174. Figure 67a shows the radiograph of a 72-year-old man who has a periprosthetic fracture nonunion that was treated with revision total hip arthroplasty using segmental structural femoral and acetabular allografts. A postoperative radiograph is shown in Figure 67b. Five months after surgery, failure of the reconstruction occurred. A current radiograph is shown in Figure 67c. What is the most likely cause of the failure of the revision procedure? 12345Allograft failure Osteoporosis Pelvic discontinuity Infection Inadequate allograft fixation

174
PREFERRED RESPONSE: 1 In this case a reconstruction ring was used. This is a large metallic cage that gains purchase via multiple screws and flanges in the ilium and ischium. They are used for more extensive contained bone defects, as well as uncontained defects involving loss of cancellous and cortical bone of the acetabular columns and dome. The purpose of these rings is to replenish lost bone stock for future revisions, place the acetabulum at the correct anatomic position, and allow progressive weight bearing earlier, because the ring is attached directly to the pelvis. Having said all that, the reference didnt particularly help much with this answer. They state in the article that defects larger than 40% that require allografts often fail, but this has nothing to do with their data. Looking at the answers, we can eliminate osteoporosis because the problem here is bone loss from osteolysis. Regardless, osteoporosis is not a common reason for failure of THAs. Answer 3, pelvic discontinuity is incorrect because these reconstruction rings are used specifically for that problem. There is no evidence given in the question to suspect infection, making answer 4 incorrect. And answer 5, inadequate allograft fixation, is incorrect because there is a ton of fixation there. In figure 67c, you can see the support for the cage, which was the allograft, has failed. RECOMMENDED READING: Goodman S, Saastamoinen H, Shasha N, et al: Complications of ilioischial reconstruction rings in revision total hip arthroplasty. J Arthroplasty 2004;19:436-446.

175
175. A 56-year-old man has had pain in the hypothenar region of his dominant right hand for the past 6 months. He reports weakness in his grip and pain that is worst with activity. Which of the following findings is most suggestive of a cervical etiology? 1- Relief of symptoms with shoulder abduction (placing hand over the head) 2- Hypothenar atrophy 3- Reproduction of pain with lateral neck flexion and contralateral rotation of the head 4- Positive Tinels sign at the levator scapulae 5- Subluxation of the ulnar nerve at the cubital tunnel

175
PREFERRED RESPONSE: 1
Davidson studied 22 patients who required myelography for unremitting radicular pain and found that 15 of them had relief of pain with shoulder abduction. This relief with abduction is a sign of extradural root compression. The proposed etiology of this is decreased root tension. Hypothenar atrophy (answer 2) can result for compression of the ulnar nerve anywhere along its course. Answer 3 is trying to fool you into thinking that this is the Spurlings sign, however Spurlings sign is described as neck extension or hyperextension with lateral rotation. The maneuver in answer 3 is not associated with reproduction or worsening of radicular pain. A Tinels sign at the levator scapulae (answer 4) would be indicative of peripheral nerve compression, in this case at the brachial plexus. Subluxation of the ulnar nerve at the cubital tunnel (answer 5) can cause grip weakness and pain independently and does not indicate cervical pathology. RECOMMENDED READINGS: Hoppenfeld S: Physical Examination of the Spine and Extremities. New York, NY, Appleton-Century Crofts, 1976. Davidson RI, Dunn EJ, Metzmaker JN: The shoulder abduction test in the diagnosis of radicular pain in cervical extradural compressive monoradiculopathies. Spine 1981;6:441-446.

Kevin Kaplan

176. Compared to harvesting of cancellous bone from the outer table of the anterior pelvis, retrieval from the posterior pelvis is associated with

12345-

a similar overall complication rate. greater blood loss. greater duration of pain. greater level of pain. less mean graft volume.

PREFERRED RESPONSE: 1
Autologous bone graft is advantageous because of both its osteoinductive and osteoconductive properties, in addition to it being nonimmunogenic. The most common site of harvest is the iliac crest because of its accessibility from any patient position. The authors retrospectively reviews 414 consecutive cases of iliac crest bone graft and reviewed them for minor and major complications associated with the harvesting.

Contd
Anterior retrieval was associated with herniation of the small bowel in the defect, injury to the lateral femoral cutaneous nerve, chronic pain and fracture of the iliac wing. Posterior retrieval was associated with vascular injury to the superior gluteal artery, chronic pain, and injury to the cluneal nerves. However, the overall complication rate between the two approaches remains similar. There is no mention in the article of greater blood loss, greater duration/level of pain, or graft volume obtained.

Contd
RECOMMENDED READINGS: Arrington ED, Smith WJ, Chambers HG, et al: Complications of iliac crest bone graft harvesting. Clin Orthop 1996;329:300-309. Ahlmann E, Patzakis M, Roidis N, et al: Comparison of anterior and posterior iliac crest bone grafts in terms of harvest-site morbidity and functional outcomes. J Bone Joint Surg Am 2002;84:716-720.

177. With a surgical approach to the pes insertion for harvest of the semitendinosus and gracilis tendons, what nerve is at risk as it emerges between the sartorius and gracilis tendons? 1- Common peroneal 2- Superficial peroneal 3- Medial femoral cutaneous 4- Terminal branch of the saphenous 5- Medial retinacular

Preferred response: 4
The terminal branch of the saphenous nerve emerges between the sartorious and gracilis tendons and lies superficial to the gracilis tendon at the PM joint line. This nerve travels down the anteromedial surface of the leg and may be encountered during harvest of the hamstring tendons close to the pes insertion.

Contd
The common peroneal and superficial peroneal nerves are on the other side of the knee. The medial femoral cutaneous nerve is a terminal branch of the femoral nerve which supplies the skin of the distal thigh and patella and should not be damaged during the dissection. The medial retinacular nerve is a terminal branch of the nerve to the vastus medialis and should not be encountered during the dissection.

Contd
RECOMMENDED READINGS: Hoppenfeld S, deBoer P: The knee, in Hoppenfeld S, deBoer P (eds): Surgical Exposures in Orthopaedics: The Anatomic Approach, ed 2. Philadelphia, PA, JB Lippincott, 1994, pp 430-482. Medvecky MJ, Noyes FR: Surgical approaches to the posteromedial and posterolateral aspects of the knee. J Am Acad Orthop Surg 2005;13:121128.

178. Which of the following bone densitometry values is indicative of osteoporosis?

12345-

T-score of +2.5 T-score of -1.0 T-score of -1.5 T-score of -2.5 Z-score of -1.0

Preferred Response: 4 The World Health organization defined certain categories based on bone density: Normal T score better than -1 Osteopenia: T score b/t -1 and -2.5 Osteoperosis T score less than -2.5 Established osteoporosis non-traumatic fractures
RECOMMENDED READING: Kanis JA, Melton LJ III, Christiansen C, et al: The diagnosis of osteoporosis. J Bone Miner Res 1994;9:1137-1141.

179. The structure at the tip of the arrow shown in Figure 68 is attached to a muscle that is innervated by which of the following nerves? 12345Axillary Subscapular Suprascapular Pectoral Musculocutaneous

Preferred Response: 2
In the picture, you see a right proximal humerus fracture with the arrow pointing to the lesser tuberostiy. The subscapularis attaches to the tuberosity and acts as an internal rotator. It is innervated by the upper and lower subscapular nerves. The axillary nerve supplies the teres minor and deltoid. The suprascapular nerve supplies the surpaspinatus, infraspinatus and shoulder joint. The Pectoral nerves supply the pectoralis major and minor

The Musculocutaneous nerve supplies the coracobrachialis, biceps brachii, and brachialis.

Contd
RECOMMENDED READINGS: Greis PE, Kuhn JE, Schultheis J, et al: Validation of the lift-off test and analysis of subscapularis activity during maximal internal rotation. Am J Sports Med 1996;24:589-593. Decker MJ, Tokish JM, Ellis HB, et al: Subscapularis muscle activity during selected rehabilitation exercises. Am J Sports Med 2003;31:126-134.

180. An 82-year-old man fell and sustained the fracture shown in Figures 69a and 69b. History reveals that the patient had undergone a hemiarthroplasty and reported postoperative groin pain after activity. Treatment should now consist of
1- allograft strut with cerclage cables. 2- combined plate and allograft strut with cerclage cables. 3- cemented bipolar revision. 4- cemented revision total hip arthroplasty. 5- cementless revision total hip arthroplasty.

Preferred Response: 5
In this cross table lateral of the femur, you can see an obvious fracture with disruption of the prosthesis bone interface. The most important characteristic to consider when determining the appropriate type of treatment for periprosthetic fractures is the stablitity of the prosthesis-bone interfrace. Based on the Duncan-Vancouver and Beals-Tower Oregon classifications, you can make a decision regarding appropriate treatment.

Contd
The referenced article is a prospective series of periprosthetic femur fractures in 33 patients. Fractures in which the prosthesis-bone interface was stable were treated with ORIF around the stable implant. Unstable fractures such as this one required removal of the primary prosthesis and revision to an uncemented long-stem prosthesis after stabilization of the femoral fracture with plates or allograft struts. The authors demonstrated low complication rates and restoration of function in a predicable manner when treating patients with this algorithm.

Contd
RECOMMENDED READING: Duwelius PJ, Schmidt AH, Kyle RF, et al: A prospective, modernized treatment protocol for periprosthetic femur fractures. Orthop Clin North Am 2004;35:485-492.

Paul Lafferty

181. A 42-year-old carpenter has had pain and has been unable to lift his dominant arm overhead for the past few months. Examination reveals atrophy of the supraspinatus fossa with grade 2/5 function of the supraspinatus. Radiographs are normal. A sagittal T1-weighted MRI scan is shown in Figure 70. Management consisting of physical therapy and two cortisone injections has failed to provide relief. Treatment should now consist of 12345implantation of an oversize humeral head component. glenohumeral arthrodesis. reverse total shoulder arthroplasty. bipolar shoulder arthroplasty. latissimus dorsi transfer.

PREFERRED RESPONSE: 5
Transfer of the latissimus dorsi muscle is a reasonable approach for salvage after failed operative treatment of a massive tear of the rotator cuff. However, salvage reconstruction of failed prior rotator cuff repairs yields more limited gains in satisfaction and function than primary latissimus dorsi transfer. RECOMMENDED READINGS: Warner JJ, Parsons IM IV: Latissimus dorsi tendon transfer: A comparative analysis of primary and salvage reconstruction of massive, irreparable rotator cuff tears. J Shoulder Elbow Surg 2001;10:514-521. Miniaci A, MacLeod M: Transfer of the latissimus dorsi muscle after failed repair of a massive tear of the rotator cuff: A two- to five-year review. J Bone Joint Surg Am 1999;81:1120-1127

182. Which of the following biologic factors or molecules is necessary for normal osteoblastic differentiation? 12345Tumor necrosis factor-alpha (TNF-alpha) Core binding factor alpha-1 (cbfa-1: RUNX2) Receptor activator of nuclear factor kappa beta ligand (RANKL) Platelet-derived growth factor (PDGF) Insulin-like growth factor-I (IGF-I)

PREFERRED RESPONSE: 2 Osteoblast differentiation and bone formation is stimulated by bone morphogenetic protein (BMP)-2 and its downstream signaling molecules Smad1 and -5 and the osteoblast-specific transcription factor core-binding factor alpha1 (Cbfa1). Core-binding factor alpha 1 (Cbfa1) is considered a prerequisite transcription factor for osteoblastic differentiation. RECOMMENDED READINGS: Ballock RT, OKeefe RJ: The biology of the growth plate. J Bone Joint Surg Am 2003;85:715-726. Ducy P, Zhang R, Geoffroy V, et al: Osf2/Cbfa1: A transcriptional activator of osteoblast differentiation. Cell 1997;89:747-754.

183. What is the most common endocrinopathy seen in association with slipped capital femoral epiphysis? 12345Panhypopituitarism Hypogonadism Hypothyroidism Hyperparathyroidism Pseudohypoparathyroidism

PREFERRED RESPONSE: 3 SCFE develops during puberty, a time of many hormonal changes. Rapid longitudinal growth occurs in response to increased levels of growth hormone. This rapid growth is associated with an increased chondrocyte proliferation rate and increased height of the zone of hypertrophy. This increased height may contribute to the decreased physeal strength seen at puberty. The effects of gonadotropins may explain the male predominance in SCFE; estrogen reduces physeal height and increases physeal strength, whereas testosterone reduces physeal strength. Although most adolescents with SCFE do not have a demonstrable endocrine disorder, a subtle, but yet undiagnosable endocrinopathy may be present. The increased incidence of SCFE in children with hypothyroidism, as well as in children undergoing growth hormone supplementation and in hypogonadal states, suggests an association between SCFE and endocrine dysfunction. RECOMMENDED READINGS: Morrissy RT (ed): General Considerations: Slipped Capital Femoral Epiphysis. Rosemont, IL, American Academy of Orthopaedic Surgeons, 2002, pp 1-18. Loder RT, Wittenberg B, DeSilva G: Slipped capital femoral epiphysis associated with endocrine disorders. J Pediatr Orthop 1995;15:349-356

184. A 56-year-old man with diabetes mellitus reports the sudden onset of hip pain with weight bearing for the past day. History reveals that he underwent an uncomplicated total hip arthroplasty 4 years ago and underwent dental work 1 week ago. Laboratory studies show an erythrocyte sedimentation rate of 95 mm/h (normal up to 20 mm/h) and a C-reactive protein of 32 mg/mL (normal value is < 10). Hip aspiration revealed a cell count of 25 x 103, with a differential of 70% polymorphonuclear leukocytes. Management should consist of 1- awaiting the results of 5-day cultures, followed by IV antibiotics if cultures are positive. 2- awaiting the results of 5-day cultures, followed by irrigation and debridement and 6 weeks of IV antibiotics if cultures are positive. 3- immediate one-stage exchange arthroplasty. 4- immediate two-stage exchange arthroplasty. 5- immediate irrigation, debridement, and femoral head and liner exchange.

PREFERRED RESPONSE: 5 There is little argument about the necessity to remove a loose total hip prosthesis from a chronically infected joint. However, removal of a well fixed total hip implant that is associated with infection carries the risk of causing major damage to the remaining bone stock. It is therefore understandable that attempts have been made to define the circumstances in which the infection may be eradicated by dbridement combined with systemic administration of antibiotics without removal of the prosthesis. It is generally accepted that the results of dbridement with retention of the prosthesis in patients who have a chronic infection are poor. This is in keeping with what we understand about the ability of infecting organisms to adhere to the surfaces of the implant and to survive within a slime layer that isolates the organism from host defense mechanisms and the effects of systemic antibiotics. However, we also know that the slime layer takes some time to form after inoculation and that there is a potential so-called window of opportunity while the infecting organism is still in its planktonic form. If the infection is treated intensively with adequate dbridement and appropriate systemic antibiotics, eradication should be possible. Both postoperative and late hematogenous infections can present acutely, and it appears reasonable to treat both types of infection in this manner shortly after onset. Difficulties with this approach revolve around the determination of the time of onset of the infection and the establishment of a cutoff time beyond which it is no longer reasonable to attempt to retain the implant. Additional difficulty arises from the inability to recognize dead infected tissues that will act as an ongoing nidus of infection. Diagnosis of late hematogenous infections poses little difficulty. The patient will frequenly have had a recent surgical teratment, dental manipulation, or remote infection. The infectious process will be heralded by the classic symptoms, including the acute onset of pain with febrile response. Laboratory evaluation will reveal elevations of ESR and C-reative protein, as well as elevation of the WBC count with a shift to the left. Purulent material can be obtained by means of aspiration. Dental procedures may lead to a transient bacteraemia lasting for up to 30 minutes. Of the numerous cases of total hip arthroplasty (THA) reported which have been infected from haematogenous sources, dental procedures have been involved only infrequently. However, Infection of a THA after dental procedures is more common than has been previously suspected. Patients with systemic disease, or who are undergoing extensive procedures, should be considered for prophylactic antibiotic treatment. RECOMMENDED READING: Salvati EA, Gonzalez Della Valle A, Masri BA, et al: The infected total hip arthroplasty. Instr Course Lect 2003;52:223-245.

185. In which of the following situations would locked plating offer the greatest theoretical advantage? 12345A 37-year-old woman with a segmentally transverse radial fracture A 37-year-old woman with an oblique radial fracture A 65-year-old woman with a humeral nonunion treated previously with a plate A 40-year-old man with a segmentally comminuted humeral fracture A 65-year-old man with a segmentally comminuted humeral

PREFERRED RESPONSE: 3 The indications for use of locking plates include the following: (1) metaphyseal and intraarticular fractures; (2) highly comminuted fractures, particularly those involving diaphyseal and metaphyseal bone; (3) osteoporotic bone; (4) proximal tibia and distal femur fractures; and (5) periprosthetic fractures. Theoretically a locking plate could be used anywhere a traditional plate is applied. Locking plates have an advantage with fractures in osteoporotic bone, where loss of fixation is a concern. Highly comminuted diaphyseal or metaphyseal fractures often can be spanned or bridged by a locked submuscular plate. Locked platesoffer an advantage in unstable fracture patterns that traditionally required dual plating, such as bicondylar tibial plateau fractures. Proximal third tibia fractures have a high malunion rate with intramedullary nailing; locked plating may be advantageous with these fractures, as well. There are few absolute contraindications to the use of locked plates. Relative contraindications include the following: (1) fractures best served with fixation other than plates (eg, patella fracture); (2) fractures in which the soft-tissue injury precludes immediate plating (eg, Gustilo grade IIIB or IIIC tibia fracture); (3) simple fracture patterns that do not require either unlocked or locked plates ; and (4) fractures that would require bending of precontoured locked plates Because of the increased cost of locked plates compared with traditional nonlocked ones, use of locking plates. should be reserved for instances in which they are clearly advantageous. RECOMMENDED READINGS: Haidukewych GJ: Innovations in locking plate technology. J Am Acad Orthop Surg 2004;12:205-212. Egol KA, Kubiak EN, Fulkerson E, et al: Biomechanics of locked plates and screws. J Orthop Trauma 2004;18:488-493.

Ryan Miyamoto

PREFERRED RESPONSE: 2 RECOMMENDED READINGS: Saleh K J, Arend t EA, E ldridge J, et al: Symposium: Operative treatment of p atellofemoral arthritis. J Bone Joint Surg Am 2005;87:659-671. Peterson L, Minas T, Brittberg M, et al: Two- to 9-year outcome after autologous chond rocyte transplantation of the kne e. Clin Orthop 2000 ;374:212-234. Autologous chondrycte implantation (ACI) is indicated for focal chondral defects in the knee. The process involves culturing chondrocytes and transplanting them into the cartilage defect beneath a periosteal patch. A bi opsy specimen is taken from either the superomedial edge of the trochlea or the lateral side of the intercondylar notch. It is then commercially cultured, processed and cryopreserved. The biopsy sample can be maintained for 18mo nths until it is processed and undergoes cellular expansion; after 3-5 weeks, it is ready for implantation. The cited reference reinforces the concept that preoperative workup should include AP, lateral, Rosenberg and Merchant views as well as a 54-inch films to determine axial alignment. If ma ltracking is suspected, a CT scan is p erformed to assess lateral patellar subluxation, t rochlear dysplasia, and patellar height. This workup is important because if mechanical malalignment is present, it needs to be addressed (osteotomy, tibial tubercle transfer, etc.) in combination with the resurfacing procedure. A group from Sweden showed improve d (62% to 85%) results when a di stal realignment procedure was performed at the time of AC I. Hence, choice 4 is not correct. Although this group showed that Workerscomp claims had a higher rate of graf t failure, this is not a co ntraindi cation (Choice 1). The procedure is designed for large erosive chondral defects (> 2-3cm2) in young, high-demand patients when the radiogra phic joint space remains intact (Choices 3, 5). A direct quote from the reference: W hen collapse of the joint space can be seen on the Merchant or skyline radiograph, cartilage repars with authologous chondrocyte implantation is no longer possibl e. The procedure relies on intact, full-thickness cartilage margins to maintain the joint space so the grow ing cartilage tissue may fill the defect.

187.

A 17-year-old fe male gymn ast has persist ent pain wh en he r ankle is i n maximal plantar flexion. The MRI scan shown in Figure 71 shows tenosynov itis of wha t tendon ?

1- Tibialis posterior 2- Flexor h allucis longu s 3- Flexor d igitorum communis 4- Peroneu s brevis 5- Peroneu s longu s

PREFERRED RESPONSE: 2 RECOMMENDED READINGS: Richardson EG (ed ): Orthopa edic Knowledge Update: Foot and Ank le 3. Rosemont, IL, A merican Academy Of O rthop aedic Surgeons, 2004, pp 81-89. Sammarco GJ, Cooper PS: Flexor h allucis longu s tendon injuries in dan cers and nondan cers. Foot Ank le Int 1998;19:356-362. Key wo rds are gymnast and maximal plantar flexion. Before you even look at the MRI, you should be thinking about FHL tedonitis or posterior a nkle impingement. Tenosynov itis of the FHL occurs as the tendon passes through a fibro-o sseous tunnel posterior to the talus. Sometimes a nodule can form causing triggering a nd further stenosing tenosynov itis especially in dancers/gymnasts (those on their toes) who place excessive and repetitive loads on the tendon. The pain of FHL injuries is typically po steromedial and exacerbated in the demi-pointe and full-pointe positions. Posterior ankle impingement is caused by either an o s trigonum or an enlarged posterior tubercle. The pain of bony ankle impingement is usually posterolaterally with plantarflexion. In FHL tendonitis, pain (or triggering) occurs with motion of the hallux. You can differentiate this from posterior ankle impingement by flexing the knee, dorsiflexing the ankle and then flex/ext the hallux. Posteromedial ankle pain with this manuver means its FHL tendonitis. So next time your in clinic with the Don and you see a da ncer (girl or guy [easy Pettrone]) with a good bod and nasty beat up feet c/o of ankle pain, you ll know wha t to do. Looking at the MRI, there is a large amount of fluid (white stuff) in the FHL tendon sheath. This fluid is not present in the peroneals (lateral), the Achilles (posterior) or the posterior t ib tendons (medial).

188.

What soft-tissue structure is m ost likely to be injured in association w ith the lowenergy tibial plateau fracture shown in Figu re 72? 12345Medial collateral ligament Lateral collateral ligament Anterior cruciate ligament Posterior cruciate ligament Lateral meniscus

PREFERRED RESPONSE: 5 RECOMMENDED READINGS: Benne tt WF, Brown er B: Tibial plateau f ractures: A study of a ssociated soft tissue injuries. J Orthop Tr auma 1994;8:183 -188. Gardne r MJ, Yacoub ian S, G eller D, et al: Th e incidence of soft tiss ue injury in oper ative tibial plateau fractures: A magnetic resonan ce imaging an alysis of 103 p atients. J Orthop Trauma 2005;19:79-84. Shephe rd L, Abdo llahi K, Lee J, et al: The p revalence o f soft tiss ue injuries in nonope rative tibial plateau fractures as determined by magnetic resonance imaging. J Orthop T rauma 2002;16:628-631. So the first reference is probably not the best since it states that both MCL and meniscal injuries were 20% in their study including MCL injuries being the most common in Schatzker II s. Since this is no t the preferred response, I m unclear as to why t his source was cited. Their ove rall incidence of soft tissue injury wa s 56%.

The second reference cited is from some hospital called HSS. Based on the nature of the question, I m sure its a busy trauma center. In their study 103 consecutive patients with tibial plateau fxs underwent MRI. Overall incidence of soft-tissue derangement was 99%. Their results showed that 91% of all pts had lateral meniscal pathology (acute intrasubs tance tear or capsular separation) including 90% of those with Schatzker II s. Medial meniscal tears were seen in 44% including 37% of Schatzker II s. Destabilizing injuries of the ACL (complete tear or f ootprint avulsion) occurred in 57% including 53% o f Schatzker II s. 28% of pts had PCL injuries (either avulsion or r upture). Complete LCL tear occurred in 29% and 18% of Schatzker II s. Complete MCL tears occurred in 32% of a ll fractures and 36% of Schatzker II s. The third reference is a study using MRI in nonoperative tibial plateau fxs in 20pts. There was an ov erall incidence of 90% soft tissue pathology including 60% having lateral meniscus tears and 55% hav ing medial meniscus tears. There was a 30% incidence of MCL injury, 10% A CL injury, a nd 5% LCL injury.

189.

Bone morphog enetic protein (BMP) is related to which of the following gro wth factors? 12345Fibroblast grow th factor (FGF) Platelet-derived g rowth fa ctor (PDGF) Transforming gro wth factor beta (TGF-beta) Insulin-like growth fa ctor (IGF) Vascular endo thelial grow th factor (VEGF)

PREFERRED RESPONSE: 3 RECOMMENDED READING: Sugama R, Koike T, Imai Y, et al: Bone morphogenetic protein activities are enhanced by 3,5-cyclic adenosine monophosphate through suppression of Smad6 expression in osteoprogenitor cells. Bone 2005;30:206-214. The first line of the reference states that BMPs belong to the TGF-beta superfamily. Hmmm...not alot else to add. BMP-2 and BMP-7 (OP1) are most commonly used clinically. Their osteogenic activity was originally identified after implanting decalcified bone matrix in rodent muscle. The reference is about some dorks that showed that their bone-inducing activities are enhanced by cAMP through the suppression of the protein Smad6.

190.

Which of the fo llowing d iseases is most likely to develop in a pa tient with hereditary retinobl astoma? 12345Squamous cell carcinoma Colon c arcinoma Transit ional cell carcinoma Chondro sarcoma Osteogen ic sarcoma

The 50-year estimated risk of developing a new primary cancer is 51% for patients with hereditary retinoblastoma (5% for nonhereditary retinoblastoma). 37% of these secondary neoplasms are osteosarco mas and 23% were soft-tissue sarcomas. Patients with hereditary retinoblastoma hav e an inherited defect in one copy of the RB1 gene (a tumor suppressor ge ne) found on chromosome 13. If the second copy o f this gene is mutated (as can happen with radiation) cell growth is no longer inhibited. In this hereditary g roup, the risk of secondary osteosarcoma decreased from 51% to 27% when radiation was not used.

Russ Nord

191.Which of the following situations results in the greatest interfragmental strain in a diaphyseal fracture treated with plate fixation? 1- Comminuted fracture treated with the bridge technique 2- Comminuted fracture anatomically reduced with lag screws 3- Short oblique fracture anatomically reduced with a lag screw 4- Transverse fracture anatomically reduced 5- Long oblique fracture with 2 mm of displacement with lag screws

PREFERRED RESPONSE: 5
191) This question and its reference bring up an interesting concept relating to relative stability and the associated indirect healing that occurs in fractures treated with this method of fixation. The question is based on the concept that with a relative stability construct, there will be motion at the fracture site, by definition. In addition, there will also be strain, defined as change in length/original length of object [delta x / x]. The strain occurs only over the fracture site during healing, not the uninvolved regions of uninjured bone. Since a relative stability fixation construct will have a given amount of motion (delta x), the strain will be greatest when the fracture site (x) is small [denominator is minimized]. This (x) is smallest in simple, non-comminuted fractures that are fixed in near approximation (no gap). To the contrary, comminuted fractures have more fracture sites (total x) and therefore less strain, given a constant total motion (delta x). The one caveat to this is when there is no displacement (x), which occurs in the setting of compression across a fracture. Because there is preloaded continuous contact across the fracture in compression, there is no motion (and thus no strain). So, back to the question, we eliminate 1 and 2 because they are comminuted and will have less strain than a simple fracture like choice 5. Choice 3 has anatomic compression with a lag screw and thus no significant strain due to compression. Choice 4 is also an anatomic reduction with a plate (presumably a compression plate), so it will not have significant strain either. Choice 5 will have the most strain because it is a simple fracture treated with fixation yielding relative stability. As a summary, choices 1,2,5 exhibit relative stability and since choice 5 is a simple fracture, it has the most strain. Choices 3 and 4 are constructs with absolute stability and thus have minimal strain across the fracture. So, a somewhat shitty reduction is OK with relative stability constructs. RECOMMENDED READING: Perren SM: Evolution of the internal fixation of long bone fractures: The scientific basis of biological internal fixation: Choosing a new balance between stability and biology. J Bone Joint Surg Br 2002;84:1093-1110.

192.A 45-year-old man reports pain and instability of the right shoulder with activities of daily living. History reveals that he underwent two previous surgeries: one arthroscopic and one open Bankart repair. A CT scan is shown in Figure 73. What is the next most appropriate step in management?
12345Physical therapy program with emphasis on strengthening of the rotator cuff and scapular stabilizers Arthroscopic stabilization with repair of the anterior labrum and capsular plication Open inferior capsular shift for circumferential capsular tightening Coracoid osteotomy and transfer with conjoint tendon to the anterior glenoid Pectoralis major transfer under the coracoid

PREFERRED RESPONSE: 4
192) This young patient (age 45) is status-post one failed arthroscopic and one failed open Bankart repair (repair of the anterior-inferior labrum to the glenoid after anterior shoulder instability/dislocation). The CT scan shows a bony lesion in the anterior glenoid, a bony Bankart. With glenoid bone loss as seen on the CT, it is clear that surgical stabilization is required (eliminate choice 1). The guy has already failed an arthroscopic and an open procedure, so what makes you think you can fix the problem arthroscopically this is a relative indication for an open procedurean absolute indication is significant glenoid bone loss, but it is tough to assess the quantity of bone loss from one CT cut. Either way, forget an arthroscopic procedure (eliminate choice 2). Eliminate choice 3 because this is a procedure for multi-directional instability while the CT and history suggest anterior instability. Pectoralis transfers (choice 5) are often used for irreparable subscapularis tears, but are not so relevant here. Choice 4, a coracoid/conjoint tendon transfer (the Modified Bristow or Latarjet procedure) will reconstitute the bone loss and act as a sling reinforcing the inferior capsular ligamentous complex and the inferior portion of the subscapularis. This is the best option and boasts recurrence rates of only 0-6%, although patients often have a limitation in external rotation of 6-23 degrees post-op. RECOMMENDED READING: Millett PJ, Clavert P, Warner JJ: Open operative treatment for anterior shoulder instability: When and why? J Bone Joint Surg Am 2005;87:419-432.

193. Nonunion of a surgically treated tibial diaphyseal fracture correlates most closely with
12345fracture comminution. gapping at the fracture site. a nonsurgically treated fibular fracture. a mid-diaphyseal location. a closed section nail.

PREFERRED RESPONSE: 2
193) This question is straightforward. The Bhandari paper is a retrospective observational study of 200 pts with operatively treated tibial shaft fractures. Three variables predicted reoperation: open fracture (RR=4.32), lack of cortical continuity between the fracture ends (choice 2) (RR=8.33) and transverse fracture (RR=20.0). In the Audinge paper, nonunion rates were increased in open fractures, fractures of the distal tibia and fractures with postoperative diastasis (choice 2). Specifically, AO classification (comminution - choice 1), and treatment modality (choice 5) were not directly correlated with non-union rates. Distal, not mid-shaft (choice 4) fractures were at increased risk for nonunion. Presence of a fibula fracture (sort of choice 3, b/c you dont fix them) was not correlated with nonunion. So, the answer is choice 2, gapping at the fracture site. RECOMMENDED READINGS: Audige L, Griffin D, Bhandari M, et al: Path analysis of factors for delayed healing and nonunion in 416 operatively treated tibial shaft fractures. Clin Orthop 2005;438:221-232. Bhandari M, Tornetta P III, Sprague S, et al: Predictors of reoperation following operative management of fractures of the tibial shaft. J Orthop Trauma 2003;17:353361.

194.Induction of cytokine release by macrophages in response to polyethylene wear debris is greatest with a polyethylene particle size of
12345less than 1.0 micron. 1.0 to 2.5 microns. 2.5 to 5.0 microns. 5.0 to 10.0 microns. greater than 10.0 microns.

PREFERRED RESPONSE: 1
194) Not much to sayto quote the reference: There is a wealth of evidence to indicate that the presence and activation of macrophages in the periprosthetic tissues around joint replacements is stimulated my UHMWPE particles. Particles within the size of 0.11.0 microns have been shown to be the most reactive. Macrophage activation involves the release of cytokines such as TNF-. Thus, the answer is choice 1, less than 1.0 micron. RECOMMENDED READING: Ingham E, Fisher J: The role of macrophages in osteolysis of total joint replacement. Biomaterials 2005;26:1271-1286.

195.Figure 74 shows the radiograph of a 75-year-old man who is undergoing a revision total hip arthroplasty. During surgery, it is noted that the acetabular component is well fixed. After revising the femur, the hip is noted to be unstable. Treatment should now consist of
1removing the liner, roughening the metal part of the shell, and cementing a polyethylene liner into the socket. removing the liner, roughening the metal part of the shell, and cementing a constrained liner. revising the metal shell to a hemispherical socket. revising the acetabular metal shell to a bipolar component. trochanteric advancement.

2345-

PREFERRED RESPONSE: 3
195) So, the guys got a hybrid total hip with what looks like lucencies at the femoral stem/cement interface (and stem is in varus), so we think were revising for aseptic loosening. We take out the femur and replace it. The acetabulum looks well fixed, but when we reduce him, the hip isnt stable. What do we do? Well, before you dispo him to rehab or try to give him an MI so he can get transferred to medicine, lets get the hip stable. There are many causes of potential hip instability, including inadequate soft tissue tension (possibly neurologic or iatrogenic in origin), impingement secondary to poor head/neck ratio or osteophytes etc., patient non-compliance and component malpositioning. Lets find the cause in this guy and fix it. He didnt dislocate pre-op, so we cant blame non-compliance. It looks like the cup takes a big head so the head neck ratio should be favorable, and weve got to presume that the surgeon has removed any osteophytes. However, the lateral opening angle (theta angle) measures 61 degrees, while it should be around 45 degrees. The Callaghan reference says that a theta angle over 60 degrees warrants revision. Thus, any response that doesnt involve revising the acetabular shell is incorrect (1,2,5). Trochanteric advancement allows you to increase soft tissue tension without increasing leg length, but this wouldnt be the right route unless the components were wellpositioned and the soft tissues seemed to be the culprit. Choice 4 is strange and wrong for a few reasons. First, bipolar cups dont do well in THA61% fail by 3 years (JBJS-A 72(8):1230-5). Second, due to the bulk of having two layers in the cup, you are forced to have thinner poly and a smaller head, both of which are negatives. Bipolar cups are really only indicated in cases with massive acetabular deficiency where secure fixation is not otherwise possible. This leaves the correct choice, 3, revise the cupand this time put it in correctly. RECOMMENDED READINGS: Masri BA, Mitchell PA, Duncan CP: Removal of solidly fixed implants during revision hip and knee arthroplasty. J Am Acad Orthop Surg 2005;13:18-27. Mitchell PA, Masri BA, Garbuz DS, et al: Removal of well-fixed, cementless acetabular components in revision hip arthroplasty. J Bone Joint Surg Br 2003;85:949-952. Callaghan JJ, ORourke MR, Goetz DD, et al: Use of a constrained tripolar acetabular liner to treat intraoperative instability and postoperative dislocation after total hip arthroplasty: A review of our experience. Clin Orthop 2004;429:117-123.

Vipul Patel

196. The injury shown in Figure 75 is classified as what type of acetabular fracture?

12345-

T-shaped Posterior wall Transverse Posterior column Both column

PREFERRED RESPONSE: 1 T type Xray shows a disruption of the iliopectineal and ilioischial line therefore both anterior and posterior columns are involved with a fracture through the obturator ring. Lets reason out the other choices, In Both-column fractures, the iliac wing is separated from the acetabulum which does not appear to be the case in this patient. Choices 2 and 4 could be ruled out because of the both column involvement. T-Type fractures differ from transverse fractures by the additional fracture line that runs through the quadrilateral surface. In a pure transverse fracture, the anterior and posterior columns may be reduced through a single approach. Once the anterior column has been reduced, the posterior column will follow the reduction and can be palpated indirectly. In a T-type fracture, the 2 columns must be reduced independently. This becomes extremely important when choosing a surgical approach; therefore, it is important to recognize the subtle difference between transverse and T-type fractures when if they are not significantly displaced
RECOMMENDED READINGS: Letournel E, Judet R (eds): Fractures of the Acetabulum, ed 2. Berlin, Germany, Springer Verlag, 1993. Baumgaertner MR, Tornetta P III (eds): Orthopaedic Knowledge Update: Trauma 3. Rosemont, IL, American Academy of Orthopaedic Surgeons, 2005, pp 259-269.

197.With current chemotherapy regimens, what is a typical 5-year survival rate for patients with nonmetastatic high-grade osteogenic sarcoma?

12345-

Less than 5% 20% 50% 70% 90%

PREFERRED RESPONSE: 4 The questions stems from a study at Sloan evaulating disease-free survival (DFS) from preoperative chemotherapy regimen in 279 patients with previously untreated OS without metastasis w/subsequent surgical resection of primary tumor. They basically found that the 5-year DFS was 76% for patients aged less than or equal to 21 years who had extremity primary tumor and were treated with the T10 protocol. You want to know what the T10 protocol is?? I dwont even know!!!

Quick facts: With metastatic disease, survival rate is ~30 percent Two suppressor genes, p53 and Rb, have major roles in tumorigenesis in osteosarcoma. Approximately 3-4 percent of children with osteosarcoma carry constitutional germline mutations in p53 Strongest genetic predisposition to osteosarcoma is found in patients with hereditary retinoblastoma About 15-20 percent of the patients have metastatic disease at the time of diagnosis If treated with surgery alone, the natural history is recurrence and more than 80 percent of patients will develop metastatic disease
RECOMMENDED READINGS: Meyers P, Heller G, Healey J, et al: Chemotherapy for nonmetastatic osteogenic sarcoma: The Memorial Sloan-Kettering experience. J Clin Oncol 1992;10:5-15. Menendez LR (ed): Orthopaedic Knowledge Update: Musculoskeletal Tumors. Rosemont, IL, American Academy of Orthopaedic Surgeons, 2002, pp 157-174.

198. What is the treatment of choice for a chronic rupture of the patellar tendon after a total knee arthroplasty?

12345-

Arthrodesis Amputation Transposition of the medial and lateral gastrocnemius muscles Reconstruction of the patellar tendon using a gracilis autograft Reconstruction using an Achilles tendon bone/tendon allograft

PREFERRED RESPONSE: 5
The answer is from the below referenced paper. Authors looked at a series of 14 patients w/ chronic extensor mechanism rupture and compared 2 methods of treatment: Achilles calcaneal block vs quad tendon-patella-patellar tendon-tibial tubercle composite graft (I know..dont ask). They basically concluded that all patients were satisfied and improved in their functional status I think key points in this question are to understand that primary repair of a chronic rupture does not do well. Arthrodesis and Amputation (choices 1 and 2) are radical but probably the preferred form of treatment in Russia. The question asks for treatment of choice and it would be an allograft. The preferrred choice of graft in this paper was achilles calcaneal block.

RECOMMENDED READING: Barrack RL, Stanley T, Allen BR: Treating extensor mechanism disruption after total knee arthroplasty. Clin Orthop 2003;416:98-104.

Q199 - Deleted

200. A skeletally immature 12-year-old boy has recurrent atraumatic knee pain associated with repetitive sports activities. A radiograph is shown in Figure 77. Initial management should consist of

12345-

application of a bone stimulator. non-weight-bearing for 6 weeks. arthroscopic excision of loose fragments. open reduction and internal fixation. autologous chondrocyte implantation.

PREFERRED RESPONSE: 2

The Xray shows a OCD of the medial femoral condyle in a skeletally immature boy. In the adolescent with OCD, the presence of adequate remaining skeletal growth (open physes) is the most important determinant of treatment. Closed treatment generally is accepted as the treatment of choice for nondisplaced lesions in juvenile OCD patients with success rates between 50%-91%; therefore, a trial of adequate closed treatment is appropriate for the skeletally immature patient. Initial treatment is limitation of activities with protected weight bearing. The success of conservative treatment depends mainly on the presence of adequate subchondral bone attached to the articular fragment, and the size of the attached bone fragment relative to the size of the femoral concavity. If no subchondral bone is attached to the fragment, there is no chance for the articular cartilage lesion to heal.
RECOMMENDED READINGS: Cain EL, Clancy WG: Treatment algorithm for osteochondral injuries of the knee. Clin Sports Med 2001;20:321342. Cahill BR, Ahten SM: The three critical components in the conservative treatment of juvenile osteochondritis dissecans (JOCD): Physician, Parent, and Child. Clin Sports Med 2001;20:287-298.

Tony Quach

201. When treating a tibial fracture with the long lateral plate shown in Figure 78, what structure is at greatest risk of iatrogenic injury during percutaneous screw insertion? 1- Common peroneal nerve with placement of the proximal posterior screw 2- Posterior tibial nerve with placement of the proximal posterior screw 3- Peroneal nerve with placement of the middle screw 4- Superficial peroneal nerve with placement of the middle screw 5- Superficial peroneal nerve with placement of the distal screw

PREFERRED RESPONSE: 5
The x-ray shows a proximal tibia/shaft fracture treated with a LISS (Less Invasive Stabilization System) plate. Deangelis et al. showed in a caderveric model that the superficial peroneal nerve is at significant risk during percutaneous screw placment in holes 11 to 13 for the 13 hole LISS plate. They applied the LISS plate to 14 cadavers and found in 12 of the 14 specimens, the SPN was 5mm or less from the center hole of 13. In 6 of the specimens, the nerve was touching the guide wire at hole 13 and in 1 specimen, the guide wire was piercing the nerve at hole 13.

RECOMMENDED READINGS: Cole PA, Zlowodzki M, Kregor PJ: Treatment of proximal tibia fractures using the less invasive stabilization system: Surgical experience and early clinical results in 77 fractures. J Orthop Trauma 2004;18:528-535. Deangelis JP, Deangelis NA, Anderson R: Anatomy of the superficial peroneal nerve in relation to fixation of tibia fractures with the less invasive stabilization system. J Orthop Trauma 2004;18:536-539.

202. A 2-year-old girl has a deformity of her left leg. Examination reveals freckling in the axilla and numerous caf-au-lait spots. A radiograph is shown in Figure 79. Based on these findings, management should consist of 1- supplemental vitamin D and phosphate. 2- a clamshell orthosis. 3- tibial osteotomy. 4- resection of the deformed region of bone and insertion of a vascularized fibula. 5- observation for spontaneous remodeling.

PREFERRED RESPONSE: 2
The freckling in the axilla and the numerous caf-au-lait spots suggest that the patient has neurofibromatosis (NF). Neurofibromatosis is the most common single gene disorder found in humans. It is an autosomal dominant disorder with variable penetrance. The two most common forms of neurofibromatosis are type I and type II. Type I is also known as "von Recklinghausen disease" and is common to have orthopedic manifestations. Type II is also called "central neurofibromatosis" or "familial acoustic neuroma" and is rare to have orthopedic manifestations. This diagnosis of NF type I is made by identifying at least two of the clinical findings:
At least six caf-au-lait spots, larger than 5 mm in diameter for children, and larger than 15 mm for adults Two neurofibromas, or a single plexiform neurofibroma Freckling in the axillae or inguinal region An optic glioma At least two Lisch nodules (hamartoma of the iris) A distinctive osseous lesion, such as vertebral scalloping, or cortical thinning A first-degree relative with NF1

Skeletal manifestations of neurofibromatosis include: scoliosis, kyphoscoliosis, c-spine abnormality, spondylolisthesis, soft tissue neoplasm, subperiosteal bone proliferation, and congenital tibia bowing and pseudarthrosis. The x-ray shows the patient has anterolateral bowing of the left tibia. Neurofibromatosis occurs in approximately 50% of patients with anterolateral bowing with or without pseudarthrosis. Spontaneous resolution is uncommon and full time brace treatment is indicated. Supplement Vit D and phospate and observation is clearly wrong. Since there is no evidence of pseudarthosis, there are no operative indications.
RECOMMENDED READINGS: Schonecker PL, Rich MM: The lower extremity, in Morrissy RT, Weinstein SL (eds): Lovell and Winters Pediatric Orthopaedics, ed 5. Philadelphia, PA, Lippincott Williams & Wilkins, 2001, pp 1059-1104. Crawford AH Jr, Bagamery N: Osseous manifestations of neurofibromatosis in childhood. J Pediatr Orthop 1986;6:72-88.

203. After a severe collision in a game, a football player is laying face down and is unconscious. He is carefully log rolled to his back onto a backboard. He then begins to show signs of respiratory distress. What is the next step in management?
12345cannula Removal of the helmet Removal of the helmet and shoulder pads Removal of the face mask only, leaving the helmet and shoulder pads in place Immediate percutaneous tracheostomy Administration of oxygen through a nasal

PREFERRED RESPONSE: 3
The player shows signs of respiratory distress, thus the facemask must be removed to maintain the airway. In a situation where the c-spine status is not certain, the helmet and shoulder pads should not be removed. Gastel et al studied 8 intact and unstable c-spine cadaveric specimens under 4 different football equipment scenerios: 1)no equipment, 2)helmet only, 3) shoulder pads only, and 4) helmet and shoulder pads on. They found that the neutral alignment was maintained only when the helmet and shoulder pads were on. In their flexion distraction injury model, they found that having only the helmet on increases forward angulation, posterior disc space height, and dorsal element distraction. They extrapolated that in a extension type model, the shoulder pads would cause the same displacement. Thus they concluded that it was best to leave the helmet and shoulder pads in place.

RECOMMENDED READINGS: Torg JS: Cervical spine injuries in the adult, in DeLee JC, Drez D Jr, Miller MD (eds): Orthopedic Sports Medicine: Principles and Practice, ed 2. Philadelphia, PA, WB Saunders, 2003, pp 791-828. Gastel JA, Palumbo MA, Hulstyn MJ, et al: Emergency removal of football equipment: A cadaveric cervical spine injury model. Ann Emerg Med 1998;32:411-417.

204. What complication occurs more frequently with current resurfacing arthroplasty compared to conventional total hip arthroplasty? 12345Osteolysis Infection Sciatic nerve palsy Periprosthetic fracture Dislocation

PREFERRED RESPONSE: 4
This is a need to know basis. To get this answer correct you just need to know that periprosthetic fractures occurs more frequently with resurfacing than in conventional hip arthroplasty. The reference wasnt very helpful. 377 hip resurfacings13 revisions8 secondary to femoral neck fractures Getting a freebie on the OITEpriceless.

RECOMMENDED READING: Little CP, Ruiz AL, Harding IJ, et al: Osteonecrosis in retrieved femoral heads after failed resurfacing arthroplasty of the hip. J Bone Joint Surg Br 2005;87:320-323.

205. A 6-year-old girl has sustained numerous long bone fractures in the past. Her parents report that there is no family history of any similar problem. Examination reveals normal-appearing sclera and brownish opalescent teeth. A radiograph of her lower extremities is shown in Figure 80. The patients disorder is the result of 1234parental abuse. abnormal osteoclast function. vitamin D deficiency. defective N-Ac-Gal-6 sulfate sulfatase enzyme. qualitative defect of type I collagen synthesis.

5-

PREFERRED RESPONSE: 5
The clinical picture suggests Osteogenesis Inperfecta (OI). There are four type of OI summarized in the chart below. The disorder results from a mutation in the genes that code for type 1 collagen (mainly bone, dentin, sclerae and ligaments) . Over 150 specific mutations have been identified in the COL1A1 and COL1A2 gene.

RECOMMENDED READINGS: Kocher MS, Shapiro F: Osteogenesis imperfecta. J Am Acad Orthop Surg 1998;6:225-236. Cole WG: Etiology and pathogenesis of heritable connective tissue diseases. J Pediatr Orthop 1993;13:392-403.

Sam Sanders

206. A 40-year-old woman who plays tennis fell and sustained a surgical neck fracture of her dominant arm. She underwent open reduction and internal fixation 24 hours after the injury. A postoperative radiograph is shown in Figure 81. Management should now consist of

1- sling immobilization until radiographic union is evident. 2- revision surgery with conversion to a humeral head arthroplasty. 3- revision surgery to correct the valgus impaction deformity. 4- revision surgery to correct varus angulation. 5- early physical therapy to prevent stiffness.

XR

PREFERRED RESPONSE: 4 RECOMMENDED READINGS: Williams GR Jr, Copley LA, Iannotti JP, et al: The influence of intramedullary fixation on figure-of-eight wiring for surgical neck fractures of the proximal humerus: A biomechanical comparison. J Shoulder Elbow Surg 1997;6:423-428. Siegel JA, Dines DM: Techniques in managing proximal humeral malunions. J Shoulder Elbow Surg 2003;12:69-78.

Although proximal humerus malunions are rare, the causes are multiple. Malunion can occur after open or closed treatment of a fracture, although it occurs more commonly after closed management. Many cases of malunion are caused by the physician knowingly and willingly accepting a poor alignment secondary to the age or underlying medical problems of the patient. Often, orthopedic surgeons assume the wait-and-see approach to three-part and some four-part proximal humerus fractures, allowing the fracture to heal, then seeing if the patient is symptomatic. In some cases, when the patient's medical problems prohibit surgical treatment, there is no other option. Because the results of late reconstruction are inferior to those of acute arthroplasty, [15] however, such conservative treatment may not be routinely warranted. On occasion, a proximal humerus fracture is overlooked or the extent of the injury is not appreciated, and as a result a malunion develops. Most notable is the failure to diagnose a posterior dislocation in association with a fracture. In some cases, these fixed impacted fractures may be overlooked by less experienced physicians, particularly in the emergency department when appropriate radiographs were not taken. The key to avoiding these errors is to obtain proper imaging studies routinely. The necessity for an axillary view radiograph cannot be overemphasized. Some fractures of the proximal humerus are inherently less stable than others. On initial presentation, the fracture alignment may be acceptable but may predictably displace over time. Experience plays a large role in avoiding this pitfall. The choice of immobilization, or lack thereof, may contribute to displacement. The patient should be placed in a sling with or without a swathe in the immediate postfracture period. The use of a hanging arm cast, as described for a humeral diaphyseal fracture, may cause distraction across the fracture and increase the chance of nonunion or malunion. Failure to use any immobilization or to discontinue immobilization and begin physical therapy prematurely may also contribute to displacement of the fracture. Rarely the long head of the biceps tendon, capsule, or deltoid muscle can be interposed at the fracture site, blocking reduction, and predispose to nonunion or malunion. The final cause of proximal humerus malunion is failure of internal fixation. This failure is frequently secondary to difficulty in obtaining adequate purchase in the cancellous bone of the humeral head. The various forms of fixation used in the treatment of acute fractures and their associated success rates are beyond the scope of this article.

DEFORMING FORCES An understanding of the various types of proximal humerus fractures and the corresponding deforming muscle forces is essential to recognizing the patterns of malunion. The four structures of the proximal humerus that are typically assessed in evaluation of a fracture of the proximal humerus are the humeral head and articular fragment, the greater tuberosity, the lesser tuberosity, and the humeral shaft. Each of these components except for the head has its own tendinous insertions that pull a free fracture fragment in a predictable direction. Two-Part Surgical Neck Malunions An isolated surgical neck fracture can heal with resultant anterior angulation and varus or valgus deformity. The deforming force in this injury is the displacement of the shaft anteriorly by the pull of the pectoralis major and potential abduction of the head by the rotator cuff. Such a deformity, if severe enough, causes limitation of forward flexion and, in some cases, loss of abduction as well. With each degree of varus angulation, a degree of overhead elevation is lost. With this deformity, the greater tuberosity abuts against the top of the glenoid and prevents sufficient external rotation to allow the arm to elevate properly

207. A 67-year-old woman has a painless lump in the adductor compartment of the thigh. Axial T1and T2-weighted MRI scans are shown in Figures 82a and 82b. A biopsy specimen is shown in Figure 82c. What is the most likely diagnosis?

1- Liposarcoma 2- Lipoma 3- Neurofibroma 4- Malignant peripheral nerve sheath tumor 5- Pleomorphic sarcoma

Images

Histo

PREFERRED RESPONSE: 1 RECOMMENDED READING: Enzinger FM, Weiss SW: Soft Tissue Tumors, ed 4. St Louis, MO, Mosby, 2001, pp 21-44.

Liposarcoma has two forms: - low grade myxoid liposarcoma; - most common; - occurs in young adults; - in myxoid liposarcoma, lipoblasts are dispersed in anastomosing capillary framework; - fine reticular vascular pattern; - high grade pleomorphic liposarcoma: - pleomorphic liposarcoma is more common in older patients; - enlarging but painless mass in the buttocks (most common site), proximal thigh, or upper arm is the classic presentation; - more aggresive tumor which tends to invade bone; - have amorphously arranged large vascular lakes; - contains large, pyknotic, bizarre, mononucleated cells in combination with immature lipoblasts; - Differential Dx: - lipoma (see below) - most liposarcomas occur proximal to the knee and elbow; - Angiography: vascularity of tumor is best demonstrated by angiography; - Bone Scan: - marked increase in radioisotope uptake in the early phase of bone scan;

CT Scan: - radiolucency noted on CT may suggest histogenesis in low grade, well differentiated lesions with a high fat content; Treatment: - stage I myxoid lesions: treated w/ wide surgical excision, - stage II pleomorphic lesions: - requires wide excision supplemented w/ XRT, or radical resection, or amputation;

Lipoma is: - benign tumor composed of lobules of mature fat; - most common tumor of soft tissue; - occurs most often in adults; - tumor usually presents as active stage 2 lesion, later becoming latent; - usually it appears as soft, slowly enlarging, asymptomatic mass located superficially in subQ tissues, commonly seen in the back, buttocks, and thighs; - it can also appear deep to the fascia, lying in fatty planes along N/V bundles; - lipomas do not cross fascial boundries but will grow in between fascial planes; - lipomas that arise in deep tissues may become remarkably large; - these lesions may have a firm consistency similar to a sarcoma; - sub-types: - intra-muscular lipoma: - these lesions can become large in size before detection; - typically has a firm consistency which mimics sarcoma; - often these lesions will be in close contact w/ neurovascular bundle; - angiolipoma: - variant of lipoma w/ vascular component, occurs in children as a deep, subcutaneous or intramuscular soft mass; - it is distinguished from ordinary lipoma by distinct tenderness to palpation; - angiography is the best method for diagnosing angiolipoma; - note that in the majority of cases multiple tumors may be present; - diff dx: - Kaposi's sarcoma; - angiosarcoma - spindle cell sarcoma: - presents as a painless nodule; - histologically the tumor contains a mixture of fat and spindle cells; - Diff Dx: liposarcoma: most occur proximal to the knee and elbow; - Radiographic studies: - crisply marginated radiolucent image; - may show calcification in areas of necrosis and metaplastic bone or cartilage; - MRI: - w/ lipoma, signal intensity of lesion parallels that of subcutaneous fat on all pulse sequences (T1 and T2); - Bone Scan: - usual avascular lipoma appears avascular on bone scan; - if bone scan is hot consider liposarcoma; - Treatment: - marginal excision is successful for lipoma, and recurrences are rare; - note that lipomas cannot be read by frozen section, and can only be interpreted by permanent sections;

Neurofibroma: - neurofibroma is a benign tumor of neural origin; - has an autosomal dominant trait w/ variable penetrance & high rate of spontaneous mutation; - it is the most common single gene disorder (1 in 3000); - presentation: - neurofibromas are most commonly found in skin and subQ tissue or associated w/ nerve fibers; - tumors may be solitary or multiple (multiple neurofibromas are most common); - may affect patients of any age; - most neurofibromas present as an active stage 2 tumors, but aggresive stage 3 infiltratrive forms may be seen occasionally; - long standing neurofibromatosis may undergo malignant transformation to neurosarcoma, which is often heralded by a new onset neurologic deficit; - diff dx: - fibrous dysplasia: - proteus syndrome: - a disorder characterized by congenital hamartomas; - hemihypertrophy, partial gigantism of hands or feet, skin nevi, subcutaneous tumors, and skull hyperostosis; - no cafe-au-lait spots and no neurofibromas are encountered; - disorder may stem from abnormal growth hormone metabolism; - Associated Anomalies: - cervical spine kyphosis; - scoliosis: - congenital psuedarthrosis of clavicle; - pseudoarthrosis of extremities: - congenital psuedarthrosis of tibia: (anterolateral bowing) - pseudoarthrosis of radius: - gigantism of the limb: - erosive defects from contiguous tumors; - multiple fibrous cortical defects; - subperiosteal calcifying hematoma; - MRI: - demonstrates an oval, avascular tumor; - neurofibroma is frequently seen as a fusiform mass in continuity w/ major nerve best demonstrated by MRI; - Histology: - loose, spindle cell stroma containing wavy eosinophilic fibrillar material; - verocay bodies may be seen; - composed of amorphorous eosinophilic material that is surrounded by spindle shaped or oval cells;

- Treatment: - scoliosis: - congenital psuedarthrosis of tibia: (anterolateral bowing) - excision of neurofibroma: - some neurofibromas infiltrate a peripheral nerve, thereby precluding en bloc extra capsular marginal excision w/o damaging the nerve;

A malignant peripheral nerve sheath tumor (MPNST) or malignant neurolemmoma is a form of cancer of the connective tissue surrounding nerves. Given its origin and behavior it is classified as a sarcoma. About half the cases are diagonosed in people with neurofibromatosis; the lifetime risk for an MPNST in patients with neurofibromatosis type 1 is 8-13% (Evans et al 2002). The first-line treatmenet is surgical resection with wide margins. Chemotherapy (e.g. high-dose doxorubicin) and often radiotherapy are done as adjuvant and/or neoadjuvant treatment

208. A 19-year-old man sustained a laceration to his palm that was repaired primarily in the emergency department 3 weeks ago. He now reports a tender pulsatile mass in his palm and intermittent tingling and numbness in his ring and little fingers. A clinical photograph of the mass is shown in Figure 83. What is the next most appropriate step in management?

1- MRI 2- Electromyography and nerve conduction velocity studies 3- Intra-arterial urokinase infusion 4- Ethanol embolization 5- Surgical exploration

Image

PREFERRED RESPONSE: 5 RECOMMENDED READINGS: Koman LA, Ruch DS, Smith BP, et al: Vascular disorders, in Greens Operative Hand Surgery, ed 4. New York, NY, Churchill Livingstone, 1999, pp 22542302. Yajima H, Tamai S, Ono H: Aneurysms of the digital artery: A review and report of three cases. Microsurgery 1995;16:566-570.

In the past 7 years, we have encountered six patients with finger ischemia as a result of digital artery occlusion associated with seven distal ulnar artery aneurysms. Our experience with the management of these patients forms the basis of this report. All patients were men, with a mean age of 29 years, and all experienced repetitive trauma to the involved upper extremity. Each patient presented with the acute onset of cool and painful digits, with no previous history of cold sensitivity or Raynaud's syndrome. None of the patients had any serologic or clinical evidence of autoimmune disease. Angiography revealed occlusion of the ulnar artery on the affected side in two patients and patent ulnar artery aneurysms in the remaining five patients. There was occlusion of multiple common and proper digital arteries in all patients. One patient with bilateral ulnar artery aneurysms underwent operative repair consisting of aneurysm excision and replacement with autogenous vein grafts from the lower extremity. All patients have improved symptoms, and the grafts remained patent over a mean follow-up of 24 months (range: 13 to 57 months). Based on these results, we recommend that excision and grafting be considered for patients with symptomatic patent ulnar artery aneurysms. Selected patients with thrombosed aneurysms with ongoing digital ischemia may also benefit from surgical intervention.

209. A posterolateral or posterior peritrochanteric portal for hip arthroscopy places the sciatic nerve at greatest risk when the hip is in which of the following positions?

12345-

Extension Internal rotation External rotation Abduction Adduction

PREFERRED RESPONSE: 3 RECOMMENDED READINGS: McCarthy JC: Hip arthroscopy: Applications and technique. J Am Acad Orthop Surg 1995;3:115-122. McCarthy JC, Lee J: Hip arthroscopy: Indications, outcomes, and complications. J Bone Joint Surg Am 2005;87:1137-1145.

Anatomic Approaches The significant neurovascular structures that surround the hip, including the femoral nerve and artery anteriorly, the lateral femoral cutaneous nerve anterolaterally, and the sciatic nerve and gluteal vessels posteriorly, make accurate portal placement imperative.
Anterior (Anterolateral) Portal The entry point for the arthroscope is at the junction of a horizontal line directed laterally from the symphysis pubis and a vertical line directed inferiorly from the anterior superior iliac spine. An 18-gauge spinal needle is advanced toward the femoral head along a line 45 degrees medial and 45 degrees proximal to this point.1 Following distention of the joint with normal saline, the arthroscope is positioned into the joint along the spinal needle pathway. The arthroscope passes close to the lateral femoral cutaneous nerve, and a neuropraxia can occur. Deeper, the ascending branch of the lateral femoral circumflex artery is at risk. The femoral nerve and artery are within 3 to 4 cm and should not be at issue unless portal misplacement occurs. This approach allows visualization of the anterior femoral neck and superior retinacular fold and the ligamentum teres. A 70-degree arthroscope is necessary for visualization of pathologic changes along the anterior labrum or acetabulum

Anterior Paratrochanteric Portal This portal is located 2 to 3 cm anterior and 1 cm proximal or distal to the greater trochanter. Puncture of the joint capsule occurs close to the intertrochanteric line. This approach allows visualization of the anterior femoral neck and head as well as the intrinsic capsular folds. Synovial fronds beneath the zona orbicularis and the raised lip of the labrum are well seen. However, because of the high degree of obliquity of the approach and the thickness of the capsule, the arthroscope can be directed too far anteriorly and can potentially damage the femoral neurovascular bundle.

Proximal Trochanteric Portal This portal is relatively safe. The entry point is just proximal (within 2 to 3 cm) to the tip of the greater trochanter, and the arthroscope is advanced medially and slightly superiorly, directed toward the center of the hip joint. If the arthroscope is not aimed directly at the femoral head, there is a risk of slipping off anteriorly and damaging the femoral neurovascular structures. The zone of 2 cm or so in the area proximal to the tip of the trochanter is commonly used in combination with other portals to allow triangulation. The acetabular labrum, the femoral head, and the fovea can be well visualized through this portal.

Posterior Paratrochanteric Portal This is a valuable portal, but it is potentially hazardous if one does not bear in mind the important adjacent structures. The approach is made 2 to 3 cm posterior to the tip of the greater trochanter at a level that corresponds to the anterior paratrochanteric portal. The hip should not be externally rotated because the sciatic nerve, which is nearby, is brought into danger with this maneuver. This offers one of the best views of the ligament of Weitbrecht and is also valuable for visualizing the posterior capsule and the inferior edge of the ischiofemoral ligament, which is seen as a thickening of the capsule.

Posterior Portal This approach should be used only with the aid of a miniarthrotomy. The sciatic nerve and the superior gluteal vessels are nearby and must be identified through a small incision. The short external rotators must then be divided under direct vision near their insertion. The combination of an arthroscopic and an open (miniarthrotomy) approach is much less traumatic than a formal open arthrotomy and allows visualization of areas that might otherwise be visible only with dislocation of the hip joint. Foreign bodies have been removed with the use of this approach.

Relative Safety of Approaches The anterior paratrochanteric and proximal trochanteric approaches are relatively safe. The posterior paratrochanteric and anterolateral approaches are also safe, but one must pay particular attention to surgical technique with special concern for neurovascular structures. The posterior approach requires a small incision for safety.

210. Which of the following features improves fluid film lubrication in a metal-on-metal total hip arthroplasty?
1- Smaller diameter femoral head, a completely congruent fit between the socket and the head, and sufficient surface roughness to allow for some microseparation between the head and the socket 2- Smaller diameter femoral head, a slight clearance between the socket and the head, and no surface roughness 3- Larger diameter femoral head, a completely congruent fit between the socket and the head, and no surface roughness 4- Larger diameter femoral head, a slight clearance between the socket and the head, and minimal surface roughness 5- Larger diameter femoral head, a slight clearance between the socket and the head, and sufficient surface roughness to allow for some microseparation between the head and the socket

PREFERRED RESPONSE: 4 RECOMMENDED READING: Dumbleton JH, Manley MT: Metal-on-metal total hip replacement: What does the literature say? J Arthroplasty 2005;20:174-188.

Second-generation THRs were introduced in the late 1980s, and their use became widespread in the mid-1990s. The rationale for the reintroduction of M/M bearings was that shortcomings with first-generation devices were well understood and could be addressed with design and material changes. This is an assumption that can be proven with second-generation hip prostheses only by success in large numbers of patients at long times of implantation. Those laboratory studies that have been reported do indicate that the issues of bearing design are better understood with second-generation M/M designs than before. Based on the literature, it may be concluded that a head-to-cup clearance between 50 and 100 m, bearing sphericity of less than 5 m, and bearing surface finish in the range 10 to 50 nm should provide satisfactory bearing performance. Wear is reduced at the lower end of each range provided tolerances can be maintained. There is disagreement on the preferred alloy for M/M bearings and, although the choice appears to be less important than bearing fit and finish. Metasul bearings were introduced in 28- and 32-mm femoral head diameters with 28mm heads being used most widely. Perhaps this choice was due to the prevalence of the 28-mm head size with M/P devices. With an M/M bearing, a larger head size can be recommended to increase range of motion and minimize neck-cup impingement. In addition, laboratory and theoretical studies indicate that there is a greater probability of developing full-film lubrication with larger head sizes because of the increased relative sliding velocity of the bearing surfaces achieved with a larger bearing diameter. The trend since the late 1990s has been to introduce femoral head sizes larger than 32 mm for M/M hip prostheses. However, the somewhat surprising finding of higher metal ion levels for M/M surface replacement devices may place an upper limit on this diameter.

Oscar Vazquez

211.What is the most significant disadvantage of ceramics in orthopaedic implants? 12345Low wear resistance Low elastic modulus Low toughness Weak under compressive load Poor wettability

Ceramics, such as alumina oxide or zirconium oxide, were introduced to orthopaedics because of their superior wear characteristics in the laboratory. Some studies have shown wear rates as up to 50% less that standard metal counterparts (ans 1: low resistance is incorrect). This is due to the fact that ceramics are highly polished lowering the coefficient of friction and are much harder and less susceptible to third body wear and scratching. Ceramics also show superior lubrication properties (ans 5-poor wettability is incorrect). Ceramics are extremely hard and stronger under compressiove load when compared to standard metals (ans 2 and 4 incorrect). However, ceramics are also brittle and are susceptible to catastrophic failure. This is more true for alumina oxide. The best answer is thus number 3: low toughness as ceramics have higher likelihood of breakage. PREFERRED RESPONSE: 3

RECOMMENDED READING: Vaccaro AR (ed): Orthopaedic Knowledge Update 8. Rosemont, IL, American Academy of Orthopaedic Surgeons, 2005, pp 57-67.

212. A 20-year-old collegiate basketball player has had a sore shin for the past 6 months and is now unable to continue playing. A lateral radiograph of the tibia shows a horizontal radiolucent line in the anterior tibial cortex at the site of maximal tenderness. To return this athlete to sports as quickly as possible, management should consist of 1- non-weight-bearing with crutches. 2- partial weight bearing in a removable boot. 3- excisional biopsy and casting. 4- intramedullary nailing. 5- onlay iliac crest bone grafting.

Stress fractures of the anterior tibia are an entity that although more uncommon than other types of stress fractures must be identified. Often occult, modalities such as MRI or bone scans may be needed for their diagnosis. Symptoms start as pain at the end of excersize and progress to sooner in routines, then to both during and after, then to continuous pain (such as is implied in this case). According to the Bruckner reference, there are many reports describing diverse treatment modalities including pneumatic bracing, non-weightbearing, and electical stimulation all with varying success depending on the study sited. In essence, one can find a paper that will support a given choice of treatment. The key to this case is return to play. Lets look at the answers. Number 3: excisional biopsy and casting would not be an initial treatment and can be thrown out, as can be bone grafting (number 5), as this too would more likely be used in the case of nonunion after failed initial treatment. A first step to fracture management is not excision. Numbers 1: NWB w/ crutches and 2: Partial weightbearing in a removable boot can be options and may lead to healing but would take up to over six weeks and would lead to a long delay from sporting activity. That leaves number 4: intramedullary nailing. Given that the nail would share the load and that there is good cortical apposition (it is a transverse fracture) early weight bearing and progression to athletic activity can be achieved more expeditiously. In the case report by Bruckner, they discuss a case where they were able to treat a patient with multiple stress fracture in a single tibia successfully with an IMN. They also describe several references that delineate excellent outcomes with IMN for tibial stress fractures. PREFERRED RESPONSE: 4 RECOMMENDED READINGS: DeLee JC, Drez D Jr, Miller MD (eds): Orthopaedic Sports Medicine, ed 2. Philadelphia, PA, WB Saunders, 2003, pp 2155-2182. Brukner P, Fanton G, Bergman AG, et al: Bilateral stress fractures of the anterior part of the tibial cortex: A case report. J Bone Joint Surg Am 2000;82:213-218.

213. A 22-year-old man injured his ankle 12 months ago and has continued, significant posteromedial ankle pain that is relieved with injection. An MRI scan is shown in Figure 84. Surgical management should consist of 1- autologous chondrocyte transplantation. 2- retrograde drilling and bone grafting. 3- excision of the loose fragment. 4- osteoarticular transfer from the knee. 5- fragment excision and drilling/micropicking.

Image

The MRI shows a defect in the dome of the talus consistent with an osteochondral lesion. The patients history is also consistent with an OCD as most of them occur following prior ankle injury (98% of lateral and 70% of medial OCD lesions). The fact that the pain is relieved by injection shows that his pain is caused by intraarticular pathology. Originally thought of as ischemic lesions, OCDs are more likely the result of compression injuries to the talar body. They can range from simple compression and incomplete lesions to complete/nondisplaced or complete/displaced. Treatment for small compression and incomplete lesions is nonoperative with casting and nonweightbearing, followed by progressive weightbearing with length of treatment 1216 weeks. For complete lesions the gold standard is fragment incision and drilling which can be done open or arthroscopically. Thus the answer here is number 5. Number 1: autologous chondrocyte transplantation is for cartilage only defects and has not been proven to be successful for OCDs. Answer 2 retrograde drilling and bone grafting is incorrect because it excludes excison of the lesion. For healing to occur for more advanced lesions the underlying fibrous tissue must be excised. Number 3: excision of the loose fragment alone is incorrect as drilling is necessary to provide vascular channels to lead to healing. Number 4: osteoarticular transfer from the knee is reserved for larger lesions but has been reported with some success in the literature. It is by no means the standard of care. This question should have gone to my man, Schachter- please refer to his JAAOS article on OCD lesions. PREFERRED RESPONSE: 5 RECOMMENDED READINGS: Tol JL, Struijs PA, Bossuyt PM, et al: Treatment strategies in osteochondral defects of the talar dome: A systematic review. Foot Ankle Int 2000;21:119-126. Barnes CJ, Ferkel RD: Arthroscopic debridement and drilling of osteochondral lesions of the talus. Foot Ankle Int 2003;8:243-257.

214. Figures 85a and 85b show the radiographs of a 17-year-old boy who has had ankle pain for the past 2 months. A biopsy specimen is shown in Figure 39c. CD99 immunohistochemistry staining of the tumor is positive. After staging, what is the next most appropriate step in management? 1- Chemotherapy 2- Curettage, culture, and antibiotics 3- Curettage and cementation 4- Wide resection 5- Radiation therapy

Images

Histology

In this case, we have a small round blue cell tumor in a young person. Small round blue cell tumors can be eosinophilic granuloma, ewings sarcoma, pnet, infection, lymphoma, and myeloma. The latter two are usually in older patients. The fact that the blue cells are of the same size is characteristic of Ewings. CD99 staining, although sometimes present in other tumors, is strongly indicative for Ewings. The pattern of staining may be described as a diffuse staining with an olympic ring like pattern. The xray also shows a permeative leasion which is consistent with the diagnosis of Ewings. Now that we have the diagnosis of Ewings we can answer the question of how is it treated. Ewings is treated with chemotherapy followed by wide resection, which leaves us with answer 1: chemotherapy as an initial step. Ewings sarcoma is a small round blue cell tumor that is arises in the pelvis, long bones, and ribs (although can be seen anywhere) of teenagers (85% between the ages of 5 and 20). It is usually characterized clinically by a painfull mass and swelling. Radiographs are characterized by a highly pemeative growth pattern with onion skin periosteal reaction, usually located in diaphysis or metaphysis. Ewings is characterized by the t(11;22) (q24;12) translocation. Histologically there are sheets of small round blue cells of equal size with hyperchromatic nuclei. Treatement is as described above and prognosis depends on metastasis. Metastatic disease is usually to the chest and other long bones and has a 5 year survival of 30%. Local disease has a 5 year survival of 65-70%.PREFERRED RESPONSE: 1 RECOMMENDED READINGS: Sluga M, Windhager R, Lang S, et al: The role of surgery and resection margins in the treatment of Ewings sarcoma. Clin Orthop 2001;392:394-399. Toni A, Neff JR, Sudanese A, et al: The role of surgical therapy in patients with nonmetastatic Ewings sarcoma of the limbs. Clin Orthop 1993;286:225-240.

215. Figure 86 shows the appearance of the tibial component at the time of revision total knee arthroplasty. The most likely reason for this appearance is that the polyethylene was sterilized using which of the following methods? 1- Ethylene oxide 2- Gamma irradiation in an inert atmosphere 3- Gamma irradiation in air 4- Gamma irradiation in a vacuum 5- Gas plasma

Image

The figure in this case shows tibial component changes due to degradation of the polyethylene. Polyethylene (PE) wear and degradation are related to 3 major factors, (1) PE manufacturing, (2) post processing steriliztion, and (3) shelf life. The question stem points us to PE sterilization. PE steilization can be either energic (radiation) or nonenergic (no irradiation). Irradiation can be low dose (2.5-4.5 Mrad) or high dose (5-15 Mrad) and is conducted to sterilize the PE while also allowing crosslinking. Crosslinking provides improved resistence to adhesive and abrasive wear. Higher doses of radiation provide more crosslinking. To prevent oxidation of the PE and accelerated degradation, irradiation is done in oxygen free environments such as in an inert environment. Oxidation of PE causes molecular chain scission, which causes accelerated PE wear and failure. Ans 3: gamma irradiation in air (has oxygen) is therefore the correct answer. Ethylene oxide (ans 1) and gas plasma (ans 5) are nonenergic forms of sterilization and do not cause crosslinking or the formation of radicalsPREFERRED RESPONSE: 3

RECOMMENDED READING: McNulty DE, Liao YS, Haas BD: The influence of andsterilization method on wear performance of the low contact stress total knee system. Orthopedics 2002;25:S243-S246.

Michael Baskies

216. A 3-year-old child has septic arthritis of the hip and osteomyelitis of the ipsilateral proximal femur. What is the most likely mechanism accounting for these findings? 1- Spread of the infection from the proximal femoral metaphysis into the hip joint 2- Spread of the infection from the proximal femoral epiphysis into the hip joint 3- Spread of the infection from the hip joint into the proximal femoral metaphysis 4- Simultaneous hematogenous infections of the proximal femoral epiphysis and hip joint 5- Immunocompromised host allowing for concomitant septic arthritis and osteomyelitis

PREFERRED RESPONSE: 1
Bacteria gain access into the hip joint cavity by two principal mechanisms, either directly via hematogenous spread or indirectly from the proximal femoral metaphysis. Septic arthritis can occur in joints in which the metaphysis is intra-articular (i.e. hip, shoulder, ankle). Approximately 15% of all cases of septic arthritis in children are secondary to bacterial osteomyelitis. According to Perlman et al., 42% of children with acute hematogenous osteomyelitis will develop adjacent septic arthritis. The clinician treating osteomyelitis must maintain a high index of suspicion for adjacent joint involvement.

RECOMMENDED READINGS: Song KM, Sloboda JF: Acute hematogenous osteomyelitis in children. J Am Acad Orthop Surg 2001;9:166-175. Perlman MH, Patzakis MJ, Kumar PJ, et al: The incidence of joint involvement with adjacent osteomyelitis in pediatric patients. J Pediatr Orthop 2000;20:40-43. Sucato DJ, Schwend RM, Gillespie R: Septic arthritis of the hip in children. J Am Acad Orthop Surg 1997;5:249-260.

217. When comparing multiply injured patients with and without foot injuries, the SF-36 scores in those patients with foot injuries show a 1- greater mean score. 2- lower mean score. 3- lower general health score. 4- greater vitality score. 5- lower mental health score.

PREFERRED RESPONSE: 2
According to Turchin et al., the outcomes of multiply injured patients with foot injuries was worse than the outcomes of multiply injured patients without foot injuries in five of the eight components of the SF-36 score. Multiply injured patients with foot injuries had a lower mean score on the SF-36.

RECOMMENDED READINGS: Turchin DC, Schemitsch EH, McKee MD, et al: Do foot injuries significantly affect the functional outcome of multiply injured patients? J Orthop Trauma 1999;13:1-4. Tran T, Thordarson D: Functional outcome of multiply injured patients foot injury. Foot Ankle Int 2002;23:340-343.

218. Which of the following amputation levels has the lowest energy cost of walking? 1- Transtibial 2- Transfemoral 3- Midfoot 4- Syme (through the ankle) 5- Through the knee

PREFERRED RESPONSE: 4
Ankle disarticulation (Symes amputation) allows for direct load transfer and is rarely complicated by late residual limb ulcers or wound breakdown. It provides a stable gait pattern and has the lowest energy cost of walking. It should be performed in one stage, and a patent tibialis posterior artery is necessary to ensure healing. The malleoli and metaphyseal flares are removed from tibia and fibula but the remaining tibial articular surface is retained. The heel pad is secured to the tibia anteriorly through drill holes or posteriorly by securing the Achilles tendon. Waters compared performance of Symes amputees with transtibial and transfemoral amputees. The Waters study found that performance was significantly better the lower the level of amputation. Symes amputees display faster gait velocities and have a lower energy cost of prosthetic walking.
RECOMMENDED READINGS: Pinzur M: Comparing midfoot amputations vs Symes in the traumatic patient. Biomechanics 1997;4:77-82. Rheinstein J, Yanke J, Marzano R: Developing an effective prescription for a lowerextremity prosthesis. Foot Ankle Clin 1999;4:113-138. Waters RL, Perry J, Antonelli D, Hislop H: Energy cost of walking of amputees: the influence of the level of amputation. JBJS-Am 1976;58A:42-46.

220. What design feature of cementless femoral stems limits osteolysis of the distal femur when used in total hip arthroplasty? 1- Fills the diaphysis of the femur 2- Fills the metaphysis of the femur 3- Collared 4- Circumferentially coated 5- Distally fluted

PREFERRED RESPONSE: 4
From the OKU Hip & Knee 3: Cementless stems without circumferential coating have essentially been abandoned. Because of the higher rate of failure of proximal patch-coated stems, circumferential coating has become a critical and uniform design feature in currently available cementless stems. The extremely low rate of distal osteolysis in circumferentially coated stems has confirmed that this feature limits the distal access of polyethylene debris and lowers rates of loosening.

RECOMMENDED READING: Sinha RK, Dungy DS, Yeon HB: Primary total hip arthroplasty with a proximally porous-coated femoral stem. J Bone Joint Surg Am 2004;86:1254-1261. Savory CG, Hamilton WG, Engh Sr CA, Della Valle CJ, Rosenberg AG, Galante JO. Hip Designs. In: OKU: Hip and Knee Reconstruction 3. Eds. Barrack RL, Booth RE, Lonner JH, McCarthy JC, Mont MA, Rubash HE. Rosemont, IL: AAOS, 2006.

Alexis Chiang

221. Following a revision anterior cervical diskectomy and fusion performed through a left-sided approach, the patient notices drooping of the left eyelid. Ophthalmologic examination will most likely reveal 12345pupillary dilation and hyperhidrosis. pupillary dilation and anhidrosis. pupillary constriction and anhidrosis. pupillary constriction and hyperhidrosis. normal pupil function and anhidrosis.

PREFERRED RESPONSE: 3
Horners syndrome (miosis, anhidrosis, ptosis) can occur during anterior exposure of the lower cervical area via damage to the sympathetic chain. Subperiosteal dissection of the longus colli muscle usually prevents this.
RECOMMENDED READINGS: Bertalanffy H, Eggert HR: Complications of anterior cervical discectomy without fusion in 450 consecutive patients. Acta Neurochir (Wien) 1989;99:41-50. Bono CM, Garfin SR: Anterior approaches of the cervical spine, in Bradford DA, Zdeblick TA (eds): Master techniques in orthopaedic surgery: The Spine. Philadelphia, PA, Lippincott Williams & Wilkins, 2004, pp 3-28.

222. Which of the following structures is the primary antagonist to the anterior tibial tendon?

12345-

Flexor hallicus longus Peroneus longus Peroneus brevis Posterior tibial tendon Achilles tendon

PREFERRED RESPONSE: 2

The importance of the relationship of the peroneus longus and ant tib muscles is seen with the cavus foot. The primary deformity is a plantarflexed 1st MT, due to a hyperactive peroneus longus. The plantarflexed 1st MT causes a higher arch as well as the medial aspect of the forefoot striking the ground during foot-flat and heel-rise portions of gait. The hindfoot can then no longer evert at subtalar and midtarsal articulations as is does immediately after heel strike. Because of the tripod effect, the foot and ankle tip into varus and lateral ankle instability is felt. Fixed heel varus then develops along with pronated forefoot. These pts usually have a tight gastrocnemius as well (as evidenced by more ankle DF with knee flexed than with knee extended). Plantarflexion of the ankle leads to a chronic muscle imbalance. The peroneus longus has a more advantageous vector pull in plantarflexing the first ray than the tib ant does in dorsiflexion. This imbalance is one reason why mild cavus deformities may progressively worsen in pts with equinus deformities.
RECOMMENDED READINGS: Manoli A II, Graham B: The subtle cavus foot, the underpronator. Foot Ankle Int 2005;26:256-263. Silver RL, de la Garza J, Rang M: The myth of muscle balance: A study of relative strengths and excursions of normal muscles about the foot and ankle. J Bone Joint Surg Br 1985;67:432-437.

223. What nerve is most likely to be injured in the establishment of an anterior portal during hip arthroscopy?

12345-

Lateral femoral cutaneous Superior gluteal Inferior gluteal Femoral Sciatic

PREFERRED RESPONSE: 1 The anterior portal in hip arthroscopy is established by the intersection of a line drawn distally from the ASIS with a transverse line across the superior margin of the GT. This portal is approximately 45 deg in a cephalad direction and 30 toward the midline. The anterolateral and posterolateral portals are positioned directly over the superior aspect of the trochanter at its anterior and posterior borders. The anterolateral portal is established first because it lies most centrally in the safe zone for hip arthroscopy. The LFCN lies close to the anterior portal. The femoral artery and nerve are well medial to the anterior portal. The sciatic nerve is posterior to the posterolateral portal.
RECOMMENDED READING: Byrd JW: Hip arthroscopy: The supine position. Instr Course Lect 2003;52:721-730.

224. Which of the following is considered the most likely predictor of eventual lower extremity amputation in patients with diabetes mellitus? 12345Hemoglobin A1c level Decreased oxygen tension levels Decreased ankle-brachial index scores Diabetic foot ulceration Charcot foot changes

PREFERRED RESPONSE: 4 Diabetic foot ulcers precede 85% of nontraumatic LE amps. The primary risk factor for developing ulcers or deep infection is loss of protective sensation from peripheral neuropathy. Once a diabetic develops a foot ulcer, their likelihood of LE amp increases 8 times! One third of those patients who undergo a transtibial amp will be dead within 2 years.

RECOMMENDED READINGS: Koval KJ (ed): Orthopaedic Knowledge Update 7. Rosemont, IL, American Academy of Orthopaedic Surgeons, 2002, pp 565-578. Brodsky JW: The diabetic foot, in Coughlin MJ, Mann RA (eds): Surgery of the Foot and Ankle, ed 7. St Louis, MO, Mosby-Year Book, 1999, pp 895-969.

225. A low-grade, soft-tissue sarcoma of the thigh is completely excised. What is the first most common location for recurrence?

12345-

Thigh Lymph nodes Skeleton Liver Lung

PREFERRED RESPONSE: 1 Local recurrence is the most common area of recurrence. Other points from this paper=size of tumor in greatest dimension at time of diagnosis is a strong predictor for survival. This is especially evident for the largest tumors (>15cm) who had a 37% survival @ 3 yrs versus a 63% survival for the rest of patients.
RECOMMENDED READING: Rougraff B, Sandler A: Karnofsky performance score as a predictor of survival in soft tissue sarcoma. Clin Orthop 2002;397:196-203.

Eric Fulkerson

226. Achondroplasia results from abnormal chondrocyte functions due to FGF-3 receptor in the growth plate. What region is affected in the growth plate as shown in Figure 87? 1- A 2- B 3- C 4- D 5- E

PREFERRED RESPONSE: 3
Sadly for all of us, this is a painful memorize it or lose question. Basic science researchers need something to do, so they identify receptors for diseases that are unlikely to ever have a cure. Kudos for trying though In case you dont know what you are looking at, C demarks the zone of proliferation, which is affected by the FGF-R defect. Simple to memorize: achondroplasia = short = no proliferation. If only there were a clever mnemonic for FGF-R.

RECOMMENDED READING: Buckwalter JA, Einhorn TA, Simon SR (eds): Orthopaedic Basic Science: Biology and Biomechanics of the Musculoskeletal System, ed 2. Rosemont, IL, American Academy of Orthopaedic Surgeons, 2000, pp 77-109.

227. Figure 88 shows the current clinical photograph of a 6-year-old boy who underwent treatment of a right elbow fracture 2 years ago. What is the most likely cause of the deformity? 1- Malunion 2- Growth arrest 3- Asymmetric growth stimulation 4- Postsurgical infection 5- Congenital

PREFERRED RESPONSE: 1
Cheeky bastards are always trying to trick us! The knee-jerk response to this question is: growth arrest. However, three striking clues in the question stem should cause you pause: 6 year old boy, large deformity, 2 year-old injury. This little tyke is young and has not yet reached his adolescent growth spurt. A cubita vara (gunstock for you laymen) deformity of say30 degrees would be unlikely after 2 years in a kid this age if he started from anatomic reduction. This realization should lead you to malunion. The other options? You just cant go there without more info... The references dont help you answer this question, only your knowledge of orthopaedics will.
France J, Strong M: Deformity and function in supracondylar fractures of the humerus in children variously treated by closed reduction and splinting, traction and percutaneous pinning. J Pediatr Orthop 1992;12:494-498. Keenan WN, Clegg J: Variation of Baumanns angle with age, sex and side: Implications for its use in radiological monitoring of supracondylar fracture of the humerus in children. J Pediatr Orthop 1996;16:97-98. Pirone AM, Graham HK, Krajbich JI: Management of displaced extension-type supracondylar fractures of the humerus in children. J Bone Joint Surg Am 1988;70:641-650.

228. If a child receives a defective copy of chromosome 15, with deletion at 15q11q13, from the mother, the result is Angelman syndrome. If a defective copy of chromosome 15, having that same deletion, is contributed through the father, the result is Prader-Willi syndrome. This is an example of 1- genomic imprinting. 2- sex-linked dominant transmission. 3- X-linked dominant transmission. 4- Robertsonian translocation. 5- uniparental translocation.

PREFERRED RESPONSE: 1
How do I keep getting these? Another either you know it or you dont question. All that basic science stuff seems to have a tendency for nipping one in the buttocks. Im pretty sure I guessed on this one, and just as likely to have missed it. Anywho, humans are diploid organisms with two copies of each gene (allele). For some genes, expression only occurs from one copy (imprinting). Expression is determined by parental origin of these genes, meaning that some are only expressed if inherited from the father and vice-versa. Imprinting is accomplished via DNA methylation and histone modifications during epigenetic processes in the ovum and sperm.
If neither copy of 15q13 has paternal imprinting, the result is Prader-Willi syndrome (characterized by hypotonia, obesity, and hypogonadism). If neither copy has maternal imprinting, the result is Angelman syndrome (characterized by epilepsy, tremors, and a perpetually smiling facial expression).

Wikipedia is a great resource if you need a quick review of the incorrect choices.

RECOMMENDED READINGS: Gabriel KR: Genetics II: Genetic syndromes with orthopaedic manifestations, in Fitzgerald RH Jr, Kaufer H, Malkani AL (eds): Orthopaedics. St Louis, MO, Mosby, 2002, pp 1256-1270. Cole WG: Genetic aspects of orthopaedic conditions, in Morrissy RT, Weinstein SL (eds): Lovell & Winters Pediatric Orthopaedics, ed 5. Philadelphia, PA, Lippincott Williams & Wilkins, 2000, pp 157-176. Vogels A, Fryns JP: The Prader-Willi syndrome and the Angelman syndrome. Genet Couns 2002;13:385-396. Driscoll DJ: Genomic imprinting in humans. Mol Genet Med 1994;4:37-77.

229. A 24-year-old man hits a deer while driving unrestrained. On arrival at the emergency department, his cervical spine is cleared, a CT of the head is normal, and he has a blood pressure of 90/60 mm Hg. He has no intraabdomial injuries. After administration of 3 L of crystalloid, his blood pressure rises to 120/90 mm Hg. In addition to a pneumothorax and a chest tube, examination reveals a right, closed midshaft femoral fracture and a left, open both bone forearm fracture with a 1.5 cm dorsal wound and visible bone. Treatment should consist of 1- external fixation of the femoral fracture, irrigation and debridement, and external fixation of the forearm fracture. 2- external fixation of the femoral fracture, irrigation and debridement, and plating of the forearm fracture. 3- intramedullary nailing of the femur, irrigation and debridement, and external fixation of the forearm fracture. 4- intramedullary nailing of the femur, irrigation and debridement, and plating of the forearm fracture. 5- femoral traction, irrigation and debridement, and casting of the forearm fracture.

PREFERRED RESPONSE: 4
Some salient points from the history: young healthy patient with appropriate response to crystalloid, workup of large organ injuries and appropriate preoperative management (neg abd CT, chest tube placement). When presented with this scenario, definitive management of orthopaedic injuries is reasonable. This is particularly true with clean open fractures and long bone fractures. The below refs concern open BBFX: Definitive ORIF provided injury is not grossly contaminated and proper I&D performed.

RECOMMENDED READINGS: Chapman MW, Gordon JE, Zissimos AG: Compressionplate fixation of acute fractures of the diaphyses of the radius and ulna. J Bone Joint Surg Am 1989;71:159-169. Moed BR, Kellam JF, Foster RJ, et al: Immediate internal fixation of open fractures of the diaphysis of the forearm. J Bone Joint Surg Am 1986;68:1008-1017.

230. A 63-year-old woman who previously underwent a Lisfranc (tarsometatarsal) amputation of her right foot now reports walking on the lateral border of her foot. Observation of her gait reveals that she inverts her foot. What is the most likely explanation for her altered gait? 1- Failure to lengthen the Achilles tendon 2- Failure to osteotomize the second metatarsal to preserve medial cuneiform stability 3- Failure to preserve the soft-tissue envelope (peroneus brevis, tertius, and plantar fascia) around the fifth metatarsal base 4- Failure to transfer the anterior tibial tendon to the toe extensors 5- Failure to transfer the posterior tibial tendon to the cuboid

PREFERRED RESPONSE: 3
Again, the references are not terribly helpful for answering this question. However, your knowledge of anatomy should be all that is required to answer this question. Failure of the foot to evert suggests that the muscles for this function are not working. This should lead to answer 3. For the other options: 1 wouldnt result in lateral border walking but equinus, 2 is just wack, 4 would probably yield claw toes. Although 5 could result in the described deformity, necessity to transfer the PT would not occur if the lateral tissues were preserved.

RECOMMENDED READINGS: Chapman MW, Gordon JE, Zissimos AG: Compressionplate fixation of acute fractures of the diaphyses of the radius and ulna. J Bone Joint Surg Am 1989;71:159-169. Moed BR, Kellam JF, Foster RJ, et al: Immediate internal fixation of open fractures of the diaphysis of the forearm. J Bone Joint Surg Am 1986;68:1008-1017.

RECOMMENDED READINGS: Koval KJ (ed): Orthopaedic Knowledge Update 7. Rosemont, IL, American Academy of Orthopaedic Surgeons, 2002, pp 127-137. Early JS: Transmetatarsal and midfoot amputations. Clin Orthop 1999;361:85-90. The objective of lower extremity amputation surgery is to create a viable, functional residual limb to maximize patient mobility and independence. When part or all of the forefoot is lost to trauma, infection or gangrene, and the hindfoot is viable, every attempt should be made to preserve as much foot function as possible. The use of the transmetatarsal level is common. In the past, amputations through the Lisfranc and Chopart's joint lines involved significant complication rates. With improvements in patient selection and surgical technique, these two amputation levels are viable options to consider when attempting salvage of the hindfoot structures.

Konrad Gruson

231. A 35-year-old man has pain, limited motion, and swelling of his middle finger after sustaining a volar puncture wound 3 days ago. Examination reveals a flexed resting posture, pain with passive stretching, and tenderness along the flexor tendon sheath. Appropriate management should consist of

12345-

outpatient management with oral antibiotics. IV antibiotics alone. surgical drainage over the entire length of the finger. surgical drainage via small incisions and intraoperative sheath irrigation with a catheter. needle aspiration of the flexor tendon sheath with continuous irrigation and oral antibiotics.

PREFERRED RESPONSE: 4

This patients clinical evaluation includes all of the Kanavel signs of suppurative flexor tenosynovitis, including: 1) swollen digit; 2) flexed posture of the digit; 3) pain with passive extension of the digit; and 4) pain along the flexor sheath. If symptoms have been present for less than 24 hours, a trial of IV antibiotics may be attempted with low threshold for operative I&D of the flexor sheath. Given the duration of this patients symptoms, choices (1), (2) and (5) may be eliminated.
When irrigating the tendon sheath, one wants to avoid disrupting the sheath unnecessarily, because this can lead to tendon adhesions, especially in the setting of infection.

Therefore, a small incision may be used, either midaxial or a variation of the Bruner incision, to approach the sheath. The sheath is incised and purulence usually egresses. Another incision is made in the volar palm similar to that used for a trigger release. A catheter is introduced and is passed proximally towards the A1 pulley; the sheath is then irrigated. Post-operatively, the patient should be treated with intravenous antibiotics.

Lille et al. retrospectively reviewed their results of simple intraoperative lavage alone compared with a combination of intraoperative lavage followed by continuous postoperative lavage. The authors found no difference in hospital length of stay, operative time, postoperative complications or ultimate digit ROM. They concluded that the elimination of

postoperative lavage would be helpful in reducing patient discomfort and allowing patients to initiate earlier motion and hopefully avoid tendon sheath adhesions.

RECOMMENDED READINGS: Trumble TE (ed): Hand Surgery Update 3: Hand, Elbow, & Shoulder. Rosemont, IL, American Society for Surgery of the Hand, 2003, pp 433-457. Lille S, Hayakawa T, Neumeister MW, et al: Continuous postoperative catheter irrigation is not necessary for the treatment of suppurative flexor tenosynovitis. J Hand Surg Br 2000;25:304-307.

232.During a two-incision minimally invasive total hip arthroplasty, the socket is inserted without any complications. While inserting the proximally coated nonmodular femoral stem, a fracture is noted at the junction between the proximal and middle thirds of the femur, roughly at the anticipated tip of the stem. What is the best course of action? 1Extend the incision with the patient in the supine position, insert the femoral stem, and use a cable-plate for internal fixation of the femur. Insert a longer stem through the minimally invasive approach, and reduce the fracture under radiographic control as the stem is being advanced. Close both incisions prior to inserting the stem, reposition

2-

3-

4-

5-

the patient into the lateral position, extend the femoral incision into an extensile posterolateral approach to expose the hip joint and the fracture, and insert a longer fully porous-coated stem to bypass the fracture. Close both incisions prior to inserting the stem, reposition the patient into the lateral position, extend the femoral incision into an extensile posterolateral approach to expose the hip joint and the fracture, reduce the fracture, insert the proximally coated stem into the proximal femur, and use cerclage cables to fix the fracture. Continue the surgery with the patient in the supine position, insert the femoral stem into the femur under radiographic control, and use a minimally invasive locking plate technique to fix the fracture.

PREFERRED RESPONSE: 3 Interestingly, the referenced article has very little to do with this particular question, though it is a quick read. The question can be answered using common sense. When facing an impending disaster as in the above question, the answer is unlikely to require continuing the case under minimally invasive conditions. Thus, answers (2) and (5) are probably wrong. The next step is to decide whether to stick with the proximally coated stem or to move on to a long diaphyseal fitting stem. This scenario is essentially a Vancouver B2, that is, a periprosthetic femur fracture around the stem or around the tip, with the stem being loose. The stem is considered loose because no bony ingrowth has occurred yet. Therefore, the appropriate treatment would be

to insert a fully porous coated stem that will achieve distal fixation. The revision stem should bypass the fracture by two shaft diameters for optimum stability. Thus, the best answer is (3). If this patient sustained this fracture around a wellingrown arthroplasty done a while ago, an appropriate answer would be to place a cable plate after reducing the shaft. (answer 1)

RECOMMENDED READING: Fehring TK, Mason JB: Catastrophic complications of minimally invasive hip surgery: A series of three cases. J Bone Joint Surg Am 2005;87:711714.

233.A 37-year-old man is undergoing an anterior cervical diskectomy and fusion for a large central disk herniation. Following placement of an interbody strut into the disk space, motor-evoked potentials are lost in the right and left upper extremities. What is the next most appropriate step in management? 12345Close the wound and perform a laminectomy. Close the wound and obtain an emergent MRI. Perform a wake-up test. Remove the interbody strut. Insert an anterior cervical plate.

PREFERRED RESPONSE: 4 It stands to reason that if you lose your motor evoked potentials after placing the interbody strut, the next most appropriate step would be to remove the strut (4). Choices (1) and (2) make little sense because youre delaying treatment for a potentially devastating neurological injury. Similarly, one would not continue with placement of the cervical plate (5) if there were any hint of cord injury. Hilibrand et al. compared somatosensory evoked potential (SSEP) monitoring with motor evoked potential (MEP) monitoring during cervical spine surgery to determine the temporal relationship of potential changes between the two methods. They hypothesized that the motor potentials would be superior in this region of the spine and alert the surgeon

to changes earlier than with SSEPs. They found that SSEPs were only 25% sensitive, though 100% specific; in comparison, MEPs were 100% sensitive and specific. They recommended use of MEPs for spine surgeons when operating on the cervical spine.
RECOMMENDED READINGS: Bose B, Sestokas AK, Schwartz DM: Neurophysiological monitoring of spinal cord function during instrumented anterior cervical fusion. Spine J 2004;4:202-207. Hilibrand AS, Schwartz DM, Sethuraman V, et al: Comparison of transcranial electric motor and somatosensory evoked potential monitoring during cervical spine surgery. J Bone Joint Surg Am 2004;86:1248-1253.

234. Medial displacement calcaneal osteotomy and flexor digitorum longus transfer to the navicular is considered the treatment of choice for which of the following patients? 1234A 24-year-old male runner with posterior tibial tenosynovitis and no hindfoot deformity A 27-year-old man with cerebral palsy and a spastic cavovarus foot A 35-year-old man with a painful cavovarus foot secondary to Charcot-Marie-Tooth disease A 60-year-old woman with a painful flexible adultacquired flatfoot deformity secondary to posterior tibial tendon dysfunction A 75-year-old woman with a rigid adult-acquired flatfoot deformity secondary to posterior tibial tendon dysfunction

5-

PREFERRED RESPONSE: 4

The medial displacement calcaneal osteotomy with transfer of the FDL to the navicular is the treatment of choice for symptomatic, acquired flexible flatfoot in the elderly. In general, flexible deformities require osteotomies and tendon transfers, whereas fixed deformities with degenerative changes require fusions in the accepted positions. Thus, the answer would be choice (4). The patient in choice (5) would require a triple arthrodesis.
The stages of posterior tibial tendon insufficiency are: stage I: pain along the tract of the PTT, no deformity, and hindfoot moves into varus when performing a heel rise; stage II: flexible deformity with hindfoot valgus and loss of the medial

longitudinal arch when bearing weight. Patients have significant pain when attempting to toe rise and may in fact be unable to perform a single heel rise because of pain; stage III: fixed hindfoot valgus, may have prominent talar head medially; stage IV: the talar orientation leads to a stress fracture of the distal fibula. The patient in choice (4) would be stage II. The treatment for choice (1) would be activity modification, medial arch supports and perhaps NSAIDs. The patient in choice (2) would probably receive a medial column lengthening, Achilles tendon lengthening and possible a transfer of the anterior tibial tendon laterally to balance the forefoot. Post-op, the patient would need bracing to prevent recurrence of the deformity. Finally, the patient in choice

(3), if his hindfoot varus corrected with Coleman block testing, would be treated with peroneus longus transfer to the brevis, first metatarsal dorsiflexion osteotomy, possible plantar fascia release and correction of any claw toe deformities. RECOMMENDED READINGS: Myerson MS, Badekas A, Schon LC: Treatment of stage II posterior tibial deficiency with flexor digitorum longus tendon transfer and calcaneal osteotomy. Foot Ankle Int 2004;25:445-450. Myerson MS: Adult acquired flatfoot deformity. J Bone Joint Surg Am 1996;78:780-792. Mizel MS, Miller RA, Scioli MW (eds): Orthopaedic Knowledge Update: Foot and Ankle 2. Rosemont, IL, American Academy of Orthopaedic Surgeons, 1998, pp 253277.

235. A 35-year-old runner who has had intermittent calf pain for the past 4 months now reports cramping, coolness of the leg, and occasional paresthesias of the foot. Symptoms are worse with walking and relieved with running. Examination reveals that dorsiflexion and plantar flexion of the ankle accentuate the patients symptoms. What diagnostic study will best confirm the diagnosis? 12345Electromyography and nerve conduction velocity studies Vascular studies Compartment pressure measurement Noncontrast MRI Noncontrast CT

PREFERRED RESPONSE: 2 The terms cramping and coolness of the leg should clue us in that this problem is vascular in nature, hence making choice (2) the correct answer for making the diagnosis. This patient has what appears to be popliteal artery entrapment syndrome (PAES), a condition whereby the popliteal artery is occluded by anatomic variations in the insertion of the medial gastrocnemius or in the course of the popliteal artery itself. It is thought to be a rare cause of exertional calf pain in the young athlete. While occasional tingling may be present, objective neurological findings are uncommon. Symptoms are accentuated following exercise, particularly with the knee extended and the ankle joint dorsiflexed, further constricting arterial flow. Post-exercise arteriography

would likely demonstrate medial deviation of the artery, segmental occlusion, stenosis with knee extension and ankle dorsiflexion, and post-occlusion dilatation. Severe cases can result in leg ischemia.
Nerve entrapment syndromes of the foot, including tarsal tunnel syndrome and deep peroneal nerve entrapment under the inferior extensor retinaculum usually are diagnosed clinically because EMG and NCV are normal (choice 1). Patients with exertional compartment syndrome usually have worsening symptoms during exercise, in contrast to the above patient. These patients have compartment pressure >15 mm Hg at rest and >30 mm Hg 1 minute following exercise. The treatment for

PAES depends on the offending lesion, but includes release of the artery from the tight medial gastrocnemius head.
RECOMMENDED READINGS: Andrish J: The leg, in DeLee JC, Drez D Jr, Miller MD (eds): Orthopedic Sports Medicine: Principles and Practice, ed 2. Philadelphia, PA, WB Saunders, 2003, pp 2155-2181.

Lysens RJ, Renson LM, Ostyn MS, et al: Intermittent claudication in young athletes: Popliteal artery entrapment syndrome. Am J Sports Med 1983;11:177-179.

Reuven Minkowitz

236. Bisphosphonates are clinically used for osteoporosis or osteolytic metastatic bone cancers. What is the common mechanism of pharmacologic action? 1- Increased mineralization of bone matrix 2- Increased apoptosis of osteoclasts 3- Increased production of alpha-v-beta-3 integrin by osteoclasts 4- Potentiation of PTM-related peptide effect on osteoblasts 5- Amplification of BMP-2 signal

PREFERRED RESPONSE: 2
All clinically used bisphosphonates can stimulate apoptosis (cell suicide) in osteoclasts On a molecular level, nitrogen-containing bisphosphonates (alendronate and risedronate), were shown to directly inhibit the enzyme farnesyl diphosphate synthase required for geranylgeranyl diphosphate production. These relevant proteins control cellular shape and ruffled-border formation in the osteoclast. Take that to the bank!
From Rodan GA, Reszka AA: Osteoporosis and bisphosphonates. J Bone Joint Surg Am 2003;85:8-12.

237.Submuscular plating techniques, when compared to conventional plating techniques, offer


1less compromise to medullary and periosteal perfusion. 2- greater compromise to medullary and periosteal perfusion. 3- no difference with regard to periosteal perfusion only. 4- no difference with regard to medullary perfusion only. 5- no difference with regard to periosteal and medullary perfusion.

PREFERRED RESPONSE: 1
Makes sense. In the reference the authors state sub muscular plating better preserves the perforating arteries and nutrient artery, subsequently improving periosteal and medullary perfusion.
Farouk O, Krettek C, Miclau T, et al: Minimally invasive plate osteosynthesis: Does percutaneous plating disrupt femoral blood supply less than the traditional technique? J Orthop Trauma 1999;13:401-406.

238. Hip pointers are contusion injuries to what area of the hip?
12345Iliac wing Greater trochanter Hip joint Quadriceps muscle Femoral shaft

PREFERRED RESPONSE: 1
An iliac crest contusion (hip pointer) is the result of a direct blow to the iliac crest, typically occurring in football, but it can result from a fall in any activity.

Point tenderness, ecchymosis, and muscle spasm are common findings with this injury. The iliac crest has multiple muscle origins and insertions, including the internal and external oblique, latissimus dorsi, paraspinal muscles, and fascia from the gluteus medius muscle. Avulsion of these muscles can be difficult to differentiate from a contusion. Resisted contraction of these muscles, if avulsed, may exacerbate symptoms to a greater degree than would a contusion.
Treatment initially focuses on minimizing swelling and bleeding with compression and ice. The hip can be rested in a position of comfort and the patient should ambulate with crutches. Gradual stretching and strengthening of adjacent muscles should be performed before resuming premorbid activity. Padding modifications may allow earlier participation in contact sports. Many physicians would also consider the judicious use of local anesthetic injection to allow continued participation in sports.
Anderson K, Strickland SM, Warren R: Hip and groin injuries in athletes. Am J Sports Med 2001;29:521-533.

239. A bone marrow biopsy is a routine part of the staging work-up for what type of sarcoma?
12345Osteosarcoma Synovial sarcoma Ewings sarcoma Fibrosarcoma Chondrosarcoma

PREFERRED RESPONSE: 3
In Ewings sarcoma, cytogenetic analysis of the biopsy specimen should be obtained to evaluate the t(11;22) translocation. Bone marrow biopsy should be considered to complete the workup. Serum LDH has been shown to have prognostic value as a tumor marker. This is right out of NCCN Clinical Practice Guidelines in Oncology Bone Cancer

Ewing's sarcoma family of tumors includes Ewings sarcoma, primitive neuroectodermal tumor (PNET), Askins tumor, PNET of bone, and extraosseous Ewings sarcoma. Ewings sarcoma and primitive neuroectodermal tumor (PNET) are small round cell neoplasms developing in bone and soft tissue, defined by a chromosomal translocation, t(11;22)(q24;q12) and closely related variants. Ewings sarcoma is poorly differentiated and is also characterized by strong expression of cell-surface glycoprotein CD99 If Ewings is suspected as a diagnosis, the patient should undergo complete staging prior to biopsy. This should include CT of the chest, plain radiographs of the primary site as well as a CT or MRI of the entire involved bone or area, bone scan, and PET scan as clinically indicated.

240. A 45-year-old woman has a limb-length discrepancy and angular deformity of the knee. Radiographs of the right hip joint and both knee joints are shown in Figures 89a and 89b. A biopsy specimen is shown in Figure 89c. What genetic abnormality is responsible for this condition?

12345-

FGF-3 Cyclic AMP RANKL TGF-1 EXT-1

PREFERRED RESPONSE: 2
Fibrous dysplasia is a benign skeletal lesion that can involve one or more bones. Its etiology has been linked to an activating mutation of Gs and the downstream effects of the resultant increase in cAMP. Polyostotic lesions tend to be larger than monostotic lesions and result in more skeletal complications, including pain, deformity, and fractures. Some patients with polyostotic bone involvement also have skin lesions and endocrinopathies (McCune-Albright disease) or multiple myxomas (Mazabraud syndrome). Monostotic lesions frequently are discovered incidentally and require only clinical observation. Confirmatory biopsy is indicated if the radiographic findings are not characteristic of fibrous dysplasia. Bisphosphonates have been shown to offer pain relief and improve skeletal strength in appropriately selected patients with either polyostotic or monostotic fibrous dysplasia. Occasionally, operative treatment is needed to correct deformity or to prevent or stabilize a pathologic or fatigue fracture. Cortical allograft or intramedullary fixation of the entire long bone provides the best material properties for patients who require operative intervention. If that is too wordy, remember Chinese characters on histo and shepherds crook on xrays (proximal femur) THE CHINESE SHEPERD WENT TO CAMP DiCaprio MR, Enneking WF: Fibrous dysplasia: Pathophysiology, evaluation, and treatment. J Bone Joint Surg Am 2005;87:1848-1864.

FGF- 3 Achondroplasia RANKL (Receptor Activator for Nuclear Factor B Ligand) activates osteoclasts, TGF beta induces apoptosis in numerous cell types EXT Osteochondroma- loss of reg protein for Indian hedgehog EXT1 & 2 multiple exostosis

Brian Neri

241. What is the optimal position for a shoulder arthrodesis?


1- 10 degrees of abduction, 45 degrees of forward flexion, 20 degrees of internal rotation 2- 25 degrees of abduction, 30 degrees of forward flexion, 10 degrees of external rotation 3- 45 degrees of abduction, 30 degrees of forward flexion, 25 degrees of internal rotation 4- 45 degrees of abduction, 90 degrees of forward flexion, 20 degrees of internal rotation 5- 70 degrees of abduction, 30 degrees of forward flexion, 30 degrees of internal rotation

PREFERRED RESPONSE: 3
The reference is a report on 43 shoulder arthrodeses performed over a 37yr period--the range of position they sought was abd 20-60, flexion 20-40, IR 0-50 (no ER)--whats important is only enough abduction for hygiene and no ER. Using these guidelines, only answer 3 is correct. Just remember, the most generally accepted position is 20-40 abd/flex/ir.

RECOMMENDED READING: Ruhmann O, Schmolke S, Bohnsack M, et al: Shoulder arthrodesis: Indications, technique, results and complications. J Shoulder Elbow Surg 2005;14:38-50.

242. Use of cock-up wrist splints for carpal tunnel syndrome is most useful for

12345-

improving nerve conduction velocities. improving thenar muscle weakness. improving sensation in the median nerve distribution. improving nocturnal symptoms. delaying the need for surgery.

PREFERRED RESPONSE: 4
In a prospective randomized study of 147 EMGconfirmed CTS receiving either nocturnal splint x 6 weeks or CTR, at 18 months f/u surgery was significantly better than splinting in all outcome measures. They concluded that splinting is therefore only indicated for pts waiting for surgery or refusing surgery. However if you happened to deeply sift through this JAMA article (and what orthopaedist hasnt been caught doing that once or twice), you would find that only nocturnal symptoms were improved by splinting.
RECOMMENDED READINGS: Gerritsen AA, de Vet HC, Scholten RJ, et al: Splinting vs surgery in the treatment of carpal tunnel syndrome: A randomized controlled trial. JAMA 2002;288:1245-1251. Omer GE Jr: Median nerve compression at the wrist. Hand Clin 1992;8:317-324.

243. Osteosarcoma most commonly metastasizes to the lung. What is the second most common site of metastasis?

12345-

Lymph node Brain Bone Spleen Liver

PREFERRED RESPONSE: 3
Osteosarcoma-10-25y/o, metaphyseal, distal femur->prox tibia->prox humerus, permeative, assc w/ Retinoblastoma and Li-Fraumeni syndrome. Metastasizes: Lung Bone Pleura, Pericardium, Kidney, Adrenals (Very rarely lymph tissue)

RECOMMENDED READINGS: Menendez LR (ed): Orthopaedic Knowledge Update: Musculoskeletal Tumors. Rosemont, IL, American Academy of Orthopaedic Surgeons, 2002, pp 21-27. Gibbs CP Jr, Weber K, Scarborough MT: Malignant bone tumors. Instr Course Lect 2002;51:413-428.

244. The natural history of cervical spondylolytic myelopathy is most commonly described as

1- waxing and waning. 2- rapid deterioration. 3- stable over time. 4- stable for long periods with stepwise deterioration. 5- improvement after an initial episode of severe symptoms.

PREFERRED RESPONSE: 4
This is asked every year by OITE, spine attendings etccervical myelopathy equals stepwise deterioration. The classic report by Clarke & Robinson found 75% stepwise deterioration, 20% slow steady deterioration and 5% rapid deterioration.
RECOMMENDED READINGS: Emery SE: Cervical spondylotic myelopathy: Diagnosis and treatment. J Am Acad Orthop Surg 2001;9:376-388. Lees F, Turner JW: Natural history and prognosis of cervical spondylosis. Br Med J 1963;28:1607-1610. Clarke E, Robinson PK: Cervical myelopathy: A complication of cervical spondylosis. Brain 1956;79:486-510.

245. Early regional osteopenia beneath a plate after dynamic compression plating is a direct result of

12345-

stress shielding. implant-bone contact. implant material. circumferential periosteal stripping. quiescent infection.

PREFERRED RESPONSE: 2
An implant which contacts bone along an extended surface, I.e. a plate fitting the radius with a smooth undersurface, inhibits blood reaching or leaving the bone and causes regional osteopenia. A Limited Contact Plate (LCP) has a trapezoidal cross-section and reduces the area of contact while a LISS plate doesnt contact bone at all and therefore completely preserves periosteal blood supply and avoids this finding.
RECOMMENDED READING: Perren SM: Evolution of the internal fixation of long bone fractures: The scientific basis of biological internal fixation: Choosing a new balance between stability and biology. J Bone Joint Surg Br 2002;84:1093-1110.

Jason Park

246. Figures 90a through 90d show the injury a 26year old man sustained in a motor vehicle accident 12 days ago. The soft-tissue envelope is and has been unremarkable, and no neurovascular deficits exist. Definitive treatment should consist of 1hybrid external fixation. transarticular external fixation. 3- dual plate fixation with two surgical approaches. 4- lateral locking plate fixation with a single surgical approach. 5- dual plating with a single surgical approach.

PREFERRED RESPONSE: 3 Since this question asks for definitive treatment, we can eliminate both external fixation choices. Considering we want a near anatomic articular reduction, an ex-fix just wont cut it. Dual plating through a single approach would strip too much soft tissue leading to too many complications such as wound complications, infections, non-unions. Though the answer is dual plating through 2 surgical approaches, their given literature seems to say that a locking plate is a viable option (LISS plate). Carlson states that with these posterior medial condyle fractures, direct visualization and stabilization through a posteromedial incision gives the best reduction. Barei also agrees that the 2 incision approach leads to minimal complications with anatomic reduction and fixation. The last 2 given articles look at the LISS plate system and shows that it is just as good as the two plate/ 2-incision technique.

RECOMMENDED READINGS: Carlson DA: Posterior bicondylar tibial plateau fractures. J Orthop Trauma 2005;19:73-78. Barei DP, Nork SE, Mills WJ, et al: Complications associated with internal fixation of high-energy bicondylar tibial plateau fractures utilizing a two-incision technique. J Orthop Trauma 2004;18:649-657. Stannard JP, Wilson TC, Volgas DA, et al: The less invasive stabilization system in the treatment of complex fractures of the tibial plateau: Short term results. J Orthop Trauma 2004;18:552-558. Gosling T, Schandelmaier P, Marti A, et al: Less invasive stabilization of complex tibial plateau fractures: A biomechanical evaluation of a unilateral locked screw pate and double plating. J Orthop Trauma 2004;18:546-551.

247. A healthy human knee contains approximately how many milliliters of synovial fluid? 123450 2 10 25 50

PREFERRED RESPONSE: 2 Straight memorization question. The AAOS Orthopaedic Basic Science bible says: Approximately 1 to 5 ml of fluid is contained in a healthy human knee joint. RECOMMENDED READING: Buckwalter JA, Einhorn TA, Simon SR (eds): Orthopaedic Basic Science: Biology and Biomechanics of the Musculoskeletal System, ed 2. Rosemont, IL, American Academy of Orthopaedic Surgeons, 2002, pp 134-180.

248. A 45-year-old woman who works as a typist reports numbness and tingling in both hands by the end of her work day. In the morning. she generally feels well. Throughout the day she notes increasing fatigue and numbness in both hands that is worse in the little and ring fingers, but all fingers are affected. Specific motor and sensory testing and electrodiagnostic tests are normal. A neck examination is normal. Radiographs of the cervical spine reveal normal alignment. What is the most likely diagnosis?
12345Central cervical disk herniation Thoracic outlet syndrome Bilateral carpal tunnel syndrome Bilateral shoulder impingement Bilateral ulnar nerve entrapment

PREFERRED RESPONSE: 2 This answer is obtained by a process of elimination. Central cervical disk herniation would give more a picture of a upper motor neuron disease with myelopathy. Plus, they tell us that there is nothing wrong with the neck exam and xrays. B/L CTS doesnt seem to fit since all fingers are affected, esp the little and ring fingers. B/L ulnar nerve entrapment, although more applicable, still doesnt fit as again the whole hand is affected and the EMGs are normal. B/L shoulder impingement is a throw away choice as the shoulder is unaffected. Finally, this leaves us with thoracic outlet syndrome. The vagueness of the symptoms also steer us more to this answer. With the feeling of numbness/tingling/ fatigue, it could be associated with a vascular issue as opposed to nervous. The articles are reviews of TOS. RECOMMENDED READING: Leffert RD: Thoracic outlet syndromes. Hand Clin 1992;8:285-297. Rayan GM: Thoracic outlet syndrome. J Shoulder Elbow Surg 1998;7:440-451.

249. A 46-year-old woman has a 3-cm soft-tissue mass in the subcutaneous forearm. With the expectation that the patient had a lipoma, a marginal excision of the lesion is performed. The pathology report shows it to be a high-grade sarcoma. Staging studies confirm no evidence of metastases, and a postexcision MRI with contrast shows no evidence of residual tumor. What is the next most appropriate step in management?
1234Amputation Wide surgical re-excision Radiation therapy without additional surgery Observation with follow-up MRI studies, followed by re-excision and radiation therapy if there is a recurrence 5- Chemotherapy

PREFERRED RESPONSE: 2 This question leads us down the path that an inadequate procedure was done for a malignant soft tissue sarcoma. Chem and radiation are only used as adjunctive treatments to a wide excision. Just because it is subcutaneous should not cloud our judgement. Peabody shows that 43% who had re-excision after an incisional biopsy or marginal resection had residual disease. This eliminates the observation choice. A subcutaneous sarcoma is likely to recur after a limited operation. Although amputation is an option, its probably a little too extreme. Plus, 90% do well after wide re-excision. So amputation is unnecessary. Wide surgical re-excision is the best choice. RECOMMENDED READINGS: Peabody TD, Monson D, Montag A, et al: A comparison of the prognoses for deep and subcutaneous sarcomas of the extremities. J Bone Joint Surg Am 1994;76:1167-1173. Gibbs CP, Peabody TD, Mundt AJ, et al: Oncological outcomes of operative treatment of subcutaneous soft-tissue sarcomas of the extremities. J Bone Joint Surg Am 1997;79:888-897.

250. A 6-year-old girl sustained a 3-cm laceration over the anteromedial aspect of her knee after falling onto a rock. Examination of the wound reveals that the joint capsule is visible, but no articular cartilage is seen. Radiographs of the knee are normal. Management should consist of
12345injection of contrast material into the wound, followed by radiographic studies. MRI of the knee. arthroscopic examination. a saline load test. stress radiographs of the knee under conscious sedation.

PREFERRED RESPONSE: 4
The saline load test is a definitive way of determining if a joint capsule has been compromised. Voit, et al looked at 50 periarticular lacerations suggestive of joint penetration. The surgeon predicted whether the joint was penetrated or not, then injected the joint. He found that there was a 39% false positive and 43% false negative rate. In otherwords, 40% of the time, treatment was altered (or the wrong treatment would have been given). Since we cant accurately diagnose traumatic arthrotomy clinically, we should perform a saline load test for any suspicious periarticular laceration. RECOMMENDED READINGS: Voit GA, Irvine G, Beals RK: Saline load test for penetration of periarticular lacerations. J Bone Joint Surg Br 1996;78:732-733. Leffers D: Dislocations and soft tissue injuries of the knee, in Browner BD (ed): Skeletal Trauma, ed 1. Philadelphia, PA, WB Saunders, 1992, pp 1717-1743.

Sarah Pettrone

251. What is the most appropriate treatment method for a displaced vertical medial malleolus fracture?
123Casting in supination Casting in pronation Two oblique screws from the tip of the medial malleolus 4- Tension band fixation with two oblique Kirschner wires from the medial malleolus and 18-gauge wire 5- Medial antiglide plate

PREFERRED RESPONSE: 5 RECOMMENDED READING: Browner BD, Jupiter JB, Levine AM, et al: Skeletal Trauma. Philadelphia, PA, WB Saunders, 1992, pp 1898-1941. In a supination adduction injury the medial malleolus fracture is usually vertically oriented. These fractures must be fixed with screws placed perpendicular to the fracture. In addition, because this fracture is subject to vertical shear forces it is important to butress the fracture with an antiglide or butress plate.

252. A stress fracture is most prone to the development of a nonunion or delayed union in which of the following areas?
12345Third metatarsal shaft Femoral shaft Anterior tibial cortex Distal third of the fibula Proximal third of the fibula

PREFERRED RESPONSE: 3 RECOMMENDED READINGS: Boden BP, Osbahr DC, Jimenez C: Low-risk stress fractures. Am J Sports Med 2001;29:100-111. Boden BP, Osbahr DC: High-risk stress fractures: Evaluation and treatment. J Am Acad Orthop Surg 2000;8:344-353. Stress fractures are common overuse injuries that are frequently seen in athletes and military recruits. While the reported incidence of stress fractures in the general athletic population is less than 1%, the incidence in runners may be as high as 15%.47 In a review of 320 athletes with stress fractures, the tibia was the most commonly involved bone (49.1%), followed by the tarsals (25.3%) and the metatarsals (8.8%). The Boden classifies stress fx as high risk or low risk base on potential for nonunion. High-risk stress fractures have a predilection for progressing to complete fracture, delayed union, or nonunion; therefore, they present treatment challenges and require a more aggressive approach. These problematic stress fractures include those in the femoral neck (tension side), patella, anterior cortex of the tibia, medial malleolus, talus, tarsal navicular, fifth metatarsal, and great toe sesamoids. When radiographs are positive, use of other imaging modalities is usually not necessary. In athletes who have chronic pain and normal findings on initial radiographs, a bone scan, CT scan, or MR study is recommended. Because of the high complication rate, high-risk stress fractures should be treated like acute fractures. In athletes, the tibial shaft is the most common location of stress fractures. When the diagnosis is delayed, nonoperative treatment is less successful. Tibial stress fractures may occur at any site along the shaft of the bone, but are most frequently encountered in the posteromedial cortex (compression side). A less common, but more problematic, location of tibial stress fractures is the anterior cortex of the middle third of the tibia. Initial treatment of stress fractures of the anterior midportion of the tibia is generally a trial of rest, with or without immobilization, for a minimum of 4 to 6 months. If the radiographs reveal chronic changes, such as a wide fissure, surgical intervention becomes necessary.

253. Which of the following is considered the most important factor in achieving patient compliance?
12345Socioeconomic level of the patient Education level of the patient Gender Shared decision-making Physician charisma

PREFERRED RESPONSE: 4 RECOMMENDED READINGS: DiMatteo MR, Reiter R: Enhancing medication adherence through communication and informed collaborative choice. Health Communication 1994;4:253-265. DiMatteo MR: The physician-patient relationship: Effects on the quality of health care. Clin Obstet Gynecol 1994;37:149-161. This is just a common sense, touchy-feely question. Unfortunately, the library does not carry articles or abstracts from Health Communication or Clinical Obstetrics and Gynecology from before 1999.

254. A 45-year-old man with a transtibial amputation is an avid golfer. What type of prosthetic foot is recommended? 12345Solid ankle cushioned heel Stationary ankle flexible endoskeleton Dynamic response Articulated dynamic Polycentric hydraulic

PREFERRED RESPONSE: 4 RECOMMENDED READINGS: Koval KJ (ed): Orthopaedic Knowledge Update 7. Rosemont, IL, American Academy of Orthopaedic Surgeons, 2002, pp 127-137. Romo HD: Specialized prostheses for activities: An update. Clin Orthop 1999;361:63-70. The solid ankle cushioned heel is one of the earlier foot assemblies. It consists of a semirigid wooden keel surrounded by a resilient material concentrated at the heel. The use of this design is limited to home ambulators.

Dynamic response feet store energy at heel strike and transmit the forces to the keel during heel-off, providing recoil. Because they provide a better spring for running and jumpin, these prosthetic foot designs are recommended for more active individuals. Articulated foot assemblies proved motion at the anatomic location of the ankle, better accommodating uneven surfaces. Articulated dynamic response feet have the unique ability to accomodate uneven surfaces and are ideal for outdoor activities on uneven terrain. The pylon is the interphase between the foot assembly and the socket. It has either an endoskeleton or exoskeletal design. Endoskeltal pylons consist of a rigid central support surrounded by a soft cosmetic covering. This situation is ideal for early prosthetic rehab. A polycentric hydraulic prosthetic is a knee design that allow adjustment of candence response by changing resitance to knee flexion.

255.A 4-year-old boy with a right clubfoot was treated with the Ponseti technique as an infant. His family has been adherent with brace wear. Examination reveals a straight lateral border of the foot and 25 degrees of ankle dorsiflexion. The foot supinates with active dorsiflexion of the ankle. Treatment should consist of 1transfer of the anterior tibialis muscle to the lateral cuneiform. split posterior tibialis tendon transfer. serial casting for 6 weeks with long leg casts and with the foot abducted and pronated. physical therapy. posterior medial release of the ankle and subtalar joints.

2345-

PREFERRED RESPONSE: 1
RECOMMENDED READINGS: Ponseti IV: Treatment of congenital clubfoot. J Bone Joint Surg Am 1992;74:448-454. Kuo KN, Hennigan SP, Hastings ME: Anterior tibial tendon transfer in residual dynamic clubfoot deformity. J Pediatr Orthop 2001;21:35-41. Ippolito E, Farsetti P, Caterini R, et al: Long-term comparative results in patients with congenital clubfoot treated with two different protocols. J Bone Joint Surg Am 2003;85:1286-1294. Ponseti: To prevent more relapses, the tendon of the anterior tibial muscle is transferred to the third cuneiform if this muscle tends to supinate the foot strongly after correction has been obtained. The anterior tibialis muscle has a strong supinatory action when the abnormal relationship between the talus and the calcaneus has not been connected fully, as indicated by smaller talocalcaneal angles. My clinical experience indicates that, in a large proportion of club feet that are treated with this procedure, the correction of the varus deformity of the heel that was obtained with manipulation and application of casts can be maintained and the AP talocalcaneal angle will become normal. The transferred anterior tibial tendon dorsiflexes the foot in neutral. Kuo: Residual forefoot adduction and supination deformities with functional problems and difficulty with shoe wear may occur during the course of management of the congenital clubfoot. Between 1975 and 1988, 55 patients with 71 feet who had residual dynamic clubfoot deformity underwent anterior tibial tendon transfer. There were 42 full anterior tibial tendon transfers (FTs) and 29 split anterior tibial tendon transfers (STs). The average age of the patient at the time of the procedure was 6 years. The age at surgery for FT was 5.3 years compared with 7.1 years in ST. The clinical appearance of the feet improved in both groups, according to Garceaus criteria. The range-of-motion improvement was noted in dorsiflexion and eversion. There was an increase of eversion strength of both groups by 1.5 grades. The radiographic improvement was noted in both forefoot adduction and supination. Although the FT group had a little better statistical data than did the ST group, the ST group had better preservation of inversion function. Long-term follow-up studies of adults who had been treated for congenital clubfoot as infants are rare.

Ippolito: The purpose of this study was to review and compare the longterm results in two groups of patients with congenital clubfoot treated with two different techniques. In both groups, treatment was started within the first three weeks of life by manipulation and application of toe-to-groin plaster casts, with a different technique in each group. At the end of the manipulative treatment, a posteromedial release was performed when the patient was between eight and twelve months of age in the first group and a limited posterior release was performed when the patient was between two and four months of age in the second group. In the second group, use of Ponsetis manipulation technique and cast immobilization followed by an open heel-cord lengthening and a limited posterior ankle release gave much better long-term results than those obtained in the first group, treated with our manipulation technique and cast immobilization followed by an extensive posteromedial release of the foot. In our hands, this operation did not prevent relapse, and neither cavovarus nor forefoot adduction was completely corrected.

Charles Preston

256. Which of the following associated injuries is most commonly noted during arthroscopy after acute shoulder dislocation?
1- Anterior labral tears 2- Posterior labral tears 3- Complete rotator cuff tear 4- Hill-Sach lesions 5- Superior labrum anterior and posterior (SLAP) lesions

To quote the swiss. Of these 212 patients, 184 (87%) patients had anterior glenoid labral tears, 168 (79%) patients had ventral capsule insufficiency, 144 (68%) patients had Hill-Sachs compression fractures, 116 (55%) patients had glenohumeral ligament insufficiency, 30 (14%) patients had complete rotator cuff tendon tears, 26 (12%) patients had posterior glenoid labral tears, 14 (7%)
patients had superior labrum anterior and inferior lesions.

Preferred response 1

257. What structure separates the greater and lesser sciatic notches into their respective foramen?
12345Iliolumbar ligament Sacrotuberous ligament Sacrospinous ligament Ventral sacroiliac ligament Transverse perineal muscle

Anatomy question Preferred response 3

258. The working length of a bicortical screw is defined as the


12345pitch of the screw. sum of thickness of the near and far bone cortices. intramedullary distance traversed by the screw. length of screw projecting beyond the far cortex. diameter of bone traversed by the screw.

This is an AO definition. The diagram summarizes the definition. The paper referenced is excellent.

259. Which of the following best defines the conditioning term, periodization?
1234Planned variation in exercise intensity and volume Ability of an athlete to meet the demands of sport Percent of maximum functional capacity Total amount of exercise performed in a specific time

5-

Training adaptation to the environment

This answer comes from OKU 8 P. 152, paragraph 1. The chapter is the medical care of athletes. Periodization has the goal of obtaining maximal performance based on the concept of progressive overload. Multiple studies have demonstrated this method as superior to techniques where intensity and duration are kept constant. Preferred response 1

260. Examination of a 52-year-old man with a 4-week history of hip pain reveals a destructive, lytic proximal femoral lesion. He has no history of malignancy. What is the most likely diagnosis?
12345Lymphoma Metastatic carcinoma Chondrosarcoma Osteosarcoma Pagets sarcoma of bone

PREFERRED RESPONSE: 2
Ask Jim, or Neri

RECOMMENDED READINGS: Rougraff BT, Kneisl JS, Simon MA: Skeletal metastases of unknown origin: A prospective study of a diagnostic strategy. J Bone Joint Surg Am 1993;75:1276-1281. Rougraff BT: Indications for operative treatment: Symposium for the management of metastatic disease. Orthop Clin North Am 2000;31:567-576.

Aaron Schacter

261. The risk of hepatitis C disease transmission in musculoskeletal freshfrozen allograft transplantation is reported to be 12341 in 10,000. 1 in 100,000. 1 in 750,000. 1 in 1,500,000.

PREFERRED RESPONSE: 2
Risk of: HBV: 1/63,000 HCV: 1/100,000 HIV : 1/650k 1,000,000 HIV : (bone) 1/1.5 million Currently, donors are screened serologically for hbv, hcv, and hiv. Tissue is sterilized with irradiation (1.0 3.0 megarad). Higer doses are required for recalcitrant bacteria, but destroy important graft characteristics. As for other infections, various studies have reported up to 3.6% culture + instances of staph species.

RECOMMENDED READINGS: Friedlaender GE: Appropriate screening for prevention of infection transmission by musculoskeletal allografts. Instr Course Lect 2000;49:615-619. Joyce MJ, Joyce DM: Musculoskeletal allograft tissue banking and safety, in Laurencin CT (ed): Bone Graft Substitutes. West Conshohocken, PA, ASTM International, 2003, pp 30-67.

262. Mechanisms of action for low-intensity pulsed ultrasound in the stimulation of fracture healing include all of the following EXCEPT 12345creates fracture site nanomotion. increases the temperature at the fracture site. increases osteocalcin and VEGF production. interacts with and activates integrins. causes functional chelation of intracellular calcium.

Low intensity pulsed ultrasound accelerates the rate of fracture healing in humans. It works by multiple (potential) mechanisms. Its molecular effects are mediated by its effects on intracellular calcium signaling. In animal models, ultrasound appears to alter the time course or the sequence of gene expression of several genes, notably aggrecan, which is a proteoglycan involved in enchondral osteogenesisfracture models in animals. Low-intensity ultrasound elevates intracellular calcium in cultured chondrocytes and stimulates endochondral bone formation. It alters potassium flux across the cell membrane (via ans 4) in cultured thymocytes, 6 and it modulates adenyle cyclase activity and TGF-b synthesis with resultant increases in osteocalcin and VEGF (ans 3) in osteoblastic cell linescultures. In addition to modulating gene expression, ultrasound may enhance angiogenesis and increase blood flow around the fracture. In addition, the acoustic pressure waves produce micro-stress fields resulting in a mechanical response of the bone, analogous to the phenomena described by Wolfs law (ans 1).

47 Small temperature fluctuations (<1 8C) appear at the fracture site as a result of the conversion of ultrasound energy to heat (ans 2). Some enzymes, such as collagenase, are exquisitely sensitive to these small temperature variations, thus, ultrasound may also facilitate some enzymatic processes. As per the first reference, chelating intracellular calcium inhibits the stimulatory effect of ultrasound on proteoglycan synthesis (ans 5 is wrong). Duh. PS, the second reference had nothing to do with this question, but fyi LIPU did not make osteopenic postmenopausal women less osteopenic. PREFERRED RESPONSE: 5
RECOMMENDED READINGS: Parvizi J, Parpura V, Greenleaf JF, et al: Calcium signaling is required for ultrasound-stimulated aggrecan synthesis by rat chrondrocytes. J Orthop Res 2002;20:51-57. Leung KS, Lee WS, Chang WH, et al: Lack of efficacy of low-intensity pulsed ultrasound on prevention of postmenopausal bone loss evaluated at the distal radius in older Chinese women. Clin Orthop 2004;427:234-240.

263. The risk factors for nonunion with nonsurgical treatment of the fracture shown in Figure 45b includes all of the following EXCEPT 12345displacement of more than 6 mm. posterior displacement. younger age. a fracture gap of more than 2 mm. treatment delay.

PREFERRED RESPONSE: 3
Topic: nonunion of type II odontoid fx. The reference was a review study of 69 patients with type II odontoid fx tx with halo vest. 46% bony union. Found to correlate with nonunion were fx gap greater than 1 mm (ans 4), posterior displacement greater than 5 mm (ans 1 and 2, kind of), delayed start of treatment greater than 4 days (ans 5), and posterior redisplacement greater than 2 mm. according to this study, anterior dislocation, gender, and age (ans 3) were unrelated to this nonunion. Kind of counterintuitive, but true. Being old doesnt always suck, I suppose.

RECOMMENDED READINGS: Koivikko MP, Kiuru MJ, Koskinen SK, et al: Factors associated with nonunion in conservatively treated type-II fractures of the odontoid process. J Bone Joint Surg Br 2004;86:11461151. Vaccaro AR (ed): Orthopaedic Knowledge Update 8. Rosemont, IL, American Academy of Orthopaedic Surgeons, 2005, pp 509-526.

264. All of the following enhance stability of a locked plate construct for a distal femoral fracture EXCEPT
1- use of bicortical rather than unicortical screws. 2- use of a longer plate. 3- divergent rather than parallel screws. 4- an increased number of metaphyseal screws. 5- deliberate 10-degree axis deviation of the locked screws relative to the hole threads.

PREFERRED RESPONSE: 5
The correct answer from this question comes from the results of reference #2. They reported that a locking head screw exhibited high stability with a moderate axis deviation in the angle of insertion up to 5 degrees, but found a significant decrease in stability with increasing axis deviation >5 degrees (ans 5). The remaining factors, bicortical vs unicortical (musculoskeletal res. Ctr), longer plate (internal ex-fix near-near, far-far yadda yadda), divergent screws, and increased number of metaphaseal screws (sounds good) all improve LCP construct stability.
RECOMMENDED READINGS: Haidukewych GJ: Innovations in locking plate technology. J Am Acad Orthop Surg 2004;12:205-212. Kaab MJ, Frenk A, Schmeling A, et al: Locked internal fixator: Sensitivity of screw/plate stability to the correct insertion angle of the screw. J Orthop Trauma 2004;18:483-487. Sommer C, Babst R, Mller M, et al: Locking compression plate loosening and plate breakage: A report of four cases. J Orthop Trauma 2004;18:571-577.

265. In a healthy adult patient with an acute amputation of the upper extremity, all of the following are considered absolute indications for replantation EXCEPT the
1- thumb at the level of the metacarpal neck. 2- index finger at the level of the proximal phalanx. 3- complete hand through the palm. 4- forearm through the radial metaphysis. 5- index, middle, and ring fingers through the proximal phalanges.

PREFERRED RESPONSE: 2
Again, an answer that comes directly from a cited article that Im sure everyone read in their OITE preparation. The first reference describes one authors 20 year experience of limb replants (293 upper extremity replants). The second reference reports on the results after single finger replant. Based upon inferir results with respect to range of motion and function, amputations of a single finger proximal to the insertion of the FDS (prox phx) are seldom indicated (ans 2). Remember, for function a person generally requires one of the dexterous digits (index or middle) and one of the power digits (ring or small). As for the remaining answers: Ans 1: always try to replant the thumb. As a salvage, pollicization of the index can be performed, vs. a stinky toe transfer Ans 3: all digits lost, attempt replant so as to avoid devastating functional loss Ans 4: replant the forearm if the remnant/amputated part isnt destroyed in the process. Remember, forearm transplants have been performed and the recipients regain some gross motor/grip function but do not regain fine dexterity (a warm prosthesis) Ans 5: multiple digit amputation, replant regardless of zone
RECOMMENDED READINGS: Tamai S: Twenty years experience of limb replantation: Review of 293 upper extremity replants. J Hand Surg Am 1982;7:549-556. Urbaniak JR, Roth JH, Nunley JA, et al: The results of replantation after amputation of a single finger. J Bone Joint Surg Am 1985;67:611-619.

Pete Tsai

266. Scapular fractures that are the result of high-energy trauma are associated with all of the following EXCEPT 12345lung injury. rib fractures. clavicle fracture. lower extremity fracture. blunt thoracic aortic injury.

PREFERRED RESPONSE: 5 I think I got this wrong. Anyways, the article data was from a trauma registry from which they formed two groups- one with scapular fractures and the rest. They compared the concurrent injuries in the two groups and came up with some conclusions. Essentially those people sustaining a scapular fracture have a ~50% incidence of rib fracture, ~28% incidence of pneumothorax and ~15% incidence of pulmonary contusion. Clavicle fractures ~39% of the time and lower ext fracture in ~7% of cases. Blunt thoracic aortic injury was never mentioned.

RECOMMENDED READINGS: Brown CV, Velmahos G, Wang D, et al: Association of scapular fractures and blunt thoracic aortic injury: Fact or fiction? Am Surg 2005;71:54-57. Veysi VT, Mittal R, Agarwal S, et al: Multiple trauma and scapula fractures: So what? J Trauma 2003;55:1145-1147.

267. A 44-year-old woman has a symptomatic bunion and a painful plantar callus under the second metatarsal head that continues to limit her activity and shoe wear despite the use of shoe modifications. Radiographs show an intermetatarsal angle of 18 degrees, a hallux valgus angle of 38 degrees, and a first metatarsal that is shorter than both the second and third metatarsals. When considering surgical options, each of the following first metatarsal procedures are appropriate for this patient EXCEPT

12345-

Z osteotomy (Scarf). oblique proximal osteotomy (Ludloff). distal chevron osteotomy. proximal crescentic osteotomy. Lapidus procedure.

PREFERRED RESPONSE: 3 The articles have really not that much to do with the question. Basically for IM angles between 9-13 degrees (mild deformity) one can consider a distal osteotomy or a soft tissue procedure. For IM angles from 13-20, one needs a proximal osteotomy. Severe deformity with IM angles >20 needs proximal osteotomy and soft tissue procedures. The Scarf procedure is this:

Answers 2 and 4 are proximal osteotomies which allow greater correction, and the lapidus is is a fusion of a hypermobile tarsometatarsal joint, which would be appropriate in this amount of deformity.

RECOMMENDED READINGS: Jones S, Al Hussainy HA, Ali F, et al: Scarf osteotomy for hallux valgus: A prospective clinical and pedobarographic study. J Bone Joint Surg Br 2004;86:830-836. Weil LS, Borelli AN: Modified scarf bunionectomy: Our experience in more than 1000 cases. J Foot Surg 1991;30:609-622.

268. Herniated disks are associated with a spontaneous increase in production of all of the following, EXCEPT 12345nitric oxide. prostaglandin E2. interleukin-6. transforming growth factor-. tumor necrosis factor-.

PREFERRED RESPONSE: 4 The article is from 1997 and states after biochemical assay that normal intervertebral disks significantly increased their production of matrix metalloproteinases, nitric oxide, interleukin-6, and prostaglandin E2 in response to interleukin-1 a factor which increases in concentration with injury to the disk. Actually there was no part of the study that searched for TNF-alpha. Additionally, a cursory lit search shows that TGF beta has now been shown to increase in levels post injury. Anyhoo, the answer for this question was apparently #4. RECOMMENDED READING: Kang JD, Stefanovic-Racic M, McIntyre LA, et al: Toward a biochemical understanding of human intervertebral disc degeneration and herniation: Contributions of nitric oxide, interleukins, prostaglandin E2, and matrix metalloproteinases. Spine 1997;22:1065-1073.

269. Factors contributing to an apex anterior sagittal plane deformity when introducing an intramedullary nail to manage a proximal tibial fracture include all of the following EXCEPT 12345anterior starting hole. interlocking in flexion. posterior cortical comminution. posteriorly directed tibial nail. posteriorly placed blocking screw.

PREFERRED RESPONSE: 5
Going down the list. When you have a proximal tibial fracture, you dont want to be too anterior with the start, but honestly you dont have much leeway there. You do want the guidewire to be pretty anterior in the tibia however. Interlocking in flexion increases the flexion forces across the fracture. Posterior cortical comminution will make flexion deformity more possible because there is no posterior buttress. A posteriorly directed tibial nail will result in flexion because if the nail is heading posteriorly in the proximal piece and then into the distal piece, it will by definition be kicked up. Finally number 5 is the only one to make sense in that the posteriorly placed blocking screw will block the nail/guidewire from heading the wrong direction.
RECOMMENDED READINGS: Freedman EL, Johnson EE: Radiographic analysis of tibial fracture malalignment following intramedullary nailing. Clin Orthop 1995;315:25-33. Henley MB, Meier M, Tencer AF: Influences of some design parameters on the biomechanics of the unreamed tibial intramedullary nail. J Orthop Trauma 1993;7:311-319. Krettek C, Stephan C, Schandelmaier P, et al: The use of Poller screws as blocking screws in stabilising tibial fractures treated with small diameter intramedullary nails. J Bone Joint Surg Br 1999;81:963-968. Tornetta P III, Collins E: Semiextended position for intramedullary nailing of the proximal tibia. Clin Orthop 1996;328:185-189. Lang GJ, Cohen BE, Bosse MJ, et al: Proximal third tibial shaft fractures: Should they be nailed? Clin Orthop 1995;315:64-74.

270. All of the following are risk factors for wound complications following Achilles tendon repair EXCEPT 12345tobacco abuse. steroid use. female gender. diabetes mellitus. timing of surgery.

PREFERRED RESPONSE: 5
There were 17 wound complications in 164 patients (10.4%). Significant risk factors for development of wound complications included tobacco use (p < 0.0001), steroid use (p = 0.0005), and female sex (p = 0.0400). For those patients who had one or more of the following risk factors: diabetes, tobacco use, or steroid use; eight of 19 (42.1%) had a complication, compared with nine of 145 (6.2%) for those without risk factors present (p < 0.0001). There is no mention of timing of surgery. There you have it.

RECOMMENDED READING: Bruggeman NB, Turner NS, Dahm DL, et al: Wound complications after open Achilles tendon repair: An analysis of risk factors. Clin Orthop 2004;427:63-66.

Brian White

271.The pathophysiology of age-related disk degeneration involves all of the following EXCEPT

12-

345-

a relative change in proportions of keratan sulfate and chondroitin sulfate with an increase in the latter. fragmentation of large aggregated proteoglycans and an increase in small and nonaggregated proteoglycans. unchanged absolute quantity of collagen. a decrease in water content. a decrease in the absolute number of cells in the nucleus pulposus.

PREFERRED RESPONSE: 1 The intervertebral disk can be basically separated into the nucleus pulposus (gelatinous core, compressible, high GAG, low collagen), and the surrounding annulus fibrosis (high collagen, low GAG). The Disk is 85% water. With increasing age the disk degenerates, likely as a result of decreasing blood supply and nutrition. The following occurs with advancing age: decrease in water content, kerratin sulfate increases with age, decrease in cell number in nucleus pulposus, no change in collagen quantity, and fragmentation of large proteoglycans and aggrecans, and decreased proteoglycan concentration (these latter two account for decreased water content). RECOMMENDED READING: Boos N, Weissbach S, Rohrbach H, et al: Classification of age-related changes in lumbar intervertebral discs: 2002 Volvo award in basic science. Spine 2002;27:26312644.

272. A 30-year-old man sustained a calcaneal fracture 4 years ago that was treated nonsurgically. Posttraumatic arthritis has now developed. He elects to be treated with a subtalar bone block distraction arthrodesis. This procedure attempts to correct all of the following abnormalities commonly seen after calcaneal fractures EXCEPT
12345anterior ankle impingement. subfibular peroneal impingement. hindfoot valgus. limb shortening. subtalar arthritis.

PREFERRED RESPONSE: 3 Late deformities and problems that occur with calcaneal malunions are subtalar arthrosis, hindfoot varus, ankle impingement, subfibular peroneal impingement from lateral wall blowout, and loss of calcaneal height. Distraction bone block arthrodesis involves removing residual cartilage from the talus and calcaneus, distracting the joint and correcting the height and varus with a tricortical graft, and fusing the subtalar joint.
RECOMMENDED READINGS: Carr JB, Hansen ST, Benirschke SK: Subtalar distraction bone block fusion for late complications of os calcis fractures. Foot Ankle Int 1988;9:81-86. Bednarz PA, Beals TC, Manoli A II: Subtalar distraction bone block fusion: An assessment of outcome. Foot Ankle Int 1997;18:785-791. Rammelt S, Grass R, Zawadski T, et al: Foot function after subtalar distraction bone-block arthrodesis: A prospective study. J Bone Joint Surg Br 2004;86:659-668.

273. Demineralized bone matrix contains all of the following EXCEPT 12345collagen. bone morphogenetic proteins. transforming growth factor . mesenchymal precursor cells. residual calcium.

PREFERRED RESPONSE: 4 Demineralized bone matrix preparations vary widely, but overall they have mild osteoinductive properties and are mostly osteoconductive. They are different from BMPs which have much greater osteoinductive capacity. DBM preparations vary widely and are dependant on the quality of the donor cadaveric bone and the processing method. They contain collagen, some BMPs, tgf beta, and residual calcium. As it is not live tissue, and has been processed, it does not have mesenchymal precursor cells.
RECOMMENDED READING: Baumgaertner MR, Tornetta P III (eds): Orthopaedic Knowledge Update: Trauma 3. Rosemont, IL, American Academy of Orthopaedic Surgeons, 2005, pp 107-113.

274. The strength of flexor tendon repair can be increased by all of the following techniques EXCEPT 1- repair of the flexor tendon sheath. 2- increasing the size of the core suture. 3- increasing the number of core suture strands that cross the repair site. 4- adding epitendinous suture. 5- adding locking loops to the core suture

PREFERRED RESPONSE: 1 Flexor tendon review: Zones: 1: distal to FDS insertion 2. No mans land-Distal palmar crease to FDS insertion 3. Palm 4. Carpal tunnel 5. Distal Forearm Zone 2 injuries carry the worse prognosis because of adhesions. A2 and A4 pulleys are the critical pulleys. Increasing the caliber of the suture and increasing the number of strands improves tendon repair performance in terms of load to failure, gap formation and cyclic loading strength. Locking sutures and an epitendinous suture also increases the strength of the repair. Rehab: Children get immobilized. Multiple protocols exist for adults, but the goal is for early motion that protects the repair sufficiently to allow healing-typically this involves passive motion. RECOMMENDED READINGS: Trumble TE (ed): Hand Surgery Update 3: Hand, Elbow, & Shoulder. Rosemont, IL, American Society for Surgery of the Hand, 2003, pp 233-251. Hatanaka H, Manske PR: Effect of suture size on locking and grasping flexor tendon repair techniques. Clin Orthop 2000;375:267-274.

275. The advantage of using blocking screws during intramedullary nailing of metaphyseal tibial fractures include all of the following EXCEPT
12345facilitate reduction. maintain reduction. enhance construct stiffness. neutralize translational forces. diminish the risk of nail failure.

PREFERRED RESPONSE: 5 Blocking screws can be placed during IM nailing of tibia fractures to facilitate appropriate nail placement and avoid predicted deformities of proximal and distal fractures. These studies look at tibia fractures treated with blocking screws and small diameter locked IM nails and found that blocking screws (orig described by Poller) can increase primary stability in metaphyseal fractures of both the proximal and distal tibia. They therefore, can help facilitate reduction, maintain reduction, enhance construct stiffness, and neutralize translational forces. There has not been a study which demonstrates that they diminish the risk of nail failure. RECOMMENDED READINGS: Krettek C, Stephan C, Schandelmaier P, et al: The use of Poller screws as blocking screws in stabilising tibial fractures treated with small diameter intramedullary nails. J Bone Joint Surg Br 1999;81:963-968. Ricci WM, OBoyle M, Borrelli J, et al: Fractures of the proximal third of the tibial shaft treated with intramedullary nails and blocking screws. J Orthop Trauma 2001;15:264-270. Krettek C, Miclau T, Schandelmaier P, et al: The mechanical effect of blocking screws (Poller screws) in stabilizing tibia fractures with short proximal or distal fragments after insertion of small-diameter intramedullary nails. J Orthop Trauma 1999;13:550-553.

S-ar putea să vă placă și